You are on page 1of 63

VISIONIAS

www.visionias.in

Test Booklet Series

TEST BOOKLET

GENERAL STUDIES (P) 2019 – Test – 2652


C
Time Allowed: Two Hours Maximum Marks: 200

INSTRUCTIONS

1. IMMEDIATELY AFTER THE COMMENCEMENT OF THE EXAMINATION, YOU SHOULD CHECK THAT THIS BOOKLET
DOES NOT HAVE ANY UNPRINTED OR TURN OR MISSING PAGES OR ITEMS, ETC. IF SO, GET IT REPLACED BY A
COMPLETE TEST BOOKLET.

2. ENCODE CLEARLY THE TEST BOOKLET SERIES A, B, C OR D AS THE CASE MAY BE IN THE APPROPRIATE PLACE IN
THE ANSWER SHEET.

3. You have to enter your Roll Number on the Test Booklet in the Box
provided alongside. Do NOT write anything else on the Test Booklet.

4. This Test Booklet contains 100 items (Questions). Each item is printed in English. Each item comprises four
responses (answers). You will select the response which you want to mark on the Answer Sheet. In case you
feel that there is more than one correct response with you consider the best. In any case, choose ONLY ONE
response for each item.

5. You have to mark all your responses ONLY on the separate Answer Sheet provided. See direction in the
answers sheet.

6. All items carry equal marks. Attempt all items. Your total marks will depend only on the number of correct
responses marked by you in the answer sheet. For every incorrect response 1/3rdof the allotted marks will be
deducted.

7. Before you proceed to mark in the Answer sheet the response to various items in the Test booklet, you have to
fill in some particulars in the answer sheets as per instruction sent to you with your Admission Certificate.

8. After you have completed filling in all responses on the answer sheet and the examination has concluded, you
should hand over to Invigilator only the answer sheet. You are permitted to take away with you the Test
Booklet.

9. Sheet for rough work are appended in the Test Booklet at the end.

DO NOT OPEN THIS BOOKLET UNTIL YOU ARE ASKED TO DO SO


1 www.visionias.in ©Vision IAS
1. Which of the following powers is/are 4. The Model Code of Conduct comes
enjoyed by Prime Minister in relation to the immediately into effect:
Parliament?
(a) once the Election Commission
1. He can dissolve the Lok Sabha at any
announces the schedule of elections.
time.
2. The President summons and prorogues (b) once the notification calling electorate to

sessions of the Parliament on his advice. elect members of a house is issued.


Select the correct answer using the code (c) always at the first date of the month in
given below. which elections are to be conducted.
(a) 1 only
(d) once the candidates start filing their
(b) 2 only
nominations in the constituencies.
(c) Both 1 and 2
(d) Neither 1 nor 2
5. With reference to elections in India, which
2. With reference to Panchayats (Extension to of the following is not correct?
the Scheduled Areas) Act, consider the (a) No person is ineligible for inclusion in
following statements: the electoral roll on grounds only of
1. It extends the provisions of Part IX of
religion, race, caste and sex.
the Constitution to both fifth and sixth
(b) Provisions related to preparation of
scheduled areas.
2. It empowers the Gram Sabha to electoral rolls can be made both by
safeguard and preserve the traditions and Parliament and state legislature.
customs of the people. (c) Orders issued by the Delimitation
Which of the statements given above is/are
Commission can be questioned in a
correct?
court of law.
(a) 1 only
(b) 2 only (d) Prisoners convicted of heinous crimes

(c) Both 1 and 2 cannot vote in the Lok Sabha elections.


(d) Neither 1 nor 2
6. The President consults the Chief Justice of
3. Consider the following statements: India in the case of appointment of judges of
1. The Constitution declares Delhi as the
1. Supreme Court
seat of the Supreme Court.
2. High Court
2. The Constitution authorises the Chief
Justice of India to appoint other place or 3. Subordinate Courts

places as seat of the Supreme Court. Select the correct answer using the code
Which of the statements given above is/are given below.
correct? (a) 1 and 2 only
(a) 1 only
(b) 1 only
(b) 2 only
(c) 1 and 3 only
(c) Both 1 and 2
(d) Neither 1 nor 2 (d) 2 and 3 only
2 www.visionias.in ©Vision IAS
1. संसद के संबंध में प्रधानमंत्री द्वारा ननम्ननिनखत में से 4. अदशभ अचार संनहता तत्काि प्रभावी हो जाती है:
ककस/ककन शनि/शनियों का ईपभोग ककया जाता है? (a) चुनाव अयोग द्वारा चुनावों की समय-सारणी की
1. वह ककसी भी समय िोकसभा को भंग कर सकता घोषणा ककये जाने के पश्चात्।
है। (b) मतदाताओं से सदन के सदस्यों के ननवाभचन का
2. राष्ट्रपनत ईसकी सिाह पर संसद के सत्रों को अह्वान करने वािी ऄनधसूचना जारी होने के
अहूत करता है और सत्रावसान करता है।
पश्चात्।
नीचे कदए गए कू ट का प्रयोग कर सही ईत्तर चुननए।
(c) नजस महीने में चुनाव अयोनजत ककए जाने हैं,
(a) के वि 1
सदैव ईस महीने की पहिी तारीख से।
(b) के वि 2
(d) ईम्मीदवारों द्वारा ननवाभचन क्षेत्रों में ऄपना
(c) 1 और 2 दोनों
नामांकन दर्भ करना अरं भ करने के पश्चात्।
(d) न तो 1, न ही 2

5. भारत में चुनावों के संदभभ में, ननम्ननिनखत में से कनन-


2. पंचायतों के प्रावधान (ऄनुसूनचत क्षेत्रों पर नवस्तार)
सा सही नहीं है?
ऄनधननयम के संदभभ में, ननम्ननिनखत कथनों पर
नवचार कीनजए: (a) कोइ भी व्यनि के वि धमभ, मूिवंश, जानत और
1. यह संनवधान के भाग 9 के प्रावधानों का पांचवीं लिग के अधार पर ननवाभचक नामाविी में
और छठीं ऄनुसूची के क्षेत्रों तक नवस्तार करता सनम्मनित ककये जाने के निए ऄपात्र नहीं है।
है।
(b) संसद और राज्य नवधानयका, दोनों के द्वारा
2. यह ग्राम सभा को िोक प्रथाओं और रीनत-
ननवाभचन नामाविी तैयार करने से संबंनधत
ररवाजों की सुरक्षा और संरक्षण करने का
ऄनधकार प्रदान करता है। प्रावधान ककए जा सकते हैं।

ईपयुभि कथनों में से कनन-सा/से सही है/हैं? (c) पररसीमन अयोग द्वारा जारी अदेशों को
न्यायािय में प्र‍नगत ककया जा सकता है।
(a) के वि 1
(d) जघन्य ऄपराधों के दोषी पाए गये कै दी िोकसभा
(b) के वि 2
चुनावों में मतदान नहीं कर सकते हैं।
(c) 1 और 2 दोनों
(d) न तो 1, न ही 2
6. राष्ट्रपनत द्वारा ननम्ननिनखत में से ककस/ककन
न्यायािय/न्यायाियों में न्यायाधीशों की ननयुनि के
3. ननम्ननिनखत कथनों पर नवचार कीनजए:
मामिे में भारत के मुख्य न्यायाधीश से परामशभ ककया
1. संनवधान कदल्िी को सवोच्च न्यायािय की पीठ के
जाता है?
रूप में घोनषत करता है।
2. संनवधान भारत के मुख्य न्यायाधीश को सवोच्च 1. सवोच्च न्यायािय
न्यायािय की पीठ के रूप में ऄन्य स्थान या 2. ईच्च न्यायािय
स्थानों को ननयत करने के निए ऄनधकृ त करता
3. ऄधीनस्थ न्यायािय
है।
नीचे कदए गए कू ट का प्रयोग कर सही ईत्तर चुननए।
ईपयुभि कथनों में से कनन-सा/से सही है/हैं?
(a) के वि 1 और 2
(a) के वि 1
(b) के वि 1
(b) के वि 2
(c) 1 और 2 दोनों (c) के वि 1 और 3

(d) न तो 1, न ही 2 (d) के वि 2 और 3

3 www.visionias.in ©Vision IAS


7. With reference to the office of whip, 10. With reference to Administrative Tribunals,
consider the following statements: consider the following statements:
1. They were conferred constitutional
1. He is appointed by the speaker to ensure
status by 42nd Amendment Act of 1976.
decorum in the house.
2. Parliament has established a Central
2. Disciplinary action can be taken against Administrative Tribunal to adjudicate
the members who do not follow matters related to employment under the
directives of the whip. Central Government.
3. State legislatures are empowered to
Which of the statements given above is/are
establish State Administrative Tribunals
correct? to adjudicate matters related to
(a) 1 only employment under the State
(b) 2 only Government.
(c) Both 1 and 2 Which of the statement given above is/are
correct?
(d) Neither 1 nor 2
(a) 1 and 2 only
(b) 2 and 3 only
8. With reference to Cabinet Committees, (c) 1 and 3 only
consider the following statements: (d) 1, 2 and 3
1. They are extra-constitutional in nature.
2. They are set up by the President. 11. Which among the following tools has/have
the effect of reducing Parliamentary control
3. Only Cabinet ministers are eligible to be
over the executive in India?
their members. 1. Guillotine
Which of the statements given above is/are 2. Delegated Legislation
correct? 3. Adjournment motion
(a) 1 only Select the correct answer using the code
given below.
(b) 2 only
(a) 1 only
(c) 1 and 2 only (b) 1 and 2 only
(d) 2 and 3 only (c) 2 and 3 only
(d) 1, 2 and 3
9. Joint sitting is provided by the Constitution
to resolve a deadlock between the two 12. With reference to inter-state trade and
commerce, consider the following
Houses of Parliament over the passage of a
statements:
bill. Which of the following categories of 1. Constitution provides that trade,
bills can a joint sitting be convened for? commerce and intercourse throughout
1. Ordinary bill the territory of India shall be free
2. Constitutional Amendment bill without any restrictions.
2. Parliament cannot impose restrictions on
3. Money bill
the freedom of trade and commerce
Select the correct answer using the code between the states in any case.
given below. Which of the statements given above is/are
(a) 1 only correct?
(b) 2 and 3 only (a) 1 only
(b) 2 only
(c) 1 and 3 only
(c) Both 1 and 2
(d) 1 and 2 only (d) Neither 1 nor 2
4 www.visionias.in ©Vision IAS
7. सचेतक के पद के संदभभ में, ननम्ननिनखत कथनों पर 10. प्रशासननक ऄनधकरणों के संदभभ में, ननम्ननिनखत
नवचार कीनजए: कथनों पर नवचार कीनजए:
1. ईसे िोकसभा ऄध्यक्ष द्वारा सदन में नश्‍टाचार 1. आन्हें 42वें संशोधन ऄनधननयम,1976 द्वारा
सुनननश्चत करने के निए ननयुि ककया जाता है। संवैधाननक दजाभ प्रदान ककया गया है।
2. सचेतक के ननदेशों का पािन न करने वािे 2. संसद ने कें द्रीय प्रशासननक ऄनधकरण की
सदस्यों के नवरूद्ध ऄनुशासनात्मक कारभ वाइ की स्थापना कें द्र सरकार के ऄधीन ननयोजन से
संबंनधत प्रकरणों का ननपटारा करने के निए की
जा सकती है।
है।
ईपयुभि कथनों में से कनन-सा/से सही है/हैं?
3. राज्य नवधानयकाओं को राज्य सरकार के ऄधीन
(a) के वि 1 ननयोजन से संबंनधत प्रकरणों का ननपटारा करने
(b) के वि 2 के निए राज्य प्रशासननक ऄनधकरण स्थानपत
करने का ऄनधकार है।
(c) 1 और 2 दोनों
ईपयुभि कथनों में से कनन-सा/से सही है/हैं?
(d) न तो 1, न ही 2 (a) के वि 1 और 2
(b) के वि 2 और 3
8. मंनत्रमंडिीय सनमनतयों के संदभभ में, ननम्ननिनखत (c) के वि 1 और 3
कथनों पर नवचार कीनजए: (d) 1, 2 और 3
1. ये सनमनतयां संनवधानेतर प्रकृ नत की होती हैं।
2. आन सनमनतयों का गठन राष्ट्रपनत द्वारा ककया 11. ननम्ननिनखत में से ककस/ककन साधन/साधनों के
जाता है। प्रभावस्वरूप भारत में कायभपानिका पर संसदीय
3. के वि कै नबनेट मंत्री ही आनके सदस्य बनने के पात्र ननयंत्रण कम हो जाता है?
होते हैं। 1. नगिोरटन
ईपयुभि कथनों में से कनन-सा/से सही है/हैं? 2. प्रत्यायोनजत नवधायन
(a) के वि 1 3. स्थगन प्रस्ताव
नीचे कदए गए कू ट का प्रयोग कर सही ईत्तर चुननए।
(b) के वि 2
(a) के वि 1
(c) के वि 1 और 2
(b) के वि 1 और 2
(d) के वि 2 और 3 (c) के वि 2 और 3
(d) 1, 2 और 3
9. ककसी नवधेयक को पाररत करने को िेकर संसद के
दोनों सदनों के मध्य गनतरोध के समाधान हेतु 12. ऄंतराभज्यीय व्यापार और वानणज्य
संनवधान द्वारा संयुि बैठक का प्रावधान ककया गया के संदभभ में ननम्ननिनखत कथनों पर नवचार कीनजए:
है। ननम्ननिनखत में से ककस/ककन श्रेणी/श्रेनणयों के
1. संनवधान के ऄनुसार, नबना ककसी प्रनतबंध
नवधेयकों के निए संयुि बैठक अहूत की जा सकती
के भारत के राज्यक्षेत्र में सवभत्र व्यापार, वानणज्य
है?
और समागम ऄबाध होगा।
1. साधारण नवधेयक 2. संसद ककसी भी नस्थनत में राज्यों के मध्य व्यापार
2. संनवधान संशोधन नवधेयक और वानणज्य की स्वतंत्रता पर ननबंधन
3. धन नवधेयक ऄनधरोनपत नहीं कर सकती है।
नीचे कदए गए कू ट का प्रयोग कर सही ईत्तर चुननए। ईपयुभि कथनों में से कनन-सा/से सही है/हैं?
(a) के वि 1 (a) के वि 1
(b) के वि 2 और 3 (b) के वि 2
(c) के वि 1 और 3 (c) 1 और 2 दोनों
(d) के वि 1 और 2 (d) न तो 1, न ही 2
5 www.visionias.in ©Vision IAS
13. Which of the following will necessarily 16. Local government in India is
occur on the resignation or death of an (a) a subject under the union list.
incumbent Prime Minister?
(b) a subject under the concurrent list.
(a) Lok Sabha will be dissolved
automatically. (c) a subject under the state list.
(b) The Lok Sabha will elect the new Prime (d) within the residuary power of the union.
Minister.
(c) Council of Ministers will be dissolved
17. Consider the following statements:
automatically.
(d) The senior most leader of his party will 1. The district judge does not have
be appointed as Prime Minister appellate jurisdiction in criminal matters.
automatically.
2. The sessions judge has power to impose

14. Consider the following statements: capital punishment.


1. The President usually appoints the Which of the statements given above is/are
seniormost member of the Lok Sabha as
correct?
the Speaker Pro Tem.
(a) 1 only
2. The President administers oath to the
Speaker Pro Tem. (b) 2 only
3. The office of the Pro Tem Speaker is (c) Both 1 and 2
temporary so it has limited powers.
(d) Neither 1 nor 2
Which of the statements given above is/are
correct?
(a) 1 only 18. With reference to the Union Budget in India,
(b) 1 and 2 only consider the following statements:
(c) 2 and 3 only
1. The term 'Budget' is not mentioned in
(d) 1, 2 and 3
the Constitution.
15. With reference to State Legislature, consider 2. The Rajya Sabha has no power to vote
the following statements:
on the demand for grants.
1. There is a uniform system of bicameral
legislatures across all the states. 3. No demand for a grant shall be made

2. Minimum strength of any state assembly except on the recommendation of the


cannot go below 60 members. President.
3. Legislative assembly of any state must
Which of the statements given above is/are
have only directly elected members.
Which of the statements given above is/are correct?
not correct? (a) 3 only
(a) 1 and 2 only
(b) 1 and 2 only
(b) 2 and 3 only
(c) 1 and 3 only (c) 1 and 3 only
(d) 1, 2 and 3 (d) 1, 2 and 3

6 www.visionias.in ©Vision IAS


13. पदासीन प्रधानमंत्री के त्यागपत्र देने या ईनकी मृत्यु 16. भारत में स्थानीय शासन है:
होने पर ननम्ननिनखत में से कनन-सी प्रकिया अव‍यक (a) संघ सूची के ऄंतगभत एक नवषय।
रूप से संपन्न होगी?
(b) समवती सूची के ऄंतगभत एक नवषय।
(a) िोकसभा स्वतः भंग हो जाएगी।
(c) राज्य सूची के ऄंतगभत एक नवषय।
(b) िोकसभा नए प्रधानमंत्री का चुनाव करे गी।
(d) संघ की ऄवनशष्ट शनियों के ऄधीन।
(c) मंनत्रपररषद स्वतः भंग हो जाएगी।
(d) ईसकी पाटी का वररष्ठतम नेता स्वतः प्रधानमंत्री
17. ननम्ननिनखत कथनों पर नवचार कीनजए:
ननयुि हो जाएगा।
1. नजिा न्यायाधीश के पास फनजदारी मामिों में

14. ननम्ननिनखत कथनों पर नवचार कीनजए: ऄपीिीय क्षेत्रानधकार नहीं होता है।

1. राष्ट्रपनत सामान्यत: िोकसभा के वररष्ठतम 2. सत्र न्यायाधीश को मृत्युदड


ं देने की ऄनधकाररता
सदस्य को प्रोटेम स्पीकर ननयुि करता है। प्राप्त होती है।
2. प्रोटेम स्पीकर को शपथ राष्ट्रपनत कदिाता है।
ईपयुभि कथनों में से कनन-सा/से सही है/हैं?
3. प्रोटेम स्पीकर का पद ऄस्थायी होता है, आसनिए
(a) के वि 1
ईसके पास सीनमत शनियां होती हैं।
ईपयुभि कथनों में से कनन-सा/से सही है/हैं? (b) के वि 2

(a) के वि 1 (c) 1 और 2 दोनों


(b) के वि 1 और 2
(d) न तो 1, न ही 2
(c) के वि 2 और 3

(d) 1, 2 और 3 18. भारत में संघीय बजट के संदभभ में, ननम्ननिनखत कथनों

पर नवचार कीनजए:
15. राज्य नवधानमंडिों के संदभभ में, ननम्ननिनखत कथनों
1. संनवधान में 'बजट' शब्द का ईल्िेख नहीं ककया
पर नवचार कीनजए:
1. सभी राज्यों में नद्वसदनीय नवधानमंडि की गया है।

एकसमान व्यवस्था नवद्यमान है। 2. राज्यसभा को ऄनुदान की मांग पर मतदान की


2. ककसी भी राज्य नवधानसभा की न्यूनतम सदस्य कोइ शनि प्राप्त नहीं है।
संख्या 60 सदस्यों से कम नहीं हो सकती है। 3. राष्ट्रपनत की नसफाररश के नबना ककसी ऄनुदान
3. ककसी भी राज्य की नवधानसभा में के वि प्रत्यक्ष की मांग नहीं की जा सकती।
रूप से ननवाभनचत सदस्य होने चानहए।
ईपयुभि कथनों में से कनन-सा/से सही है/हैं?
ईपयुभि कथनों में से कनन-सा/से सही नहीं है/हैं?
(a) के वि 3
(a) के वि 1 और 2
(b) के वि 1 और 2
(b) के वि 2 और 3

(c) के वि 1 और 3 (c) के वि 1 और 3

(d) 1, 2 और 3 (d) 1, 2 और 3
7 www.visionias.in ©Vision IAS
19. "The pocket of the Indian President is bigger 22. If the President wants to seek legal opinion
than that of American President." Which on a matter of public importance from the
among the following statements justifies the Supreme Court, then:
remark?
(a) Supreme Court can assume to itself the
(a) Indian President can withhold his assent
power of legislation.
to a bill while American President
cannot. (b) President has to accept the advice given
(b) Indian President can return the bill for by the court.
reconsideration of Parliament while (c) Supreme Court may refuse to tender its
American President cannot. opinion.
(c) Indian President enjoys absolute veto (d) President has to first get the approval of
while American President does not.
the Parliament for the same.
(d) Indian President can keep the bill
pending for indefinite period while
American President has to return the bill 23. Which of the following Parliamentary
to the Congress within 10 days. committees are chaired by the Speaker?
1. Public Accounts Committee
20. Which of the following electoral reforms 2. Business advisory Committee
have been implemented to make the 3. Ethics Committee
elections more fair and transparent? 4. Rules Committee
1. Equitable sharing of air time on 5. General Purpose Committee
television by political parties.
Select the correct answer using the codes
2. Reporting of contribution in excess of
given below.
Rs 1,000 to the Election Commission.
3. Declaration of criminal antecedents by (a) 1 and 3 only
the candidates. (b) 2, 4 and 5 only
Select the correct answer using the code (c) 1, 4 and 5 only
given below. (d) 2 and 3 only
(a) 1 and 2 only
(b) 2 and 3 only
24. In which of the following cases a member
(c) 1 and 3 only
(d) 1, 2 and 3 incurs disqualification under the Tenth
Schedule?
21. Which of the following is/are correct with 1. If he votes contrary to the direction
reference to the Cantonment Board? given by his political party.
1. It is established for municipal 2. If any nominated member joins any
administration for civilian population in political party before the expiry of six
the cantonment area. months.
2. It works under the administrative control
3. If he is expelled from the political
of the Union Ministry of Home Affairs.
party to which he belonged immediately
3. It consists of partly elected and partly
nominated members. before the election.
Select the correct answer using the code Select the correct answer using the code
given below. given below.
(a) 1 only (a) 1 only
(b) 2 and 3 only (b) 1 and 2 only
(c) 1 and 3 only
(c) 3 only
(d) 1, 2 and 3
(d) 1, 2 and 3
8 www.visionias.in ©Vision IAS
19. "भारतीय राष्ट्रपनत की जेब ऄमेररकी राष्ट्रपनत की 22. यकद राष्ट्रपनत ईच्चतम न्यायािय से व्यापक महत्व व के
तुिना में बडी है।" ननम्ननिनखत में से कनन-सा कथन ककसी नवषय पर नवनधक राय िेना चाहता है तो:

आस रटप्पणी को सही नसद्ध करता है? (a) ईच्चतम न्यायािय स्वयं नवनध ननमाभण की शनि
धारण कर सकता है।
(a) भारतीय राष्ट्रपनत नवधेयक पर ऄपनी सहमनत
रोक सकता है जबकक ऄमेररकी राष्ट्रपनत नहीं। (b) न्यायािय द्वारा कदया गया परामशभ राष्ट्रपनत के
(b) भारतीय राष्ट्रपनत नवधेयक को पुनर्ववचार के निए निए बाध्यकारी होता है।
संसद को वापस भेज सकता है जबकक ऄमेररकी (c) ईच्चतम न्यायािय ऄपनी राय देने से मना कर
राष्ट्रपनत नहीं। सकता है।
(c) भारतीय राष्ट्रपनत अत्यांनतक वीटो का ईपयोग (d) आसके निए राष्ट्रपनत को पहिे संसद की स्वीकृ नत
कर सकता है जबकक ऄमेररकी राष्ट्रपनत नहीं। प्राप्त करना अव‍यक है।
(d) भारतीय राष्ट्रपनत ऄनननश्चत ऄवनध तक नवधेयक
िंनबत रख सकता है जबकक ऄमेररकी राष्ट्रपनत
23. ननम्ननिनखत में से ककन संसदीय सनमनतयों की
को 10 कदनों के भीतर नवधेयक कांग्रेस को वापस
ऄध्यक्षता िोकसभा ऄध्यक्ष द्वारा की जाती है?
िनटाना होता है।
1. िोक िेखा सनमनत

20. चुनावों को ऄनधक नन्‍पक्ष और पारदशी बनाने के 2. कायभ मंत्रणा सनमनत


निए ननम्ननिनखत में से कनन-से चुनाव सुधार 3. अचार सनमनत
कायाभनन्वत ककए गए हैं? 4. ननयम सनमनत
1. राजनीनतक दिों के निए टेिीनवजन पर प्रसारण 5. सामान्य प्रयोजन सनमनत
समय का न्यायसंगत अवंटन। नीचे कदए गए कू ट का प्रयोग कर सही ईत्तर चुननए।
2. ननवाभचन अयोग को 1,000 रुपये से ऄनधक के (a) के वि 1 और 3
ऄंशदान की सूचना।
(b) के वि 2, 4 और 5
3. ईम्मीदवारों द्वारा अपरानधक पूवभवृत्त की
(c) के वि 1, 4 और 5
घोषणा।
नीचे कदए गए कू ट का प्रयोग कर सही ईत्तर चुननए। (d) के वि 2 और 3
(a) के वि 1 और 2
(b) के वि 2 और 3 24. ननम्ननिनखत में से ककस/ककन नस्थनत/नस्थनतयों में
(c) के वि 1 और 3 ककसी सदस्य को दसवीं ऄनुसच
ू ी के ऄंतगभत ऄयोग्य
(d) 1, 2 और 3 घोनषत ककया जा सकता है?
1. यकद वह ऄपने राजनीनतक दि द्वारा कदए गए
21. छावनी पररषद के संदभभ में ननम्ननिनखत में से कनन- ननदेश के नवरुद्ध मत देता है।
सा/से सही है/हैं? 2. यकद कोइ मनोनीत सदस्य छह माह की समानप्त
1. आसकी स्थापना छावनी क्षेत्र में ऄसैन्य जनसंख्या से पूवभ ककसी राजनीनतक दि में शानमि हो जाता
हेतु नगरपानिका प्रशासन के निए की जाती है। है।
2. यह कें द्रीय गृह मंत्रािय के प्रशासननक ननयंत्रण के 3. यकद ईसे ईस राजनीनतक दि से नन्‍कानसत कर
ऄधीन कायभ करती है। कदया जाता है, नजससे वह चुनाव से ठीक पहिे
3. आसमें अंनशक रूप से ननवाभनचत और अंनशक रूप संबंनधत था।
से मनोनीत सदस्य सनम्मनित होते हैं।
नीचे कदए गए कू ट का प्रयोग कर सही ईत्तर चुननए।
नीचे कदए गए कू ट का प्रयोग कर सही ईत्तर चुननए।
(a) के वि 1
(a) के वि 1
(b) के वि 1 और 2
(b) के वि 2 और 3
(c) के वि 1 और 3 (c) के वि 3

(d) 1, 2 और 3 (d) 1, 2 और 3
9 www.visionias.in ©Vision IAS
25. Consider the following statements: 28. For a cut motion to be admissible, it:
1. The advice of the Council of Ministers is 1. should relate to one demand only.
binding on the President. 2. should not make suggestions for the
2. The Indian Constitution provides for the amendment or repeal of existing laws.
system of legal responsibility of a 3. should not raise a question of privilege.
minister. Select the correct answer using the code
Which of the statements given above is/are given below.
correct? (a) 1 and 2 only
(a) 1 only (b) 2 and 3 only
(b) 2 only (c) 1 and 3 only
(c) Both 1 and 2 (d) 1, 2 and 3
(d) Neither 1 nor 2

29. For ascertaining quorum in any house of the


26. With reference to the Indian Parliament, a Parliament, which of the following are taken
'Motion of thanks' is moved for into consideration?
(a) discussion on address given by the 1. Elected members
Speaker to the Lok Sabha upon
2. Nominated members
assuming office.
3. Presiding officer.
(b) discussion on address given by
Select the correct answer using the code
the President in the first session of each
given below.
year.
(a) 1 and 2 only
(c) discussion on address given by the
(b) 2 and 3 only
President to Parliament after a general
(c) 1 and 3 only
election.
(d) 1, 2 and 3
(d) Both 'b' and 'c' are correct.

30. With reference to Deputy Chairman of Rajya


27. In which of the following cases can the
Sabha, consider the following statements:
President exercise the discretionary powers?
1. To be elected as Deputy Chairman, one
1. When he sends back advice given by the
must not be less than 30 years of age
Council of Ministers and asks the
2. Whenever the office of the Deputy
council to reconsider the decision.
Chairman falls vacant, President
2. When he withholds or refuses to give
appoints the seniormost member to fill
assent to a Money bill passed by the
the vacancy.
Parliament.
3. Being subordinate to the Chairman,
3. When he dismisses the council of
ministers after it cannot prove the Deputy Chairman is directly responsible

confidence of the Lok Sabha. to Chairman as well as Rajya Sabha.


Select the correct answer using the code Which of the statements given above is/are
given below. correct?
(a) 1 and 3 only (a) 1 only
(b) 2 only (b) 1 and 2 only
(c) 2 and 3 only (c) 2 and 3 only
(d) 1, 2 and 3 (d) 1 and 3 only
10 www.visionias.in ©Vision IAS
25. ननम्ननिनखत कथनों पर नवचार कीनजए: 28. सदन में चचाभ हेतु स्वीकृ नत योग्य होने के निए, ककसी
1. मंनत्रपररषद का परामशभ राष्ट्रपनत के निए कटनती प्रस्ताव को:

बाध्यकारी होता है। 1. के वि एक मांग से संबंनधत होना चानहए।


2. वतभमान कानूनों में संशोधन या ननरसन का
2. भारतीय संनवधान एक मंत्री के नवनधक
सुझाव देने वािा नहीं होना चानहए।
ईत्तरदानयत्व की प्रणािी का प्रावधान करता है।
3. नवशेषानधकार का प्र‍न ईठाने वािा नहीं होना
ईपयुभि कथनों में से कनन-सा/से सही है/हैं? चानहए।
नीचे कदए गए कू ट का प्रयोग कर सही ईत्तर चुननए।
(a) के वि 1
(a) के वि 1 और 2
(b) के वि 2
(b) के वि 2 और 3
(c) 1 और 2 दोनों (c) के वि 1 और 3
(d) न तो 1, न ही 2 (d) 1, 2 और 3

26. भारतीय संसद के संदभभ में, 'धन्यवाद प्रस्ताव' िाया 29. संसद के ककसी सदन में गणपूर्वत के ननधाभरण हेतु,
ननम्ननिनखत में से ककन्हें गणना में शानमि ककया जाता
जाता है:
है?
(a) िोकसभा ऄध्यक्ष द्वारा पद धारण करने पर कदये
1. ननवाभनचत सदस्य
गए भाषण पर चचाभ के निए।
2. मनोनीत सदस्य
(b) प्रत्येक वषभ के पहिे सत्र में राष्ट्रपनत द्वारा कदए 3. पीठासीन ऄनधकारी
गए भाषण पर चचाभ के निए। नीचे कदए गए कू ट का प्रयोग कर सही ईत्तर चुननए।
(c) अम चुनाव के बाद राष्ट्रपनत द्वारा संसद में कदए (a) के वि 1 और 2
गए भाषण पर चचाभ के निए। (b) के वि 2 और 3
(d) 'b' और 'c' दोनों सही हैं। (c) के वि 1 और 3
(d) 1, 2 और 3

27. ननम्ननिनखत में से ककस नस्थनत/ककन नस्थनतयों में


राष्ट्रपनत नववेकाधीन शनियों का प्रयोग करता है? 30. राज्यसभा के ईप सभापनत के संदभभ में, ननम्ननिनखत
कथनों पर नवचार कीनजए:
1. जब वह मंनत्रपररषद द्वारा दी गइ सिाह वापस
1. ईप सभापनत के रूप में ननवाभनचत होने के निए,
भेजता है और मंनत्रपररषद को ननणभय पर
न्यूनतम अयु 30 वषभ होनी ऄननवायभ है।
पुनर्ववचार करने के निए कहता है।
2. ईप सभापनत का पद ररक्त होने की नस्थनत में,
2. जब वह संसद द्वारा पाररत धन नवधेयक पर
राष्ट्रपनत ररनि को भरने हेतु सदन के वररष्ठतम
सहमनत रोक िेता है या देने से मना कर देता है। सदस्य को ननयुि करता है।
3. जब वह मंनत्रपररषद द्वारा िोकसभा का नवश्वास 3. सभापनत के ऄधीनस्थ होने के नाते, ईप सभापनत
खोने के बाद ईसे बखाभस्त करता है। प्रत्यक्ष रूप से सभापनत और राज्यसभा के प्रनत
नीचे कदए गए कू ट का प्रयोग कर सही ईत्तर चुननए। ईत्तरदायी होता है।
ईपयुभि कथनों में से कनन-सा/से सही है/हैं?
(a) के वि 1 और 3
(a) के वि 1
(b) के वि 2
(b) के वि 1 और 2
(c) के वि 2 और 3
(c) के वि 2 और 3
(d) 1, 2 और 3 (d) के वि 1 और 3
11 www.visionias.in ©Vision IAS
31. Which of the following qualifications are 33. Consider the following statements:
laid down by Constitution and the 1. Governor's assent is not required when
the bill is reserved for the consideration
Parliament, for a person to be chosen a
of the President.
member of the Parliament?
2. If a state's bill, returned by the President,
1. He must be a citizen of India. is passed again by the state legislature
2. He must be registered as an elector in after reconsideration, President must
the parliamentary constituency from give his assent to the bill.
Which of the statements given above is/are
where he wishes to contest the elections.
correct?
3. He must be a member of a scheduled (a) 1 only
caste or scheduled tribe in any state or (b) 2 only
union territory if he wants to contest a (c) Both 1 and 2
(d) Neither 1 nor 2
seat reserved for them.
Select the correct answer using the code
34. Consider the following statements:
given below. 1. The civil servants are conferred personal
(a) 1 only immunity from legal liability for official
(b) 2 and 3 only contracts.
2. The judicial officers enjoy immunity
(c) 1 and 3 only
from any liability in respect of their
(d) 1, 2 and 3
official acts.
Which of the statements given above is/are
32. Consider the following pairs: not correct?
Person Post Held (a) 1 only
1. Sachidanand : First Speaker of (b) 2 only
(c) Both 1 and 2
Sinha the central
(d) Neither 1 nor 2
legislative
assembly 35. With reference to the special status granted
2. Vithalbhai J. : First elected to the state of Jammu and Kashmir, which of
Patel Speaker of the the following statements is/are correct?
1. Jammu and Kashmir is the only State in
central legislative
India to have its own separate
assembly constitution.
3. G V : First Speaker of 2. None of the provisions of the Indian

Mavalankar the Lok Sabha constitution are applicable to the state of


Jammu and Kashmir.
Which of the pairs given above is/are
Select the correct answer using the code
correctly matched?
given below.
(a) 1 and 2 only (a) 1 only
(b) 2 and 3 only (b) 2 only
(c) 1 and 3 only (c) Both 1 and 2
(d) Neither 1 nor 2
(d) 1, 2 and 3
12 www.visionias.in ©Vision IAS
31. ककसी व्यनि को संसद का सदस्य चुने जाने के निए 33. ननम्ननिनखत कथनों पर नवचार कीनजए:

संनवधान और संसद द्वारा ननम्ननिनखत में से कनन सी 1. ककसी नवधेयक को राष्ट्रपनत हेतु नवचार के निए
अरनक्षत रखे जाने की नस्थनत में ईस पर
योग्यताएं ननधाभररत की गइ है/हैं? राज्यपाि की सहमनत की अव‍यकता नहीं होती
है।
1. ईसे भारत का नागररक होना चानहए।
2. यकद राष्ट्रपनत द्वारा िनटाया गया राज्य का
2. ईसे ईस संसदीय ननवाभचन क्षेत्र में मतदाता के नवधेयक पुनर्ववचार के बाद राज्य नवधानयका
द्वारा पुन: पाररत कर कदया जाता है, तो राष्ट्रपनत
रूप में पंजीकृ त होना चानहए, जहां से वह चुनाव
को नवधेयक पर ऄपनी सहमनत देना ऄननवायभ है।
िडना चाहता है। ईपयुभि कथनों में से कनन-सा/से सही है/हैं?
3. ऄनुसूनचत जानतयों या ऄनुसूनचत जनजानतयों के (a) के वि 1

निए अरनक्षत सीट पर चुनाव िडने के निए ईसे (b) के वि 2

ककसी राज्य या संघ राज्यक्षेत्र में ऄनुसूनचत जानत (c) 1 और 2 दोनों


(d) न तो 1, न ही 2
या ऄनुसूनचत जनजानत का सदस्य होना चानहए।

नीचे कदए गए कू ट का प्रयोग कर सही ईत्तर चुननए।


34. ननम्ननिनखत कथनों पर नवचार कीनजए:
(a) के वि 1
1. िोक सेवकों को ऄपने अनधकाररक कायों के
नवनधक ईत्तरदानयत्व से व्यनिगत ईन्मुनि प्रदान
(b) के वि 2 और 3
की गइ है।
(c) के वि 1 और 3 2. न्यानयक ऄनधकारी को ऄपने अनधकाररक कृ त्यों
के संबंध में ककसी भी ईत्तरदानयत्व से ईन्मुनि
(d) 1, 2 और 3 प्रदान की गयी है।
ईपयुभि कथनों में से कनन-सा/से सही नहीं है/हैं?
(a) के वि 1
32. ननम्ननिनखत युग्मों पर नवचार कीनजए:
(b) के वि 2
व्यनि : धारण ककया गया पद
(c) 1 और 2 दोनों
1. सनच्चदानंद : कें द्रीय नवधानसभा के प्रथम ऄध्यक्ष (d) न तो 1, न ही 2
नसन्हा

2. नवठ्ठिभाइ : कें द्रीय नवधानसभा के प्रथम 35. जम्मू-क‍मीर राज्य को प्रदान ककए गए नवशेष दजे के
संदभभ में, ननम्ननिनखत में से कनन-सा/से कथन सही
जे. पटेि ननवाभनचत ऄध्यक्ष
है/हैं?
3. जी.वी. : िोकसभा के प्रथम ऄध्यक्ष
1. जम्मू-क‍मीर भारत का एकमात्र राज्य है नजसका
माविंकर
ऄपना ऄिग संनवधान है।
ईपयुभि युग्मों में से कनन-से सही सुमेनित हैं? 2. भारतीय संनवधान का कोइ भी प्रावधान जम्मू-
क‍मीर राज्य पर िागू नहीं होता है।
(a) के वि 1 और 2 नीचे कदए गए कू ट का प्रयोग कर सही ईत्तर चुननए।
(a) के वि 1
(b) के वि 2 और 3
(b) के वि 2
(c) के वि 1 और 3 (c) 1 और 2 दोनों

(d) 1, 2 और 3 (d) न तो 1, न ही 2

13 www.visionias.in ©Vision IAS


36. With reference to the leader of opposition, 39. Who among the following allocates ranks as
consider the following statements: well as portfolios to ministers?
1. The office of leader of opposition has no
(a) President on advice of Prime Minister
statutory recognition.
(b) Cabinet Secretary
2. He is appointed only in the Lok Sabha
3. He is the leader of the largest opposition (c) Parliamentary Affairs Committee
party having 10 percent or more seats of (d) Appointments Committee of the Cabinet
the total strength of the House.
Which of the statements given above 40. According to the Constitution of India,
is/are correct? which among the following are the grounds
(a) 1 and 2 only
of removal for a Supreme Court judge?
(b) 1 only
1. Proved misbehavior
(c) 2 and 3 only
(d) 3 only 2. Incapacity
3. Violation of constitution
37. With regard to the judge of a High Court, 4. Corruption
consider the following statements: 5. Insolvency
1. He can be removed from his office by
Select the correct answer using the code
the Governor on the recommendation of
given below.
the President.
2. He can resign from his office by writing (a) 2 and 5 only
to the President. (b) 1, 3, 4 and 5 only
Which of the statements given above is/are (c) 1, 2, 3 and 4 only
correct?
(d) 1 and 2 only
(a) 1 only
(b) 2 only
41. What are the main objectives of 'E-
(c) Both 1 and 2
(d) Neither 1 nor 2 Governance'?
1. Empowering people through
38. Consider the following statements: information.
1. The President can transfer a judge from 2. Improved efficiency within
one high court to another after
Governments.
consulting the Chief Ministers of the
3. Better service delivery to the citizens.
respective states.
2. The transfer of judges from one high 4. Ushering in transparency and
court to another is not subject to judicial accountability.
review. Select the correct answer using the code
Which of the statements given above is/are given below.
correct?
(a) 1 and 3 only
(a) 1 only
(b) 2, 3 and 4 only
(b) 2 only
(c) Both 1 and 2 (c) 1, 2, 3 and 4
(d) Neither 1 nor 2 (d) 2 and 4 only
14 www.visionias.in ©Vision IAS
36. नवपक्ष के नेता के संदभभ में, ननम्ननिनखत कथनों पर 39. ननम्ननिनखत में से कनन मंनत्रयों को पद और नवभाग
नवचार कीनजए: अवंरटत करता है?
1. नवपक्ष के नेता के पद को कोइ सांनवनधक मान्यता
प्राप्त नहीं है। (a) प्रधानमंत्री के परामशभ पर राष्ट्रपनत

2. वह के वि िोकसभा में ननयुि ककया जाता है। (b) कै नबनेट सनचव


3. वह ऐसे सबसे बडे नवपक्षी दि का नेता होता है
(c) संसदीय मामिों की सनमनत
नजसे सदन की कु ि सदस्य संख्या का 10 प्रनतशत
या ऄनधक सीटें प्राप्त हुइ हों। (d) कै नबनेट की ननयुनि सनमनत
ईपयुभि कथनों में से कनन-सा/से सही है/हैं?
(a) के वि 1 और 2
40. भारत के संनवधान के ऄनुसार, ननम्ननिनखत में से
(b) के वि 1
कनन-से ईच्चतम न्यायािय के न्यायाधीश को पद से
(c) के वि 2 और 3
हटाने के अधार हैं?
(d) के वि 3
1. सानबत कदाचार

37. ईच्च न्यायािय के न्यायाधीश के संबंध में, ननम्ननिनखत 2. ऄसमथभता


कथनों पर नवचार कीनजए: 3. संनवधान का ईल्िंघन
1. राष्ट्रपनत की ऄनुशस
ं ा पर राज्यपाि द्वारा ईसे
4. भ्रष्टाचार
ईसके पद से हटाया जा सकता है।
2. वह राष्ट्रपनत को निनखत संबोधन के माध्यम से 5. कदवानिया होना
त्यागपत्र दे सकता है। नीचे कदए गए कू ट का प्रयोग कर सही ईत्तर चुननए।
ईपयुभि कथनों में से कनन-सा/से सही है/हैं? (a) के वि 2 और 5
(a) के वि 1
(b) के वि 1, 3, 4 और 5
(b) के वि 2
(c) के वि 1, 2, 3 और 4
(c) 1 और 2 दोनों
(d) न तो 1, न ही 2 (d) के वि 1 और 2

38. ननम्ननिनखत कथनों पर नवचार कीनजए: 41. 'इ-गवनेंस' के मुख्य ईद्दे‍य क्या हैं?
1. राष्ट्रपनत संबंनधत राज्यों के मुख्यमंनत्रयों से
परामशभ करने के पश्चात् ककसी न्यायाधीश का 1. सूचना के जररए िोगों का सशनिकरण।
एक ईच्च न्यायािय से दूसरे ईच्च न्यायािय को 2. सरकार के भीतर बेहतर दक्षता।
ऄन्तरण कर सकता है।
3. नागररकों को बेहतर सेवा प्रदायगी।
2. एक ईच्च न्यायािय का दूसरे ईच्च न्यायािय में
न्यायाधीशों का ऄन्तरण न्यानयक समीक्षा के 4. पारदर्वशता और जवाबदेही का अरं भ।
ऄधीन नहीं है। नीचे कदए गए कू ट का प्रयोग कर सही ईत्तर चुननए।
ईपयुभि कथनों में से कनन-सा/से सही है/हैं?
(a) के वि 1 और 3
(a) के वि 1
(b) के वि 2, 3 और 4
(b) के वि 2
(c) 1 और 2 दोनों (c) 1, 2, 3 और 4

(d) न तो 1, न ही 2 (d) के वि 2 और 4

15 www.visionias.in ©Vision IAS


42. Consider the following statements: 45. With regard to principle of collective
1. The representatives of states in the Rajya responsibility in the Indian Parliamentary
Sabha are elected only by the elected
system of government, consider the
members of state legislative assemblies.
following statements:
2. The representatives of each union
territory in the Rajya Sabha are 1. All the ministers of Council of Ministers
nomiated by the Lieutenant governor. own joint responsibility to the Lok
Which of the statements given above is/are Sabha.
correct? 2. Cabinet decisions bind all ministers of
(a) 1 only
the Council of Ministers.
(b) 2 only
Which of the statements given above is/are
(c) Both 1 and 2
(d) Neither 1 nor 2 correct?
(a) 1 only
43. Consider the following statements: (b) 2 only
1. Autonomous bodies are established by (c) Both 1 and 2
the Government to discharge the
(d) Neither 1 nor 2
activities which are related to
governmental functions.
46. With reference to the Constitution of India,
2. Attached offices are responsible for
providing executive direction required in consider the following pairs:
the implementation of the policies laid Subject List
down by the department to which they 1. Cyber Laws Union List
are attached.
2. Agriculture State List
Which of the statements given above is/are
3. Trade Unions Concurrent List
correct?
(a) 1 only Which of the pairs given above is/are
(b) 2 only correctly matched?
(c) Both 1 and 2 (a) 1 only
(d) Neither 1 nor 2 (b) 2 and 3 only
(c) 1, 2 and 3
44. Consider the following statements:
(d) None of the above
1. The laws made by the Parliament are
applicable to Indian citizens and their
property in any part of the world. 47. The provisions in Fifth Schedule and Sixth
2. The laws made by the State legislature Schedule in the Constitution of India are
are not applicable outside the state under made in order to
any circumstances. (a) protect the interests of Scheduled Tribes
Which of the statements given above is/are
(b) determine the boundaries between States
correct?
(c) determine the powers, authority and
(a) 1 only
(b) 2 only responsibilities of Panchayats
(c) Both 1 and 2 (d) protect the interests of all the border
(d) Neither 1 nor 2 States
16 www.visionias.in ©Vision IAS
42. ननम्ननिनखत कथनों पर नवचार कीनजए: 45. सरकार की भारतीय संसदीय प्रणािी में सामूनहक
1. राज्यसभा में राज्यों के प्रनतनननधयों को के वि ईत्तरदानयत्व के नसद्धांत के संबंध में, ननम्ननिनखत
राज्य नवधानसभाओं के ननवाभनचत सदस्यों द्वारा
कथनों पर नवचार कीनजए:
ननवाभनचत ककया जाता है।
1. मंनत्रपररषद के सभी मंनत्रयों का िोकसभा के
2. राज्यसभा में संघ राज्यक्षेत्रों के प्रनतनननधयों को
प्रनत संयुि ईत्तरदानयत्व होता है।
सम्बंनधत ईप-राज्यपािों द्वारा मनोनीत ककया
जाता है। 2. मंनत्रमंडि के ननणभय मंनत्रपररषद के सभी मंनत्रयों

ईपयुभि कथनों में से कनन-सा/से सही है/हैं? के निए बाध्यकारी होते हैं।

(a) के वि 1 ईपयुभि कथनों में से कनन-सा/से सही है/हैं?

(b) के वि 2 (a) के वि 1

(c) 1 और 2 दोनों (b) के वि 2

(d) न तो 1, न ही 2 (c) 1 और 2 दोनों

(d) न तो 1, न ही 2
43. ननम्ननिनखत कथनों पर नवचार कीनजए:
1. सरकार द्वारा सरकारी कायों से संबंनधत
46. भारत के संनवधान के संदभभ में, ननम्ननिनखत युग्मों पर
गनतनवनधयों के नन्‍पादन हेतु स्वायत्त ननकायों
की स्थापना की जाती है। नवचार कीनजए:
2. संिग्न कायाभिय, ऄपने संिग्न नवभाग द्वारा नवषय : सूची

ननधाभररत नीनतयों के कायाभन्वयन हेतु अव‍यक 1. साआबर कानून : संघ सूची


कायभकारी ननदेश प्रदान करने के निए ईत्तरदायी : राज्य सूची
2. कृ नष
होते हैं।
3. ट्रेड यूननयन : समवती सूची
ईपयुभि कथनों में से कनन-सा/से सही है/हैं?

(a) के वि 1 ईपयुभि युग्मों में से कनन-सा/से सही सुमेनित है/हैं?

(b) के वि 2 (a) के वि 1

(c) 1 और 2 दोनों (b) के वि 2 और 3


(d) न तो 1, न ही 2 (c) 1, 2 और 3

(d) ईपयुभि में से कोइ नहीं


44. ननम्ननिनखत कथनों पर नवचार कीनजए:
1. संसद द्वारा ननर्वमत कानून नव‍व के ककसी भी
47. भारत के संनवधान की पांचवीं ऄनुसच
ू ी और छठी
भाग में नस्थत भारतीय नागररकों और ईनकी
ऄनुसूची में प्रावधान ककए गए हैं;
संपनत्त पर िागू होते हैं।
2. राज्य नवधानमंडि द्वारा ननर्वमत कानून ककसी भी (a) ऄनुसूनचत जनजानतयों के नहतों की रक्षा करने के
पररनस्थनत में राज्य से बाहर िागू नहीं होते हैं। निए
ईपयुभि कथनों में से कनन-सा/से सही है/हैं? (b) राज्यों के मध्य सीमा ननधाभररत करने के निए
(a) के वि 1 (c) पंचायतों की शनियां, ऄनधकार और
(b) के वि 2 ईत्तरदानयत्व ननधाभररत करने के निए
(c) 1 और 2 दोनों (d) सभी सीमावती राज्यों के नहतों की रक्षा करने के
(d) न तो 1, न ही 2 निए

17 www.visionias.in ©Vision IAS


48. Consider the following statements: 51. On premature dissolution of panchayat after
1. The President may entrust to the state 2 years of its constitution:
government any of the executive
functions of the Centre without the (a) It will be reconstituted to serve a full
consent of that state. term of 5 years
2. The Governor of a state may entrust to (b) No fresh elections shall take place for
the Central government any of the
executive functions of the state with the remainder of the term
consent of the Centre. (c) It will be reconstituted to serve
Which of the statements given above is/are remainder of the period
correct?
(a) 1 only (d) State legislative assembly shall decide
(b) 2 only the course of action
(c) Both 1 and 2
(d) Neither 1 nor 2
52. "e-Governance facilitates interaction

49. With reference to the appointment of Prime between different stakeholders in


Minister of India, consider the following governance". Which of the following
statements:
interaction models does exist in e-
1. Before his appointment as Prime
Minister, he must prove his majority in Governance ?
the Lok Sabha. 1. G2G (Government to Government)
2. At the time of appointment, he must be a
2. G2E (Government to Employees)
member of either House of Parliament.
Which of the statements given above is/are 3. G2C (Government to Citizens)
correct? Select the correct answer using the code
(a) 1 only
given below.
(b) 2 only
(c) Both 1 & 2 (a) 2 only
(d) Neither 1 nor 2 (b) 1 only
(c) 1 and 3 only
50. With reference to political parties as
(d) 1, 2 and 3
recognized by the Election Commission
(EC), consider the following statements:
1. EC recognizes parties as national parties 53. Under Article 371, the President of India
or state parties on the basis of their
mandates which of the following to have
membership.
2. The recognition granted by EC to special responsibility for the establishment
political parties determines their right to of separate development boards for the states
certain privileges in elections.
of Maharashtra and Gujarat?
Which of the statements given above is/are
correct? (a) Governors of the respective states
(a) 1 only (b) Prime Minister
(b) 2 only
(c) State Legislatures of the respective states
(c) Both 1 and 2
(d) Neither 1 nor 2 (d) Parliament

18 www.visionias.in ©Vision IAS


48. ननम्ननिनखत कथनों पर नवचार कीनजए: 51. गठन के 2 वषभ पश्चात् पंचायत के समय पूवभ नवघटन
1. राष्ट्रपनत ककसी राज्य की सहमनत के नबना कें द्र होने पर:
सरकार के ककसी भी कायभकारी कृ त्यों को ईस
(a) 5 वषभ की पूणभ ऄवनध तक कायभ करने के निए
राज्य सरकार को सौंप सकता है।
2. राज्य का राज्यपाि कें द्र की सहमनत से ईस राज्य आसका पुनगभठन ककया जाता है।
के ककसी भी कायभकारी कृ त्यों को कें द्र सरकार को (b) शेष कायाभवनध के निए कोइ नया चुनाव नहीं
सौंप सकता है।
होता है।
ईपयुभि कथनों में से कनन-सा/से सही है/हैं?
(c) शेष कायाभवनध तक कायभ करने के निए आसका
(a) के वि 1
पुनगभठन ककया जाता है।
(b) के वि 2
(d) राज्य नवधानसभा कायभवाही का ननधाभरण करती
(c) 1 और 2 दोनों
है।
(d) न तो 1, न ही 2

52. "इ-गवनेंस” शासन में नवनभन्न नहतधारकों के मध्य


49. भारत के प्रधानमंत्री की ननयुनि के संदभभ में,
ऄंतरकिया को सुगम बनाता है"। इ-गवनेंस में
ननम्ननिनखत कथनों पर नवचार कीनजए:
1. प्रधानमंत्री के रूप में ऄपनी ननयुनि से पहिे ईसे ननम्ननिनखत में से कनन-सा/से ऄंतःकिया मॉडि
िोकसभा में ऄपना बहुमत नसद्ध करना पडता है। नवद्यमान है/हैं?
2. ननयुनि के समय ईसे संसद के दोनों सदनों में से
1. G2G (सरकार से सरकार)
एक का सदस्य होना चानहए।
ईपयुभि कथनों में से कनन-सा/से सही है/हैं? 2. G2E (सरकार से कमभचारी)

(a) के वि 1 3. G2C (सरकार से नागररक)


(b) के वि 2 नीचे कदए गए कू ट का प्रयोग कर सही ईत्तर चुननए।
(c) 1 और 2 दोनों (a) के वि 2
(d) न तो 1, न ही 2
(b) के वि 1

50. ननवाभचन अयोग द्वारा मान्यता प्राप्त राजनीनतक दिों (c) के वि 1 और 3

के संदभभ में, ननम्ननिनखत कथनों पर नवचार कीनजए: (d) 1, 2 और 3


1. ननवाभचन अयोग, दिों को ईनकी सदस्यता के
अधार पर राष्ट्रीय या राज्य दिों के रूप में 53. ऄनुच्छेद 371 के ऄंतगभत, भारत का राष्ट्रपनत महाराष्ट्र
मान्यता प्रदान करता है।
और गुजरात राज्यों के निए पृथक नवकास बोडों की
2. ननवाभचन अयोग द्वारा राजनीनतक दिों को प्रदत्त
मान्यता चुनाव में कु छ नवशेषानधकारों के प्रनत स्थापना का नवशेष ईत्तरदानयत्व ननम्ननिनखत में से
ईनके ऄनधकार को ननधाभररत करती है। ककसे ऄनधदेनशत करता है?
ईपयुभि कथनों में से कनन-सा/से सही है/हैं?
(a) संबंनधत राज्यों के राज्यपाि को
(a) के वि 1
(b) प्रधानमंत्री को
(b) के वि 2
(c) संबंनधत राज्यों के नवधानमंडिों को
(c) 1 और 2 दोनों

(d) न तो 1, न ही 2 (d) संसद को


19 www.visionias.in ©Vision IAS
54. Consider the following statements: 57. Which of the following committees made
1. The salaries of members of parliament recommendations with respect to Centre-
are determined by the President. State relations?
2. There is no provision of pension in the 1. Sarkaria Commission
Constitution for members of Parliament. 2. M M Punchhi Commission
Which of the statements given above is/are
3. Rajamannar Committee
correct?
Select the correct answer using the code
(a) 1 only
given below.
(b) 2 only
(a) 1 and 2 only
(c) Both 1 and 2
(b) 2 and 3 only
(d) Neither 1 nor 2
(c) 1 and 3 only

55. In the context of Governance, consider the (d) 1, 2 and 3


following regarding the 'Sevottam' model:
1. It focuses on the effective 58. With reference to the Municipal
implementation of Citizen's Charter. Corporation, consider the following
2. It has been created by the Department of statements:
Administrative Reforms and Public 1. It is established for the administration of
Grievances. towns.
Which of the statements given above is/are 2. It is established in the states by an act of
correct? the concerned state legislature.
(a) 1 only
3. The Mayor is the chief executive
(b) 2 only
authority of the corporation.
(c) Both 1 and 2
Which of the statements given above is/are
(d) Neither 1 nor 2
correct?
(a) 1 and 2 only
56. Which among the following categories of
bills would lapse upon the dissolution of the (b) 3 only

Lok Sabha? (c) 1 and 3 only


1. A bill passed by the Lok Sabha but (d) 2 only
pending in the Rajya Sabha.
2. A bill pending in the Rajya Sabha but 59. Which of the following is not a criterion for
not passed by the Lok Sabha. being appointed as District Judge?
3. A bill passed by both Houses but (a) He should be recommended by High
pending assent of the President. Court.
Select the correct answer using the code (b) He should not be less than 35 years old.
given below. (c) He should not already be in the service
(a) 1 only
of the central or state government.
(b) 1 and 3 only
(d) He should have been an advocate or
(c) 1, 2 and 3
pleader for seven years.
(d) 3 only

20 www.visionias.in ©Vision IAS


54. ननम्ननिनखत कथनों पर नवचार कीनजए: 57. ननम्ननिनखत में से ककन सनमनतयों द्वारा कें द्र-राज्य
1. संसद सदस्यों का वेतन राष्ट्रपनत द्वारा ननधाभररत संबंधों के संदभभ में ऄनुशस
ं ाएं की गयी हैं?
ककया जाता है।
1. सरकाररया अयोग
2. संनवधान में संसद सदस्यों के निए पेंशन का कोइ
2. एम. एम. पुंछी अयोग
प्रावधान नहीं है।
ईपयुभि कथनों में से कनन-सा/से सही है/हैं? 3. राजमन्नार सनमनत
नीचे कदए गए कू ट का प्रयोग कर सही ईत्तर चुननए।
(a) के वि 1
(a) के वि 1 और 2
(b) के वि 2

(c) 1 और 2 दोनों (b) के वि 2 और 3

(d) न तो 1, न ही 2 (c) के वि 1 और 3

(d) 1, 2 और 3
55. गवनेंस के संदभभ में, 'सेवोत्तम' मॉडि के संबंध में
ननम्ननिनखत कथनों पर नवचार कीनजए: 58. नगर ननगम के संदभभ में, ननम्ननिनखत कथनों पर
1. यह नसटीजन चाटभर के प्रभावी कायाभन्वयन पर
नवचार कीनजए:
कें कद्रत है।
1. आसकी स्थापना कस्बों के प्रशासन के निए की
2. आसे प्रशासननक सुधार और िोक नशकायत
नवभाग द्वारा बनाया गया था। जाती है।

ईपयुभि कथनों में से कनन-सा/से सही है/हैं? 2. राज्यों में आसकी स्थापना संबंनधत राज्य
नवधानमंडि के ऄनधननयम द्वारा की जाती है।
(a) के वि 1
3. महापनर नगर ननगम का मुख्य कायभकारी
(b) के वि 2
प्रानधकारी होता है।
(c) 1 और 2 दोनों
ईपयुभि कथनों में से कनन-सा/से सही है/हैं?
(d) न तो 1, न ही 2
(a) के वि 1 और 2

(b) के वि 3
56. िोकसभा का नवघटन होने पर ननम्ननिनखत में से
ककस/ककन श्रेणी/श्रेनणयों के नवधेयक व्यपगत हो जाते (c) के वि 1 और 3
हैं?
(d) के वि 2
1. िोकसभा द्वारा पाररत परं तु राज्यसभा में िंनबत
नवधेयक।
59. ननम्ननिनखत में से कनन-सा नजिा न्यायाधीश के रूप
2. राज्यसभा में िंनबत परं तु िोकसभा द्वारा पाररत
नहीं ककया गया नवधेयक। में ननयुि होने के निए एक मानदंड नहीं है?

3. दोनों सदनों द्वारा पाररत परं तु राष्ट्रपनत की (a) ईसे ईच्च न्यायािय द्वारा ऄनुशंनसत होना
सहमनत के निए िंनबत नवधेयक। चानहए।
नीचे कदए गए कू ट का प्रयोग कर सही ईत्तर चुननए।
(b) ईसे 35 वषभ से कम अयु का नहीं होना चानहए।
(a) के वि 1
(c) ईसे पहिे से ही कें द्र या राज्य सरकार की सेवा में
(b) के वि 1 और 3
नहीं होना चानहए।
(c) 1, 2 और 3
(d) ईसे सात वषों तक ऄनधवक्ता या प्िीडर होना
(d) के वि 3
चानहए।
21 www.visionias.in ©Vision IAS
60. With reference to election to the Parliament, 62. With reference to Inter-State River Water
Disputes Act, consider the following
which of the following statements
statements:
is not correct? 1. The Supreme Court or any other court
(a) If member of one House gets elected to does not have jurisdiction in respect of
any water dispute referred to a tribunal.
other House, his seat in first House
2. The Parliament has power to set up
becomes vacant. inter-state water dispute tribunals.
(b) If a person is elected to both the Houses Which of the statements given above is/are
correct?
of Parliament, the seat in Rajya Sabha
(a) 1 only
becomes vacant if he fails to declare (b) 2 only
(c) Both 1 and 2
which House he desires to serve.
(d) Neither 1 nor 2
(c) If a person is elected to two seats in a
House, his both seats become vacant if 63. With reference to centre state financial
he fails to declare which seat he wants to relations, which of the following net
proceeds of taxes not form a part of
represent. Consolidated Fund of India?
(d) If a person is elected to Parliament and 1. Taxes on the consignment of goods in
the course of inter-state trade or
State legislature both, his seat in State
commerce.
legislature becomes vacant by default. 2. Stamp duties on bills of exchange and
policies of insurance.
3. Taxes on agricultural income.
61. To regulate media from influencing voter
Select the correct answer using the code
behaviour during elections, which of the given below.
following provisons is/are mentioned in the (a) 1 only
(b) 2 and 3 only
Representation of People Act, 1951?
(c) 1 and 3 only
1. It prohibits displaying any election (d) 1, 2 and 3
matter by means of television during
64. With regard to the First Past the Post system
polling of votes.
of elections in India, which of the following
2. It prohibits the conduct of exit polls at statements is/are correct?
any time before or after the voting at 1. In this system each constituency elects
one representative.
polling stations.
2. The winning candidate in this system
Select the correct answer using the code must secure more than fifty percent
votes.
given below.
Select the correct answer using the code
(a) 1 only given below.
(b) 2 only (a) 1 only
(b) 2 only
(c) Both 1 and 2
(c) Both 1 and 2
(d) Neither 1 nor 2 (d) Neither 1 nor 2

22 www.visionias.in ©Vision IAS


60. संसद के चुनावों के संदभभ में ननम्ननिनखत में से कनन- 62. ऄंतराभज्यीय नदी जि नववाद ऄनधननयम के संदभभ में,
सा कथन सही नहीं है? ननम्ननिनखत कथनों पर नवचार कीनजए:
1. ककसी न्यायानधकरण को सौंपा गया कोइ जि
(a) यकद एक सदन का सदस्य दूसरे सदन के निए
नववाद, ईच्चतम न्यायािय तथा ककसी ऄन्य
ननवाभनचत हो जाता है, तो पहिे सदन में ईसकी
न्यायािय के क्षेत्रानधकार से बाहर होता है।
सीट ररक्त हो जाती है। 2. ऄंतराभज्यीय जि नववाद न्यायानधकरण के गठन
(b) यकद कोइ व्यनि संसद के दोनों सदनों के निए की शनि संसद में नननहत है।

ननवाभनचत होता है, तो राज्यसभा में सीट ररक्त ईपयुभि कथनों में से कनन-सा/से सही है/हैं?
(a) के वि 1
हो जाती है यकद वह यह घोनषत करने में नवफि
रहता है कक वह ककस सदन में सेवा करना चाहता (b) के वि 2

है। (c) 1 और 2 दोनों

(c) यकद कोइ व्यनि सदन में दो सीटों के निए (d) न तो 1, न ही 2

ननवाभनचत होता है, तो ईसकी दोनों सीटें ररक्त


63. कें द्र-राज्य नवत्तीय संबंधों के संदभभ में, ननम्ननिनखत में
हो जाती हैं यकद वह यह घोनषत करने में नवफि
से ककन करों की शुद्ध प्रानप्तयां भारत की संनचत नननध
रहता है कक वह ककस सीट का प्रनतनननधत्व करना
का भाग नहीं होती हैं?
चाहता है।
1. ऄंतराभज्यीय व्यापार या वानणज्य के दनरान
(d) यकद कोइ व्यनि संसद और राज्य नवधानमंडि
वस्तुओं के परे षण पर कर।
दोनों के निए ननवाभनचत होता है, तो राज्य 2. नवननमय पत्रों और बीमा पॉनिनसयों पर स्टाम्प
नवधानमंडि में ईसकी सीट स्वतः ररक्त हो जाती शुल्क।
है। 3. कृ नष अय पर कर।
नीचे कदए गए कू ट का प्रयोग कर सही ईत्तर चुननए।
(a) के वि 1
61. चुनावों के दनरान मीनडया को मतदाता व्यवहार
प्रभानवत करने से रोकने हेतु जन प्रनतनननधत्व (b) के वि 2 और 3

ऄनधननयम, 1951 में ननम्ननिनखत में से ककस/ककन (c) के वि 1 और 3


(d) 1, 2 और 3
प्रावधान/प्रावधानों का ईल्िेख ककया गया है?

1. यह मतदान के दनरान ककसी भी चुनाव संबंधी


64. भारत में चुनाव की ‘फस्टभ पास्ट द पोस्ट’ प्रणािी के
प्रकरण को टेिीनवजन के माध्यम से प्रदर्वशत
संबंध में ननम्ननिनखत में से कनन-सा/से कथन सही
करना नननषद्ध करता है। है/हैं?
2. यह मतदान कें द्रों पर मतदान से पहिे या बाद में 1. आस प्रणािी में प्रत्येक ननवाभचन क्षेत्र से एक
ककसी भी समय एनक्जट पोि के संचािन को प्रनतनननध ननवाभनचत होता है।
नननषद्ध करता है। 2. आस प्रणािी में जीतने वािे प्रत्याशी को पचास
नीचे कदए गए कू ट का प्रयोग कर सही ईत्तर चुननए। प्रनतशत से ऄनधक मत प्राप्त करने होते हैं।
नीचे कदए गए कू ट का प्रयोग कर सही ईत्तर चुननए।
(a) के वि 1
(a) के वि 1
(b) के वि 2
(b) के वि 2
(c) 1 और 2 दोनों (c) 1 और 2 दोनों
(d) न तो 1, न ही 2 (d) न तो 1, न ही 2
23 www.visionias.in ©Vision IAS
65. Consider the following statements: 68. With reference to the District Planning
1. The Rajya Sabha is a permanent body. Committee (DPC), consider the following
2. The Constitution has fixed the term of statements:
1. The 74th Amendment Act provides for
office of members of Rajya Sabha as 6
the establishment of DPC.
years. 2. All members of the DPC are nominated
Which of the statements given above is/are by the Governor of the state.
correct? 3. It has representation of both rural and
(a) 1 only urban population of the district.
Which of the statements given above is/are
(b) 2 only
correct?
(c) Both 1 and 2
(a) 1 only
(d) Neither 1 nor 2 (b) 2 and 3 only
(c) 1 and 3 only
66. Which of the following is/are correct with (d) 1, 2 and 3
reference to the election of members and
chairpersons of panchayats under the 69. Which of the following statements is/are
correct with regard to Committee on Public
Constitution (73rd Amendment) Act, 1992?
Undertakings?
1. Members of panchayats at all levels are
1. The members are elected every year
elected directly by the people. from both the houses of the parliament.
2. Chairpersons of panchayats at all levels 2. The chairman of this Committee can be
are elected indirectly by and from only from Lok sabha.
amongst members of panchayats. Select the correct answer using the code
given below.
Select the correct answer using the code
(a) 1 only
given below. (b) 2 only
(a) 1 only (c) Both 1 and 2
(b) 2 only (d) Neither 1 nor 2
(c) Both 1 and 2
(d) Neither 1 nor 2 70. Consider the following statements with
reference to the speaker of Lok Sabha:
1. The resolution for removal of Speaker
67. Which among the following provisions
has to be passed by special majority.
ensure impartial functioning of the Supreme 2. The work and conduct of the Speaker
Court of India? cannot be discussed or
1. Separation of judiciary from executive 3. criticised in the Lok Sabha except on a
2. Expenses not charged on Consolidated substantive motion.
4. The powers of Speaker regarding
Fund of India.
maintenance of order in the House are
3. Its jurisdiction cannot be curtailed.
not subject to jurisdiction of any court.
Select the correct answer using the code Which of the statements given above is/are
given below. not correct?
(a) 1 and 2 only (a) 1 only
(b) 2 and 3 only (b) 2 only
(c) 1 and 3 only
(c) 1 and 3 only
(d) 2 and 3 only
(d) 1, 2 and 3
24 www.visionias.in ©Vision IAS
65. ननम्ननिनखत कथनों पर नवचार कीनजए: 68. नजिा योजना सनमनत (DPC) के संदभभ में,
1. राज्यसभा एक स्थायी सदन है। ननम्ननिनखत कथनों पर नवचार कीनजए:
2. संनवधान में राज्यसभा के सदस्यों की पदावनध 6 1. 74वां संशोधन ऄनधननयम DPC के गठन का
वषभ ननधाभररत की गयी है। प्रावधान करता है।

ईपयुभि कथनों में से कनन-सा/से सही है/हैं? 2. DPC के सभी सदस्यों को राज्य के राज्यपाि
द्वारा मनोनीत ककया जाता है।
(a) के वि 1
3. आसमें नजिे की ग्रामीण और नगरीय दोनों
(b) के वि 2 जनसंख्या के प्रनतनननध शानमि होते हैं।
(c) 1 और 2 दोनों ईपयुभि कथनों में से कनन-सा/से सही है/हैं?

(d) न तो 1, न ही 2 (a) के वि 1
(b) के वि 2 और 3

66. संनवधान (73वां संशोधन) ऄनधननयम, 1992 के (c) के वि 1 और 3


तहत पंचायत के सदस्यों और ऄध्यक्षों के ननवाभचन के (d) 1, 2 और 3
संदभभ में, ननम्ननिनखत में से कनन-सा/से कथन सही

है/हैं? 69. सावभजननक ईपिम सनमनत के संदभभ में ननम्ननिनखत में


से कनन-सा/से कथन सही है/हैं?
1. सभी स्तरों पर पंचायत के सदस्यों को िोगों
द्वारा प्रत्यक्ष रूप से ननवाभनचत ककया जाता है। 1. आसके सदस्य प्रनत वषभ संसद के दोनों सदनों से
ननवाभनचत ककए जाते हैं।
2. सभी स्तरों पर पंचायतों के ऄध्यक्षों को पंचायत
2. आस सनमनत का ऄध्यक्ष के वि िोकसभा से ही हो
के सदस्यों द्वारा ईन्हीं में से ऄप्रत्यक्ष रूप से
सकता है।
ननवाभनचत ककया जाता है।
नीचे कदए गए कू ट का प्रयोग कर सही ईत्तर चुननए।
नीचे कदए गए कू ट का प्रयोग कर सही ईत्तर चुननए।
(a) के वि 1
(a) के वि 1
(b) के वि 2
(b) के वि 2
(c) 1 और 2 दोनों
(c) 1 और 2 दोनों
(d) न तो 1, न ही 2
(d) न तो 1, न ही 2

70. िोकसभा ऄध्यक्ष के संदभभ में, ननम्ननिनखत कथनों पर


67. ननम्ननिनखत में से कनन-से प्रावधान भारत के ईच्चतम नवचार कीनजए:
न्यायािय की नन्‍पक्ष कायभपद्धनत को सुनननश्चत करते
1. ऄध्यक्ष को पद से हटाने से संबनं धत संकल्प को
हैं? नवशेष बहुमत से पाररत ककया जाना चानहए।
1. न्यायपानिका का कायभपानिका से पृथक्करण। 2. स्वतंत्र या मननिक प्रस्ताव के ऄनतररि ईसके
2. व्ययों का भारत की संनचत नननध पर भाररत न कायभ व अचरण की िोकसभा में चचाभ एवं
अिोचना नहीं की जा सकती है।
होना।
3. सदन में व्यवस्था बनाए रखने के संबंध में ऄध्यक्ष
3. आसके क्षेत्रानधकार को कम नहीं ककया जा सकता
की शनियां ककसी न्यायािय के ऄनधकार क्षेत्र के
है ।
ऄधीन नहीं हैं।
नीचे कदए गए कू ट का प्रयोग कर सही ईत्तर चुननए।
ईपयुभि कथनों में से कनन-सा/से सही नहीं है/हैं?
(a) के वि 1 और 2
(a) के वि 1
(b) के वि 2 और 3
(b) के वि 2
(c) के वि 1 और 3
(c) के वि 1 और 3
(d) 1, 2 और 3 (d) के वि 2 और 3
25 www.visionias.in ©Vision IAS
71. With reference to the Central Council of 74. With reference to the 'Zero hour' in the
Local Government, consider the following Indian Parliament, consider the following
statements: statements:
1. It is an informal device available to the
1. It was constituted under Article 263 of
members of the Parliament to raise
the Constitution of India. matters without any prior notice.
2. It deals with both urban as well as rural 2. It is a feature borrowed from the
local governments. British Constitution.
3. The Union Minister for Housing and 3. It is elaborated in the rules of procedure
Urban Affairs acts as its chairman. of the Houses of Parliament.
Which of the statements given above is/are
Which of the statements given above is/are
correct?
correct? (a) 3 only
(a) 1 only (b) 1 and 2 only
(b) 2 and 3 only (c) 1 only
(c) 1 and 3 only (d) 1, 2 and 3
(d) 1, 2 and 3
75. With reference to Delimitation Act, 2002,
consider the following statements:
72. The Eleventh Schedule of the Constitution
1. The total number of seats allotted to
places which of the following under the each state in Lok Sabha is based on the
purview of panchayats? population data of 1971 census.
1. Agriculture 2. The number of reserved seats for
2. Khadi, village and cottage industries scheduled castes and scheduled tribes is
fixed on the basis of the census data of
3. Slum improvement and upgradation
2001.
4. Public Distribution System
Which of the statements given above is/are
Select the correct answer using the code correct?
given below. (a) 1 only
(a) 1 and 2 only (b) 2 only
(b) 3 and 4 only (c) Both 1 and 2
(d) Neither 1 nor 2
(c) 1, 2 and 4 only
(d) 2, 3 and 4 only
76. With reference to Legislative Councils,
consider the following statements:
73. With reference to Gram Nyayalayas, 1. For the abolition or creation of
consider the following statements: legislative councils in states, a resolution
1. They are established for every panchayat must be passed by the state assembly by
at the village level under the supervision a special majority.
2. Union Territories cannot have a
of Gram Sabha.
legislative council.
2. They exercise the powers of both
3. Governor can nominate one member of
Criminal and Civil courts. Anglo-Indian community in legislative
Which of the statements given above is/are councils.
correct? Which of the statements given above is/are
(a) 1 only correct?
(a) 1 only
(b) 2 only
(b) 2 and 3 only
(c) Both 1 and 2 (c) 1 and 2 only
(d) Neither 1 nor 2 (d) 1 and 3 only

26 www.visionias.in ©Vision IAS


71. कें द्रीय स्थानीय शासन पररषद के संदभभ में, 74. भारतीय संसद में 'शून्यकाि' के संदभभ में, ननम्ननिनखत
ननम्ननिनखत कथनों पर नवचार कीनजए: कथनों पर नवचार कीनजए:
1. आसका गठन भारत के संनवधान के ऄनुच्छेद 263 1. यह संसद सदस्यों के निए ईपिब्ध एक
ऄननपचाररक साधन है, नजसमें सदस्य नबना
के तहत ककया गया था।
ककसी पूवभ सूचना के मामिे को ईठा सकते हैं।
2. यह नगरीय और ग्रामीण दोनों स्थानीय शासनों
2. आस नवशेषता को निरटश संनवधान से ग्रहण ककया
से संबंनधत है। गया है।
3. अवास और शहरी मामिों का कें द्रीय मंत्री आसके 3. संसद के सदनों के प्रकिया नवषयक ननयमों में
ऄध्यक्ष के रूप में कायभ करता है। आसका सनवस्तार वणभन ककया गया है।
ईपयुभि कथनों में से कनन-सा/से सही है/हैं? ईपयुभि कथनों में से कनन-सा/से सही है/हैं?

(a) के वि 1 (a) के वि 3 (b) के वि 1 और 2


(c) के वि 1 (d) 1, 2 और 3
(b) के वि 2 और 3

(c) के वि 1 और 3
75. पररसीमन ऄनधननयम, 2002 के संदभभ में ननम्ननिनखत
(d) 1, 2 और 3 कथनों पर नवचार कीनजए:
1. िोकसभा में प्रत्येक राज्य को अवंरटत सीटों की
72. संनवधान की ग्यारहवीं ऄनुसच
ू ी के तहत ननम्ननिनखत कु ि संख्या 1971 की जनगणना के जनसंख्या
में से ककन नवषयों को पंचायतों की पररनध के ऄंतगभत अंकडों पर अधाररत है।
2. ऄनुसूनचत जानतयों और ऄनुसूनचत जनजानतयों
सनम्मनित ककया गया है?
के निए अरनक्षत सीटों की संख्या 2001 की
1. कृ नष
जनगणना के अंकडों के अधार पर ननधाभररत की
2. खादी, ग्रामोद्योग और कु टीर ईद्योग गइ है।
3. मनिन बस्ती सुधार और ईन्नयन ईपयुभि कथनों में से कनन-सा/से सही है/हैं?

4. सावभजननक नवतरण प्रणािी (a) के वि 1


नीचे कदए गए कू ट का प्रयोग कर सही ईत्तर चुननए। (b) के वि 2
(a) के वि 1 और 2 (c) 1 और 2 दोनों

(b) के वि 3 और 4 (d) न तो 1, न ही 2

(c) के वि 1, 2 और 4
76. नवधान पररषदों के संदभभ में, ननम्ननिनखत कथनों पर
(d) के वि 2, 3, और 4
नवचार कीनजए:
1. राज्यों में नवधान पररषदों के ईत्सादन या सृजन
73. ग्राम न्यायाियों के संदभभ में, ननम्ननिनखत कथनों पर के निए, संबंनधत प्रस्ताव को राज्य नवधानसभा
नवचार कीनजए: द्वारा नवशेष बहुमत से पाररत ककया जाना
चानहए।
1. आन्हें ग्राम स्तर पर प्रत्येक पंचायत के निए ग्राम
2. कें द्र शानसत प्रदेशों में नवधान पररषद नहीं हो
सभा के पयभवक्ष
े ण में स्थानपत ककया जाता है।
सकती है।
2. आन्हें अपरानधक और दीवानी दोनों न्यायाियों 3. राज्यपाि, नवधान पररषद में अंग्ि-भारतीय
की शनियााँ प्राप्त हैं। समुदाय के एक सदस्य को मनोनीत कर सकता
ईपयुभि कथनों में से कनन-सा/से सही है/हैं? है।
(a) के वि 1 ईपयुभि कथनों में से कनन-सा/से सही है/हैं?
(a) के वि 1
(b) के वि 2
(b) के वि 2 और 3
(c) 1 और 2 दोनों
(c) के वि 1 और 2
(d) न तो 1, न ही 2
(d) के वि 1 और 3
27 www.visionias.in ©Vision IAS
77. Which among the following factors limit the 80. Consider the following statements:
sovereignty of the Indian Parliament? 1. The Zonal Councils are established by
1. Presence of a written Constitution.
the States Reorganisation Act of 1956.
2. Incorporation of fundamental rights in
2. The Parliamentary Affairs Minister acts
the Constitution.
3. Veto powers of the President. as a chairman of all the zonal councils.
Select the correct answer using the code Which of the statements given above is/are
given below. correct?
(a) 1 and 2 only (a) 1 only
(b) 2 and 3 only
(b) 2 only
(c) 1 and 3 only
(c) Both 1 and 2
(d) 1, 2 and 3
(d) Neither 1 nor 2
78. Consider the following statements:
1. The grants made by the Parliament to the 81. With reference to allocation of taxation
states who are in need of financial powers between Centre and States, consider
assistance are charged on the the following statements:
Consolidated Fund of India every year.
1. The Parliament has exclusive power to
2. Both the Centre and the states are
empowered to make grants for any levy taxes on subject enumerated in the
public purpose within their respective Union and Concurrent lists.
legislative competence only. 2. The residuary power of taxation is
Which of the statements given above is/are vested in both the Parliament and the
correct?
state legislatures.
(a) 1 only
Which of the statements given above is/are
(b) 2 only
(c) Both 1 and 2 correct?
(d) Neither 1 nor 2 (a) 1 only
(b) 2 only
79. With reference to the evolution of (c) Both 1 and 2
Panchayati Raj in India, consider the (d) Neither 1 nor 2
following statements:
1. The Balwant Rai Mehta Committee
recommended establishment of a two- 82. The Constitution has specified which of the
tier Panchayati Raj system. following as languages of the Union?
2. Rajasthan was the first state in 1. Hindi (Devanagari Script)
independent India to establish 2. English
Panchayati Raj.
3. All languages mentioned in the eighth
3. The Panchayati Raj institutions got
schedule.
constitutional status in 1982.
Which of the statements given above is/are Select the correct answer using the code
correct? given below.
(a) 1 and 2 only (a) 1 only
(b) 2 only (b) 2 and 3 only
(c) 1 and 3 only
(c) 1, 2 and 3
(d) 1, 2 and 3
(d) 1 and 2 only
28 www.visionias.in ©Vision IAS
77. ननम्ननिनखत में से कनन-से कारक भारतीय संसद की 80. ननम्ननिनखत कथनों पर नवचार कीनजए:
संप्रभुता को सीनमत करते हैं?
1. क्षेत्रीय पररषदों का गठन राज्य पुनगभठन
1. निनखत संनवधान की ईपनस्थनत। ऄनधननयम, 1956 द्वारा ककया गया है।
2. संनवधान में मूि ऄनधकारों का समावेश।
2. संसदीय मामिों का मंत्री सभी क्षेत्रीय पररषदों के
3. राष्ट्रपनत की वीटो शनियां।
ऄध्यक्ष के रूप में कायभ करता है।
नीचे कदए गए कू ट का प्रयोग कर सही ईत्तर चुननए।
(a) के वि 1 और 2 ईपयुभि कथनों में से कनन-सा/से सही है/हैं?

(b) के वि 2 और 3 (a) के वि 1

(c) के वि 1 और 3 (b) के वि 2
(d) 1, 2 और 3 (c) 1 और 2 दोनों

(d) न तो 1, न ही 2
78. ननम्ननिनखत कथनों पर नवचार कीनजए:
81. कें द्र और राज्यों के मध्य कराधान शनियों के अबंटन
1. नवत्तीय सहायता की अव‍यकता वािे राज्यों को
संसद द्वारा कदया जाने वािा ऄनुदान प्रत्येक वषभ के संदभभ में, ननम्ननिनखत कथनों पर नवचार कीनजए:
भारत की संनचत नननध पर भाररत होता है।
1. संसद को संघ और समवती सूनचयों में ईनल्िनखत
2. कें द्र और राज्य दोनों को ककसी भी िोक प्रयोजन
नवषयों पर करारोपण की ऄनन्य शनि प्राप्त है।
के निए के वि ऄपनी संबंनधत नवधायी सामर्थयभ के
भीतर ही ऄनुदान प्रदान करने की शनि प्राप्त है। 2. कराधान की ऄवनशष्ट शनि संसद और राज्य

ईपयुभि कथनों में से कनन-सा/से सही है/हैं? नवधानमंडिों दोनों में नननहत है।

(a) के वि 1 ईपयुभि कथनों में से कनन-सा/से सही है/हैं?

(b) के वि 2 (a) के वि 1

(c) 1 और 2 दोनों (b) के वि 2


(d) न तो 1, न ही 2 (c) 1 और 2 दोनों

(d) न तो 1, न ही 2
79. भारत में पंचायती राज के ईद्भव के संदभभ में,
ननम्ननिनखत कथनों पर नवचार कीनजए:
82. संनवधान द्वारा ननम्ननिनखत में से ककसे/ककन्हें संघ की
1. बिवंत राय मेहता सनमनत ने नद्व-स्तरीय
पंचायती राज व्यवस्था की संस्तुनत की थी। भाषा के रूप में ननर्ददष्ट ककया गया है?
2. राजस्थान पंचायती राज की स्थापना करने वािा
1. नहन्दी (देवनागरी निनप)
स्वतंत्र भारत का प्रथम राज्य था।
2. ऄंग्रेजी
3. पंचायती राज संस्था को वषभ 1982 में
3. अठवीं ऄनुसच
ू ी में ईनल्िनखत सभी भाषाएं
संवैधाननक दजाभ प्राप्त हुअ।
नीचे कदए गए कू ट का प्रयोग कर सही ईत्तर चुननए।
ईपयुभि कथनों में से कनन-सा/से सही है/हैं?
(a) के वि 1
(a) के वि 1 और 2

(b) के वि 2 (b) के वि 2 और 3

(c) के वि 1 और 3 (c) 1, 2 और 3

(d) 1, 2 और 3 (d) के वि 1 और 2
29 www.visionias.in ©Vision IAS
83. Supreme Court is a Court of Record. It 86. Who among the following has the power to
implies that appoint acting judges of High Court?
1. The judgements of Supreme Court are
(a) Chief Justice of India
recorded for perpetual memory.
2. The Supreme Court has the power to (b) Chief Justice of the concerned High
punish for contempt of court. Court
Select the correct answer using the code
(c) Governor
given below.
(a) 1 only (d) President of India
(b) 2 only
(c) Both 1 and 2 87. With reference to Right To Information Act,
(d) Neither 1 nor 2
2005 consider the following statements:

84. With reference to the Parliament of India, 1. The information seeker is required to
consider the following statements: give reason for requesting the
1. The original Constitution of India information.
adopted the names 'Rajya Sabha' and
2. Non-governmental bodies are not
'Lok Sabha' for the two houses of the
Parliament. covered under the ambit of the act.
2. Both the President and the Vice Which of the statements given above is/are
President constitute a part of the
correct?
Parliament.
Select the correct answer using the code (a) 1 only
given below. (b) 2 only
(a) 1 only (c) Both 1and 2
(b) 2 only
(d) Neither 1 nor 2
(c) Both 1 and 2
(d) Neither 1 nor 2
88. Which of the following parameters is/are
85. With reference to Permanent Lok Adalats, used to divide the country into electoral
which of the following statements is/are
constituencies for elections to the Lok
correct?
1. They are established to deal with cases Sabha?
pertaining to public utility services. 1. Underdevelopment
2. Its success is mainly dependent on the
2. National Security
settlement between aggrieved parties
outside the courts. 3. Population
3. It seeks to reduce the workload of the Select the correct answer using the code
regular courts. given below.
Select the correct answer using the code
(a) 1 only
given below.
(a) 1 and 2 only (b) 2 and 3 only
(b) 2 and 3 only (c) 3 only
(c) 1 and 3 only
(d) 1, 2 and 3
(d) 1, 2 and 3
30 www.visionias.in ©Vision IAS
83. ईच्चतम न्यायािय एक ऄनभिेख न्यायािय है, आसका 86. ननम्ननिनखत में से ककसे ईच्च न्यायािय के कायभकारी
तात्पयभ हैः
न्यायाधीशों को ननयुि करने की शनि प्राप्त है?
1. ईच्चतम न्यायािय के ननणभय सावभकानिक
ऄनभिेख के रूप में रखे जाते हैं। (a) भारत के मुख्य न्यायाधीश को
2. ईच्चतम न्यायािय को न्यायािय की ऄवमानना
(b) संबंनधत ईच्च न्यायािय के मुख्य न्यायाधीश को
के निए दण्ड देने की शनि प्राप्त है।
नीचे कदए गए कू ट का प्रयोग कर सही ईत्तर चुननए। (c) राज्यपाि को
(a) के वि 1
(d) भारत के राष्ट्रपनत को
(b) के वि 2
(c) 1 और 2 दोनों
87. सूचना का ऄनधकार ऄनधननयम, 2005 के संदभभ में
(d) न तो 1, न ही 2
ननम्ननिनखत कथनों पर नवचार कीनजए:
84. भारत की संसद के संदभभ में ननम्ननिनखत कथनों पर 1. सूचना मांगने वािे व्यनि को सूचना मांगने का
नवचार कीनजए: कारण बताना अव‍यक है।
1. भारत के मूि संनवधान में संसद के दोनों सदनों
2. गैर-सरकारी ननकाय आस ऄनधननयम के दायरे में
के निए 'राज्य सभा' और 'िोक सभा' नाम को
नहीं अते हैं।
ऄपनाया गया था।
2. राष्ट्रपनत और ईपराष्ट्रपनत दोनों ही संसद के ऄंग ईपयुभि कथनों में से कनन-सा/से सही है/हैं?
हैं।
(a) के वि 1
नीचे कदए गए कू ट का प्रयोग कर सही ईत्तर चुननए।
(a) के वि 1 (b) के वि 2

(b) के वि 2
(c) 1 और 2 दोनों
(c) 1 और 2 दोनों
(d) न तो 1, न ही 2
(d) न तो 1, न ही 2

85. स्थायी िोक ऄदाितों के संदभभ में, ननम्ननिनखत में से 88. िोकसभा चुनावों हेतु देश को ननवाभचन क्षेत्रों में

कनन-से कथन सही हैं? नवभानजत करने के निए ननम्ननिनखत में से ककस/ककन
1. आन्हें सावभजननक ईपयोनगता सेवाओं से संबंनधत मापदंड/मापदंडों का ईपयोग ककया जाता है?
वादों का ननपटान करने हेतु स्थानपत ककया गया
है। 1. ऄल्प नवकास

2. आनकी सफिता मुख्य रूप से न्यायािय से बाहर 2. राष्ट्रीय सुरक्षा


पीनडत पक्षों के मध्य समझनते पर ननभभर है।
3. जनसंख्या
3. ये ननयनमत न्यायाियों के कायभभार को कम करने
नीचे कदए गए कू ट का प्रयोग कर सही ईत्तर चुननए।
का प्रयास करती हैं।
नीचे कदए गए कू ट का प्रयोग कर सही ईत्तर चुननए। (a) के वि 1
(a) के वि 1 और 2
(b) के वि 2 और 3
(b) के वि 2 और 3
(c) के वि 3
(c) के वि 1 और 3

(d) 1, 2 और 3 (d) 1, 2 और 3
31 www.visionias.in ©Vision IAS
89. Which of the following features are visible 92. With reference to State Governor, consider
in the Indian political party system? the following statements:
1. Multi Party System 1. Method of appointment of Governor is
2. Parties are organized around eminent adopted from Canadian Constitution.
leaders. 2. The same person can be appointed as
3. Emergence of regional parties. Governor for two or more states.
Select the correct answer using the code
Which of the statements given above is/are
given below.
correct?
(a) 1 and 2 only
(a) 1 only
(b) 2 and 3 only
(b) 2 only
(c) 1 and 3 only
(c) Both 1 and 2
(d) 1, 2 and 3
(d) Neither 1 nor 2

90. With reference to 'money bills' in the Indian


93. With reference to the Chief Minister,
Parliament, consider the following
statements: consider the following statements:

1. A money bill can be introduced only on 1. He holds office during the pleasure of
the recommendation of the President. the Governor and he can be dismissed by
2. The decision of the Speaker on whether the Governor at any time.
a bill is a money bill can be questioned 2. He can recommend the dissolution of the
in Parliament. Legislative Assembly to the Governor at
Which of the statements given above is/are any time.
correct? Which of the statements given above is/are
(a) 1 only correct?
(b) 2 only (a) 1 only
(c) Both 1 and 2 (b) 2 only
(d) Neither 1 nor 2
(c) Both 1 and 2
(d) Neither 1 nor 2
91. Consider the following statements:
1. The corporations or the companies
94. Which among the following is not correct in
created by the Central government
the context of Judicial Activism?
are immune from state taxation.
(a) It has forced executive accountability.
2. The property and income of a state is
(b) It has overburdened the courts.
exempted from Central taxation.
(c) It has strengthened the principle of
Which of the statements given above is/are
correct? separation of power among the three

(a) 1 only organs of state.


(b) 2 only (d) It has made the judicial system
(c) Both 1 and 2 accessible to disadvantageous groups in
(d) Neither 1 nor 2 society.

32 www.visionias.in ©Vision IAS


89. भारतीय राजनीनतक दि प्रणािी में ननम्ननिनखत में से 92. राज्य के राज्यपाि के संदभभ में ननम्ननिनखत कथनों पर

कनन-सी नवशेषताएं पररिनक्षत होती हैं? नवचार कीनजए:

1. बहु-दिीय व्यवस्था 1. राज्यपाि की ननयुनि की पद्धनत को कनाडा के

2. प्रनतनष्ठत नेताओं के नेतृत्व में दिों की स्थापना संनवधान से ग्रहण ककया गया है।

3. क्षेत्रीय दिों का ईदय 2. एक ही व्यनि को दो या ऄनधक राज्यों का

नीचे कदए गए कू ट का प्रयोग कर सही ईत्तर चुननए। राज्यपाि ननयुि ककया जा सकता है।

(a) के वि 1 और 2 ईपयुभि कथनों में से कनन-सा/से सही है/हैं?

(b) के वि 2 और 3 (a) के वि 1

(c) के वि 1 और 3 (b) के वि 2

(d) 1, 2 और 3 (c) 1 और 2 दोनों

(d) न तो 1, न ही 2
90. भारतीय संसद में धन नवधेयक के संदभभ में,

ननम्ननिनखत कथनों पर नवचार कीनजए: 93. मुख्यमंत्री के संदभभ में ननम्ननिनखत कथनों पर नवचार
1. धन नवधेयक के वि राष्ट्रपनत की ऄनुशस
ं ा पर ही
कीनजए:
प्रस्तुत ककया जा सकता है।
1. वह राज्यपाि के प्रसादपयंत पद धारण करता है
2. िोकसभा ऄध्यक्ष का यह ननणभय कक नवधेयक धन
और ईसे ककसी भी समय राज्यपाि द्वारा
नवधेयक है या नहीं, संसद में प्र‍नगत ककया जा
पदच्युत ककया जा सकता है।
सकता है।
2. वह ककसी भी समय राज्यपाि से नवधान सभा
ईपयुभि कथनों में से कनन-सा/से सही है/हैं?
को भंग करने की ऄनुशस
ं ा कर सकता है।
(a) के वि 1
ईपयुभि कथनों में से कनन-सा/से सही है/हैं?
(b) के वि 2
(a) के वि 1
(c) 1 और 2 दोनों
(b) के वि 2
(d) न तो 1, न ही 2
(c) 1 और 2 दोनों

(d) न तो 1, न ही 2
91. ननम्ननिनखत कथनों पर नवचार कीनजए:

1. कें द्र सरकार द्वारा गरठत ननगम या कं पननयों को


94. न्यानयक सकियता के संदभभ में ननम्ननिनखत में से कनन-
राज्य कराधान से ईन्मुनि प्राप्त होती है।
2. राज्य की संपनत्त और अय कें द्रीय कराधान से सा कथन सही नहीं है?
मुि होती है। (a) आसने कायभपानिका पर ईत्तरदानयत्व का दबाव
ईपयुभि कथनों में से कनन-सा/से सही है/हैं? बनाया है।
(a) के वि 1 (b) आसने न्यायाियों के कायभभार में वृनद्ध की है।

(b) के वि 2 (c) आसने राज्य के तीनों ऄंगों के मध्य शनि


पृथक्करण के नसद्धांत को सुद ृ ककया है।
(c) 1 और 2 दोनों
(d) आसने न्यानयक प्रणािी को समाज के वंनचत
(d) न तो 1, न ही 2
समूहों के निए सुिभ बनाया है।
33 www.visionias.in ©Vision IAS
95. Under which of the following circumstances 98. Supreme Court has the power to hear the
can Parliament enact laws on subjects disputes in the first instance regarding the
enumerated in the State List?
election of:
(a) When Lok Sabha passes a resolution to
1. President
that effect.
(b) When a state makes such a request to 2. Vice-President
Parliament. 3. Members of Parliament
(c) When President’s Rule is in operation in Select the correct answer using the code
a state. given below.
(d) When a proclamation of financial
(a) 1 and 3 only
emergency is in operation.
(b) 2 and 3 only

96. With respect to the Inter-state council, (c) 1 and 2 only


consider the following statements: (d) 1, 2 and 3
1. It is established once every five years by
the President to effect coordination 99. Legislative Council enjoys equal status as
among the states and between centre and that of Legislative Assembly with respect to:
states.
1. Voting on the demands for grants.
2. Its nature of duties to be performed and
2. Ratification of Constitutional
its organisation and procedure is defined
by the Parliament. Amendment Bills
3. It can enquire and give binding decisions 3. Selection and allocation of rank to
on inter-state disputes. ministers.
Which of the statements given above is/are
4. Approval of ordinances issued by the
correct?
governor.
(a) 1 and 2 only
(b) 2 and 3 only Select the correct answer using the code
(c) 1, 2 and 3 given below.
(d) None (a) 1 and 2 only
(b) 3 and 4 only
97. With reference to Public Account of India,
(c) 1 and 3 only
consider the following statements:
(d) 2 and 4 only
1. All the loans raised by the government
by the issue of treasury bills form a part
of Public account of India. 100. Which of the following refers to the
2. Payment from this account can be made termination of a session of a House of
without parliamentary appropriation. Parliament?
Which of the statements give above is/are (a) Adjournment
correct?
(b) Adjournment sine die
(a) 1 only
(c) Prorogation
(b) 2 only
(c) Both 1 and 2 (d) Dissolution
(d) Neither 1 nor 2
34 www.visionias.in ©Vision IAS
95. ननम्ननिनखत में से ककन पररनस्थनतयों में संसद, राज्य 98. ईच्चतम न्यायािय के पास ननम्ननिनखत में से ककनके

सूची में ईनल्िनखत नवषयों पर कानून बना सकती है? ननवाभचन से संबंनधत नववादों की प्राथनमक सुनवाइ की
(a) जब िोकसभा आस अशय का प्रस्ताव पाररत शनि है:
करती है।
1. राष्ट्रपनत
(b) जब कोइ राज्य, संसद से आस हेतु ऄनुरोध करता
2. ईप-राष्ट्रपनत
है।
3. संसद सदस्यों
(c) जब राज्य में राष्ट्रपनत शासन िागू हो।
नीचे कदए गए कू ट का प्रयोग कर सही ईत्तर चुननए।
(d) जब नवत्तीय अपात की ईद्घोषणा िागू हो।
(a) के वि 1 और 3
96. ऄंतराभज्यीय पररषद के संदभभ में, ननम्ननिनखत कथनों
(b) के वि 2 और 3
पर नवचार कीनजए:
(c) के वि 1 और 2
1. राज्यों के मध्य तथा कें द्र एवं राज्यों के मध्य
समन्वय स्थानपत करने हेतु प्रत्येक पांच वषों में (d) 1, 2 और 3
राष्ट्रपनत द्वारा आसकी स्थापना की जाती है।
2. आसके द्वारा ककए जाने वािे कत्तभव्यों की प्रकृ नत को
आसके संगठन और प्रकिया को संसद द्वारा 99. ननम्ननिनखत में से ककन सन्दभों में, नवधान पररषद को

ननधाभररत ककया जाता है। नवधानसभा के समान दजाभ प्राप्त है:


3. यह ऄंतराभज्यीय नववादों की जांच कर सकती है
1. ऄनुदान मांग पर मतदान।
तथा आन पर बाध्यकारी ननणभय दे सकती है।
ईपयुभि कथनों में से कनन-सा/से सही है/हैं? 2. संवैधाननक संशोधन नवधेयकों का ऄनुसमथभन

(a) के वि 1 और 2 3. मंनत्रयों का चयन और रैं क का अबंटन।

(b) के वि 2 और 3
4. राज्यपाि द्वारा जारी ऄध्यादेशों को स्वीकृ नत।
(c) 1, 2 और 3
नीचे कदए गए कू ट का प्रयोग कर सही ईत्तर चुननए।
(d) ईपयुभि में से कोइ नहीं
(a) के वि 1 और 2

97. भारत के िोक िेखा के संदभभ में ननम्ननिनखत कथनों (b) के वि 3 और 4

पर नवचार कीनजए: (c) के वि 1 और 3


1. ट्रेजरी नबि के माध्यम से सरकार द्वारा जुटाए
गए सभी ऋण भारत के िोक िेखा का भाग होते (d) के वि 2 और 4

हैं।
2. संसदीय नवननयोजन के नबना आस खाते से 100. ननम्ननिनखत में से कनन-सा संसद के ककसी सदन के
भुगतान ककया जा सकता है।
एक सत्र की समानप्त को संदर्वभत करता है?
ईपयुभि कथनों में से कनन-सा/से सही है/हैं?

(a) के वि 1 (a) स्थगन

(b) के वि 2 (b) ऄनननश्चत काि के निए स्थगन

(c) 1 और 2 दोनों (c) सत्रावसान

(d) न तो 1, न ही 2 (d) नवघटन


35 www.visionias.in ©Vision IAS
VISIONIAS
www.visionias.in

ANSWERS & EXPLANATION


GENERAL STUIDES (P) TEST – 2652 (2019)

Q 1.B
 प्रधानमंत्री संसद के ननम्न सदन का नेता होता है। आस सन्दभभ में वह ननम्ननिनखत शनियों का ईपभोग करता है:
o वह संसद के सत्रों को अहूत करने और ईनका सत्रावसान करने के संबंध में राष्ट्रपनत को परामशभ प्रदान करता है। आसनिए,
कथन 2 सही है।
o वह राष्ट्रपनत से कभी भी िोकसभा को भंग करने की ऄनुशंसा कर सकता है। िेककन के वि राष्ट्रपनत ही िोकसभा भंग कर
सकता है। आसनिए, कथन 1 सही नहीं है।
o वह सदन के पटि पर सरकार की नीनतयों की घोषणा करता है।

Q 2.B
 कथन 1 सही नहीं है: संनवधान के भाग 9 के पंचायतों से संबंनधत ईपबंध पांचवीं ऄनुसच
ू ी के क्षेत्रों पर िागू नहीं होते हैं।
हािााँकक संसद ऐसे क्षेत्रों में आन प्रावधानों का नवस्तार कर सकती है (यकद चाहे तो स्वयं ्ारा ननयत ककए जाने वािे ऄपवादों
और संशोधनों के साथ) । आस प्रावधान के ऄंतगभत, संसद ने "पंचायतों के प्रावधान (ऄनुसूनचत क्षेत्रों पर नवस्तार) ऄनधननयम",
1996 को ऄनधननयनमत ककया है। आसे िोकनप्रय रूप से PESA (पेसा) ऄनधननयम या नवस्तार ऄनधननयम के रूप में भी जाना
जाता है। वतभमान में पांचवीं ऄनुसूची के क्षेत्र दस राज्यों में नवस्तृत हैं। ये राज्य अंध्र प्रदेश, छत्तीसगढ़, गुजरात, नहमाचि प्रदेश,
झारखंड, मध्य प्रदेश, महाराष्ट्र, ओनडशा, राजस्थान और तेिंगाना हैं। पेसा छठी ऄनुसच
ू ी के क्षेत्रों में नवस्तारितरत नहीं होता है।
 कथन 2 सही है: आस ऄनधननयम के ऄनुसार प्रत्येक रामाम सभा को िोक प्रथाओं और रीनत-रितरवाजों, ईनकी सांस्कृ नतक पहचान,
सामुदानयक संसाधनों और नववाद समाधान की पारं परितरक नवनध की सुरक्षा और संरक्षण करने का ऄनधकार होगा।

Q 3.C
 कथन 1 और 2 दोनों सही हैं।
 संनवधान कदल्िी को सवोच्च न्यायािय की पीठ के रूप में घोनषत करता है।
 साथ ही यह भारत के मुख्य न्यायाधीश को सवो्च न्यायािय की पीठ के रूप में ऄन्य स्थान या स्थानों को ननयत करने के निए
ऄनधकृ त भी करता है। मुख्य न्यायाधीश आस संबंध में के वि राष्ट्रपनत के ऄनुमोदन से ही ननणभय िे सकता है।
ध्यातव्य है कक यह प्रावधान के वि वैकनल्पक है, ऄननवायभ नहीं। आसका ऄथभ है कक कोइ भी न्यायािय राष्ट्रपनत या मुख्य
न्यायाधीश को ककसी ऄन्य स्थान को सवो्च न्यायािय की पीठ के रूप में ननयत करने के सम्बन्ध में कोइ कदशा-ननदेश नहीं दे
सकता।

Q 4.A
 चुनाव अयोग सामान्यतः औपचारितरक प्रकिया अरं भ होने के कु छ सप्ताह पूवभ एक महत्वपूणभ प्रेस कांफ्रेंस अयोनजत कर चुनावों
की समय-सारणी की घोषणा करता है। आस प्रकार की घोषणा के पश्चात् ईम्मीदवारों और राजनीनतक दिों के मागभदशभन हेतु
तत्काि प्रभाव से अदशभ अचार संनहता िागू हो जाती है। वहीं चुनाव की औपचारितरक प्रकिया का प्रारम्भ मतदाताओं से सदन के
सदस्यों का ननवाभचन करने का अह्वान करने वािी ऄनधसूचना या ऄनधसूचनाओं के जारी होने के साथ होता है। ऄनधसूचनाएं
जारी ककये जाने के साथ ही, ईम्मीदवार जहां से चुनाव िडऩे की आच्छा रखते हैं ईन ननवाभचन क्षेत्रों से ऄपना नामांकन दजभ
करना अरं भ कर सकते हैं। आसनिए, नवकल्प (a) सही है।
1 www.visionias.in ©Vision IAS
Q 5.C
 संनवधान के भाग 15 में नननहत ऄनुच्छेद 324 से 329 में, देश की ननवाभचन प्रणािी के संबंध में ननम्ननिनखत प्रावधान ककये गए
हैं:
o कोइ भी व्यनि के वि धमभ, मूिवंश, जानत, लिग या आनमें से ककसी एक के अधार पर ननवाभचन नामाविी में सनम्मनित
ककये जाने के निए ऄपात्र नहीं होगा।
o संनवधान ने चुनाव में मतानधकार के मामिे में प्रत्येक नागरितरक को समानता प्रदान की है। संसद और राज्य नवधानयका,
दोनों के ्ारा ननवाभचन नामाविी तैयार करने से संबंनधत प्रावधान ककए जा सकते हैं।
o परितरसीमन अयोग ्ारा जारी अदेशों को न्यायािय में प्रनगतगत नहीं ककया जा सकता है। आसनिए, कथन (c) सही नहीं है।
o जघन्य ऄपराधों के दोषी पाए गये कै दी िोकसभा चुनावों में मतदान नहीं कर सकते हैं।

Q 6.A
 नवकल्प 1 सही है: सवो्च न्यायािय के न्यायाधीशों की ननयुनि राष्ट्रपनत ्ारा की जाती है। मुख्य न्यायाधीश की ननयुनि
राष्ट्रपनत ्ारा सवो्च न्यायािय तथा ई्च न्यायािय के ऐसे न्यायाधीशों के परामशभ पर की जाती है नजन्हें वह अवश्यक
समझता है। ऄन्य न्यायाधीशों की ननयुनि राष्ट्रपनत ्ारा मुख्य न्यायाधीश या सवो्च न्यायािय तथा ई्च न्यायािय के ऐसे
न्यायाधीशों के परामशभ पर की जाती है नजन्हें वह अवश्यक समझता है। मुख्य न्यायाधीश से परामशभ करना मुख्य न्यायाधीश
को छोड़कर ककसी ऄन्य न्यायाधीश की ननयुनि के मामिे में ऄननवायभ होता है।
 नवकल्प 2 सही है। ई्च न्यायािय के न्यायाधीशों की ननयुनि राष्ट्रपनत ्ारा की जाती है। राष्ट्रपनत ्ारा ई्च न्यायािय के मुख्य
न्यायाधीश की ननयुनि भारत के मुख्य न्यायाधीश एवं संबंनधत राज्य के राज्यपाि के परामशभ से की जाती है। ऄन्य न्यायाधीशों
की ननयुनि के निए संबनं धत ई्च न्यायािय के मुख्य न्यायाधीश से भी परामशभ ककया जाता है।
न्तीय न्यायाधीश वाद (1993) में सवो्च न्यायािय ने यह ननणभय कदया कक भारत के मुख्य न्यायाधीश की राय के ऄनुरूप ही
ई्च न्यायािय के न्यायाधीश की ननयुनि की जा सकती है। तृतीय न्यायाधीश वाद (1998) में सवो्च न्यायािय ने तय ककया
कक ई्च न्यायािय के न्यायाधीशों की ननयुनि के मामिे में भारत के मुख्य न्यायाधीश को सवो्च न्यायािय के दो वरितरष्ठतम
न्यायाधीशों के कॉिेनजयम से परामशभ करना चानहए। आस प्रकार, के वि भारत के मुख्य न्यायाधीश की राय से “परामशभ प्रकिया”
पूरी नहीं होगी।
 नवकल्प 3 सही नहीं है। ककसी राज्य की न्यानयक सेवा के कार्ममकों (नजिा न्यायाधीशों के ऄनतरितरक्त) की ननयुनि राज्य िोक
सेवा अयोग और ईच्च न्यायािय के साथ परामशभ के पश्चात् राज्य के राज्यपाि ्ारा की जाती है।

Q 7.B
 कथन 1 सही नहीं है: सचेतक (न्हप) के पद का ईल्िेख न तो भारत के संनवधान में, न सदन के ननयमों में और न ही ककसी
संसदीय संनवनध में है। यह संसदीय सरकार की परम्पराओं पर अधारितरत है। प्रत्येक राजनीनतक दि का (चाहे सत्ताधारी हो या
नवपक्ष) संसद में एक सचेतक होता है। ईसे राजनीनतक दि ्ारा सदन के सहायक नेता के रूप में कायभ करने के निए ननयुि
ककया जाता है।
 कथन 2 सही है: ईसे ऄपनी पाटी के सदस्यों की बड़ी संख्या में ईपनस्थनत सुनननश्चत करने एवं ककसी नवशेष मुद्दे के पक्ष या
नवपक्ष में ईनके समथभन को सुनननित करने का ईत्तरदानयत्व प्रदान ककया जाता है। वह संसद में ईनके व्यवहार का नवननयमन
और ननगरानी करता है। सदस्यों से 'सचेतक’ ्ारा कदए गए ननदेशों का पािन करने की ऄपेक्षा की जाती है; ऄन्यथा
ऄनुशासनात्मक कारभ वाइ की जा सकती है।

Q 8.A
 कथन 1 सही है: मंनत्रमंडिीय (कै नबनेट) सनमनतयों का ईद्भव संनवधानेतर रूप से हुअ ऄथाभत् संनवधान में आनका ईल्िेख नहीं
ककया गया है। हािााँकक, कायभ संचािन के ननयम ईनकी स्थापना का प्रावधान करते हैं।
 कथन 2 सही नहीं है: ईनकी स्थापना राष्ट्रपनत के ्ारा नहीं, ऄनपतु प्रधानमंत्री के ्ारा समय की ऄननवायभताओं एवं नस्थनत की
अवश्यकताओं के ऄनुसार की जाती है।
 कथन 3 सही नहीं है: मंनत्रमंडिीय सनमनतयों में अमतौर पर के वि कै नबनेट मंत्री सनम्मनित होते हैं। हािााँकक, गैर-कै नबनेट
मंनत्रयों को ईनकी सदस्यता से वंनचत नहीं ककया जाता है।

2 www.visionias.in ©Vision IAS


Q 9.A
 संनवधान ्ारा संयुि बैठक का प्रावधान, सदन के दोनों सदनों के समक्ष नवधेयक पारितरत करने के नवषय में ईत्पन्न गनतरोध को
समाप्त करने के निए ककया गया है। ककसी नवधेयक को एक सदन ्ारा पारितरत ककये जाने और दूसरे सदन को ऄरामेनषत ककये जाने
के पश्चात्, ननम्ननिनखत तीन दशाओं में गनतरोध ईत्पन्न हुअ माना जाता है:
o दूसरे सदन ्ारा नवधेयक ऄस्वीकार कर कदया गया हो, या
o नवधेयक में ककये जाने वािे संशोधनों के बारे में दोनों सदन ऄंनतम रूप से ऄसहमत हो गए हों; या
o दूसरे सदन को नवधेयक प्राप्त होने की तारीख से 6 माह से ऄनधक समय बीत गया हो और ईसके ्ारा नवधेयक पारितरत न
ककया गया हो।
 ईपयुभि तीन दशाओं में राष्ट्रपनत नवधेयक पर नवचार-नवमशभ और मतदान के निए दोनों सदनों की संयुि बैठक अहूत कर सकता
है। आस बैठक की ऄध्यक्षता िोकसभा ऄध्यक्ष ्ारा की जाती है।
 संयुि बैठक का यह प्रावधान के वि सामान्य नवधेयकों और नवत्त नवधेयकों के निए िागू होता है; धन नवधेयकों या संनवधान
संशोधन नवधेयकों के निए नहीं। धन नवधेयक के मामिे में िोकसभा को ऄनधभावी शनियां प्राप्त हैं, जबकक संनवधान संशोधन
नवधेयक दोनों सदनों ्ारा ऄननवायभत: पृथक रूप से पारितरत ककया जाना चानहए।

Q 10.A
 मूि संनवधान में ऄनधकरणों के संबंध में ईपबंध नहीं ककया गया था। 42वें संनवधान संशोधन ऄनधननयम,1976 के ्ारा
संनवधान में एक नया भाग XIV-A जोड़ा गया नजसके माध्यम से ऄनधकरणों को संवध
ै ाननक प्रनस्थनत प्रदान की गइ। आसनिए
कथन 1 सही है।
 आस भाग का शीषभक ‘ऄनधकरण’ है और आसमें दो ऄनुच्छेद सनम्मनित हैं- ऄनुच्छेद 323 A, जोकक प्रशासननक ऄनधकरणों से
संबंनधत है और ऄनुच्छेद 323 B, जोकक ऄन्य नवषयों के निए ऄनधकरणों से संबंनधत है। ऄनुच्छेद 323 A के ऄनुपािन में संसद
ने 1985 में प्रशासननक ऄनधकरण ऄनधननयम पारितरत ककया। यह ऄनधननयम कें द्र सरकार को एक कें द्रीय प्रशासननक ऄनधकरण
एवं राज्य प्रशासननक ऄनधकरणों की स्थापना करने के निए ऄनधकृ त करता है। आसनिए कथन 2 सही है।
 ऄनुच्छेद 323-A कें द्र सरकार और राज्य सरकार के ऄधीन सरकारी कमभचारितरयों की भती और सेवा की शतों कें संबध
ं में
ऄनधननणभय या नववादों के परीक्षण और नशकायतों के निए प्रशासननक ऄनधकरणों की स्थापना संसदीय नवनध ्ारा ककये जाने
का प्रावधान करता है। आसनिए, कथन 3 सही नहीं है।

Q 11.B
 भारत में कइ कारणों जैसे कक गणपूर्मत के ऄभाव, नवपक्ष के सदस्यों ्ारा सत्रों के बनहष्कार (जो सदन को चचाभ के माध्यम से
कायभपानिका को ननयंनत्रत करने के ऄनधकार से वंनचत कर देता है) अकद के कारण कायभपानिका पर संसदीय ननयंत्रण में कमी अ
जाती है।
 नगिोरितटन कायभ समापन का प्रस्ताव है। आसके तहत समयाभाव के कारण (क्योंकक चचाभ के निए अवंरितटत समय समाप्त हो चुका
होता है) ककसी नवधेयक या प्रस्ताव के नजन खण्डों पर चचाभ की जा चुकी है, के साथ ही ईन खण्डों पर भी मतदान करवाया
जाता है नजन पर चचाभ नहीं की गइ है। 'नगिोरितटन' का ऄनधकानधक सहारा निए जाने के कारण नवत्तीय ननयंत्रण की गुंजाआश
कम हो गइ है।
 प्रत्यायोनजत नवधायन: ये वे कानून हैं नजनका ननमाभण कायभकारी प्रानधकारितरयों ्ारा ईन्हें प्राथनमक नवधान ्ारा प्रदत्त शनियों
के ऄधीन ककया जाता है। कायभकारी प्रानधकारितरयों को ये शनियााँ प्राथनमक नवधान ्ारा ईसकी अवश्यकताओं को कायाभनन्वत
एवं प्रशानसत करने के निए दी जाती हैं। 'प्रत्यायोनजत नवधायन' में वृनि ने कानूनों के नवस्तृत ननमाभण में संसद की भूनमका को
कम ककया है और नौकरशाही की शनियों में वृनि हुइ है। आसके परितरणामस्वरूप कायभपानिका पर संसदीय ननयंत्रण में कमी अती
है।
 भारत के संनवधान ने सरकार के संसदीय स्वरूप का चयन ककया नजसमें कायभपानिका ऄपनी नीनतयों और कारभ वाआयों के निए
संसद के प्रनत ईत्तरदायी होती है। आसनिए संसद प्रनगतकाि, शून्यकाि, ध्यानाकषभण प्रस्ताव एवं स्थगन प्रस्ताव अकद के माध्यम
से कायभपानिका पर ननयंत्रण स्थानपत करती है। आसनिए, नवकल्प 3 सही नहीं है।

3 www.visionias.in ©Vision IAS


Q 12.D
 ऄनुच्छेद 301 में यह ईल्िेख है कक भारत के राज्यक्षेत्र में सवभत्र व्यापार, वानणज्य और समागम ऄबाध होगा। आस
प्रकार, ऄनुच्छेद 301 ्ारा गारं टीकृ त यह स्वतंत्रता सभी ननबंधनों से मुि है, नसवाये ईन ननबंधनों के जो स्वयं संनवधान के

भाग XIII के ऄन्य प्रावधानों (ऄनुच्छेद 302 से 305) में ईनल्िनखत हैं। आसनिए, कथन 1 सही नहीं है।
 संसद िोकनहत में राज्यों के मध्य या राज्य की सीमा के भीतर व्यापार, वानणज्य और समागम की स्वतंत्रता पर ननबंधन
ऄनधरोनपत कर सकती है। िेककन संसद एक राज्य को दूसरे राज्य की तुिना में वरीयता प्रदान नहीं कर सकती या राज्यों के
मध्य भेदभाव नहीं कर सकती (के वि ईन मामिों को छोड़कर जबकक भारत के ककसी भाग में वस्तुओं की कमी ईत्पन्न हो गयी
हो)। आसनिए, कथन 2 सही नहीं है।

Q 13.C
 नवकल्प 1 सही नहीं है: यकद पदासीन प्रधानमंत्री की पाटी को िोकसभा में बहुमत प्राप्त रहता है, तो िोकसभा भंग नहीं होगी।
 नवकल्प 2 सही नहीं है: िोकसभा को नये प्रधानमंत्री का चुनाव करने की ऐसी कोइ शनि प्राप्त नहीं है।
 नवकल्प 3 सही है: प्रधानमंत्री मंनत्रपरितरषद का प्रमुख होता है। यकद प्रधानमंत्री त्यागपत्र दे देता है या ईसकी मृत्यु हो जाती है तो
ऄन्य मंत्री कायभ नहीं कर सकते। दूसरे शब्दों में पदासीन प्रधानमंत्री का त्यागपत्र या मृत्यु, मंनत्रपरितरषद को स्वत: भंग कर देते हैं।
 नवकल्प 4 सही नहीं है: पदासीन प्रधानमंत्री के ननधन पर, यकद सत्ताधारी दि नये नेता का चुनाव करता है (अवश्यक नहीं कक
वह वरितरष्ठतम नेता हो), तो राष्ट्रपनत ईसे नये प्रधानमंत्री के रुप में ननयुि करे गा; ऄन्यथा राष्ट्रपनत ऄपने नववेकानधकार का
प्रयोग कर सकता है।

Q 14.B
 राष्ट्रपनत िोकसभा के एक सदस्य को प्रोटेम स्पीकर के रूप में ननयुनि करता है। सामान्य रूप से आस पद के निए वरितरष्ठतम
सदस्य का चयन ककया जाता है। राष्ट्रपनत स्वयं ही प्रोटेम स्पीकर को शपथ कदिाता है।
 आसनिए, कथन 1 और 2 सही हैं।
 यह एक ऄस्थायी पद होता है, नजसका कायभकाि के वि कु छ ही कदनों का होता है। िेककन प्रोटेम स्पीकर को िोकसभा ऄध्यक्ष
की सभी शनियााँ प्राप्त होती हैं। आसनिए, कथन 3 सही नहीं है।

Q 15.D
 कथन 1 सही नहीं है: राज्य नवधानमंडिों के संगठन में कोइ एकरूपता नहीं होती है। ऄनधकांश राज्यों में एकसदनीय व्यवस्था
है, जबकक कु छ में न्सदनात्मक व्यवस्था नवद्यमान है।
 कथन 2 सही नहीं है: नवधानसभा में िोगों ्ारा प्रत्यक्ष रुप से सावभभौनमक वयस्क मतानधकार के अधार पर ननवाभनचत
प्रनतनननध होते हैं। आसकी ऄनधकतम सदस्य संख्या 500 और न्यूनतम सदस्य संख्या 60 ननयत की गइ है। आसका तात्पयभ यह है
कक राज्य की जनसंख्या के अधार पर आसकी सदस्य संख्या 60 से 500 के मध्य होती है।
हािााँकक ऄरुणाचि प्रदेश, नसकिम और गोवा के मामिे में न्यूनतम सदस्य संख्या 30 तथा नमजोरम और नागािैंड के मामिे में
िमशः 40 और 46 ननयत की गइ है।
 कथन 3 सही नहीं है: यद्यनप नवधायकों का ननवाभचन प्रत्यक्ष रुप से ककया जाता है, ककन्तु नसकिम और नागािैंड में नवधानसभा
के कु छ सदस्यों का ननवाभचन ऄप्रत्यक्ष रुप से भी ककया जाता है। ननवाभनचत प्रनतनननधयों के ऄनतरितरि, राज्यपाि अंग्ि भारतीय
समुदाय से एक सदस्य को (यकद ईसकी राय में आस समुदाय को पयाभप्त प्रनतनननधत्व प्राप्त नहीं हुअ है तो) नवधानसभा के निए
मनोनीत कर सकता है। 95वें संनवधान संशोधन ऄनधननयम, 2009 के तहत यह व्यवस्था 2020 तक जारी रहेगी।

4 www.visionias.in ©Vision IAS


 ऄतः तीनों कथन सही नहीं हैं और नवकल्प (d) सही ईत्तर है।
Q 16.C
 स्थानीय शासन ‘राज्य सूची का नवषय’ है। राज्य आस नवषय पर स्वयं के कानून ननमाभण के निए स्वतंत्र हैं। परन्तु 73वें और 74वें
संनवधान संशोधन ऄनधननयम के माध्यम से संनवधान में संशोधन के पश्चात्, राज्यों को स्थानीय ननकायों के संबंध में ऄपने
कानूनों को संशोनधत संनवधान के ऄनुरूप बनाने के निए ईनमें परितरवतभन करना पड़ा था। राज्यों को आन संशोधनों को दृनिगत
रखते हुए ऄपने संबनं धत राज्य कानूनों में अवश्यक परितरवतभन करने के निए 1 वषभ का समय कदया गया था।

Q 17.B
 कथन 1 सही नहीं है। नजिा न्यायाधीश नजिे में सवो्च न्यानयक ऄनधकारी होता है। ईसे दीवानी और फौजदारी, दोनों ही
प्रकार के मामिों में मूि एवं ऄपीिीय क्षेत्रानधकार प्राप्त होता है। नजिा न्यायाधीश को न्यानयक एवं प्रशासननक, दोनों प्रकार
की शनियां प्राप्त होती हैं। ईसे नजिे में सभी ऄधीनस्थ न्यायाियों पर पयभवक्ष
े ण की शनि भी प्राप्त होती है। ईसके अदेशों और
ननणभय के नवरुि ई्च न्यायािय में ऄपीि की जा सकती है।
 कथन 2 सही है। सत्र न्यायाधीश को ककसी भी दोषी को ईम्रकै द व मृत्यु दंड देने की ऄनधकारितरता प्राप्त है, ककतु मृत्यु दंड देने के
मामिे में ई्च न्यायािय का ऄनुमोदन अवश्यक है, चाहे कोइ ऄपीि की जाए या न की जाए।

Q 18.D
 बजट, एक नवत्तीय वषभ के दौरान भारत सरकार की ऄनुमाननत प्रानप्तयों और व्ययों का नववरण होता है। भारत में नवत्तीय वषभ 1
ऄप्रैि से प्रारं भ होकर 31 माचभ को समाप्त होता है। संनवधान में बजट को ‘वार्मषक नवत्तीय नववरण’ के रूप में संदर्मभत ककया
गया है। ऄथाभत संनवधान में ‘बजट’ शब्द का कहीं ईल्िेख नहीं है। वास्तव में बजट, वार्मषक नवत्तीय नववरण का प्रचनित नाम है।
आसका ईल्िेख संनवधान के ऄनुच्छेद 112 में ककया गया है।
 संनवधान में बजट के कियान्वयन से संबंनधत कु छ ऄन्य प्रावधान ननम्ननिनखत हैं:
o राष्ट्रपनत, प्रत्येक नवत्तीय वषभ के संबंध में, संसद के दोनों सदनों के समक्ष ईस वषभ हेतु भारत सरकार की ऄनुमाननत प्रानप्तयों
और व्ययों के नववरण को प्रस्तुत कराएगा।
o राष्ट्रपनत की नसफारितरश के नबना ककसी ऄनुदान की मांग नहीं की जाएगी।
o कानून के ऄंतगभत ककए गए नवननयोग को छोड़कर, भारत की संनचत नननध से ककसी प्रकार की धनरानश का अहरण नहीं
ककया जाएगा।
o राष्ट्रपनत की ऄनुशंसा के नबना, कर अरोनपत करने वािा कोइ धन नवधेयक संसद में पुर:स्थानपत नहीं होगा। आस तरह के
ककसी नवधेयक को राज्यसभा में पुर:स्थानपत नहीं ककया जाएगा।
o कानून के प्रानधकार के ऄनतरितरि, ककसी ऄन्य प्रकार से ककसी कर की ईगाही या संरामहण नहीं ककया जाएगा।
o संनवधान में बजट के कियान्वयन के संबंध में संसद के दोनों सदनों की सापेक्ष भूनमकाओं या नस्थनत को भी ननम्ननिनखत
प्रकार से परितरभानषत ककया है:
 कराधान से संबनं धत धन नवधेयक या नवत्त नवधेयक को राज्यसभा में पुर:स्थानपत नहीं ककया जा सकता— आसे
ऄननवायभ रूप से के वि िोकसभा में पुर:स्थानपत ककया जाना चानहए।
 राज्यसभा को ऄनुदान की मांग पर मतदान की कोइ शनि प्राप्त नहीं है; यह िोकसभा का ऄनन्य नवशेषानधकार है।
 राज्यसभा को 14 कदन के भीतर धन नवधेयक (या नवत्त नवधेयक) िोकसभा को वापस िौटा देना चानहए। िोकसभा
आस नवषय में राज्यसभा ्ारा की गइ ऄनुशस
ं ाओं को स्वीकार या ऄस्वीकार कर सकती है।
o बजट में सनन्ननहत व्यय ऄनुमानों में, भारत की संनचत नननध पर भारितरत व्यय एवं भारत की संनचत नननध से ककए गए व्यय,
दोनों को पृथक-पृथक रूप से प्रदर्मशत ककया जाना चानहए।

Q 19.D
 भारत के राष्ट्रपनत के पास तीन प्रकार की वीटो शनियााँ हैं- पॉके ट वीटो, ननिंबनकारी वीटो और अत्यांनतक वीटो। ऄमेरितरकी
राष्ट्रपनत को पॉके ट वीटो का ऄनधकार नहीं होता है, ऄथाभत वह ककसी नवधेयक को सहज रूप से ऄनननश्चत ऄवनध तक िंनबत

5 www.visionias.in ©Vision IAS


नहीं रख सकता, जबकक भारतीय राष्ट्रपनत ऐसा कर सकता है। आसनिए ऐसा कहा जाता है कक भारतीय राष्ट्रपनत की जेब बड़ी
है।
Q 20.C
 ननवाभचन और ऄन्य संबंनधत कानून (संशोधन) ऄनधननयम, 2003 के ऄंतगभत चुनाव अयोग को चुनावों के दौरान ककसी भी
मामिे को प्रदर्मशत या प्रचारितरत करने या जनता को संबोनधत करने के निए निए के बि टेिीनवजन नेटवकभ और ऄन्य
आिेक्रॉननक मीनडया पर राजनीनतक दिों के निए समय का न्यायसंगत अवंटन करना चानहए। आस अवंटन का ननधाभरण
मान्यता प्राप्त राजनीनतक दि के नपछिे प्रदशभन के अधार पर ककया जाना चानहए। आसनिए, कथन 1 सही है।
 राजनीनतक दिों को सरकारी कं पनी के ऄनतरितरि ककसी भी व्यनि ऄथवा कं पनी से ककतनी भी धनरानश प्राप्त करने का ऄनधकार
होगा। हािााँकक ईन्हें अयकर राहत के ककसी दावे के निए 2000 रु. से ऄनधक के ककसी भी ऄंशदान का नववरण ननवाभचन
अयोग को देना होगा। आसके ऄनतरितरि कं पननयों को ऄंशदान की मात्रा के अधार पर अयकर में छू ट प्राप्त होगी। आसनिए, कथन
2 सही नहीं है।
 वषभ 2003 में चुनाव अयोग ने एक अदेश जारी कर संसद या राज्य नवधानमंडि में ननवाभचन की आच्छा रखने वािे प्रत्येक
ईम्मीदवार को ऄपने नामांकन पत्र में यह सूचना प्रदान करने का ननदेश कदया कक क्या ईसे भूतकाि में ककसी दंडनीय ऄपराध
का दोषी ठहराया गया या ईससे नवमुि या बरी ककया गया है। आसनिए, कथन 3 सही है।

Q 21.C
 छावनी परितरषद् की स्थापना छावनी क्षेत्र में ऄसैन्य जनसंख्या हेतु नगरपानिका प्रशासन के निए की जाती है। आसका गठन
2006 के छावनी ऄनधननयम के ईपबंधों के ऄंतगभत ककया गया है। आस ऄनधननयम को कें द्र सरकार ्ारा ऄनधननयनमत ककया
गया है।
 यह कें द्र सरकार के रक्षा मंत्रािय के प्रशासननक ननयंत्रण के ऄंतगभत कायभ करता है। आस प्रकार, छावनी परितरषद् का गठन और
प्रशासन कें द्र सरकार ्ारा ककया जाता है।
 छावनी परितरषद में अंनशक रूप से ननवाभनचत एवं अंनशक रूप से मनोनीत सदस्य शानमि होते हैं। ननवाभनचत सदस्य 5 वषभ की
ऄवनध के निए पद धारण करते हैं, जबकक मनोनीत सदस्य (ऄथाभत पदेन सदस्य) तब तक के निए पदधारण करते हैं जब तक वे
ईस स्टेशन पर तैनात रहते हैं। स्टेशन की कमान संभािने वािा सैन्य ऄनधकारी आस परितरषद् का पदेन ऄध्यक्ष होता है और
आसकी बैठकों की ऄध्यक्षता करता है। परितरषद् के ईपाध्यक्ष का चुनाव ईन्हीं सदस्यों में से ननवाभनचत सदस्यों ्ारा पााँच वषभ की
ऄवनध के निए ककया जाता है।

Q 22.C
 संनवधान (ऄनुच्छेद 143) राष्ट्रपनत को ननम्ननिनखत दो श्रेणी के नवषयों पर ई्च तम न्यायािय से राय करने के निए प्रानधकृ त
करता है:
o यकद ककसी समय राष्ट्रपनत को प्रतीत होता है कक नवनध या तथ्य का कोइ ऐसा प्रनगत ईत्पन्न हुअ है या ईत्पन्न होने की
संभावना है, जो व्यापक महत्त्व का है। आस नवषय में ई्च तम न्यायािय राष्ट्रपनत को ऄपनी राय प्रदान कर सकता है या
प्रदान करने से मना कर सकता है।
o ककसी पूवभ संवैधाननक संनध, समझौते, प्रसंनवदा, ऄनुबंध, सनद या ऄन्य समान साधनों से ईत्पन्न होने वािे ककसी भी
नववाद पर। आस नवषय में, ई्च तम न्यायािय के निए राष्रपनत को ऄपनी राय देना ऄननवायभ है।
 हािांकक, दोनों नवषयों में ई्च तम न्यायािय ्ारा ्यक्त की गयी राय न्यानयक ननणभय के बजाय के वि सिाहकारी होती है।
आसनिए, यह राष्रपनत पर बाध्यकारी नहीं होती है। वह राय का ऄनुपािन कर भी सकता है और नहीं भी। हािांकक, यह
सरकार को ककसी नवषय पर ननणभय निए जाने के संबंध में एक अनधकारितरक नवनधक सिाह प्राप्त करने की सुनवधा प्रदान करता
है।

Q 23.B
 िोकसभा ऄध्यक्ष, िोकसभा की सभी संसदीय सनमनतयों के ऄध्यक्षों की ननयुनि और ईनकी कायभप्रणािी की ननगरानी करता
है। वह स्वयं कायभ मंत्रणा सनमनत, ननयम सनमनत और सामान्य प्रयोजन सनमनत का ऄध्यक्ष होता है। आसनिए, ईत्तर (b) है।

6 www.visionias.in ©Vision IAS


 िोक िेखा सनमनत: आस सनमनत का कायभ संसद के समक्ष प्रस्तुत भारत के ननयंत्रक-महािेखापरीक्षक (CAG) की वार्मषक िेखा
परीक्षा रितरपोटभ की जांच करना है। CAG राष्ट्रपनत को तीन रितरपोटभ प्रस्तुत करता है। ये हैं: नवननयोग िेखा पर िेखा परीक्षा
रितरपोटभ, नवत्त खातों पर िेखा परीक्षा रितरपोटभ और सावभजननक ईपिमों पर िेखा परीक्षा रितरपोटभ।
वतभमान में आस सनमनत में 22 सदस्य (िोकसभा से 15 और राज्य सभा से 7) शानमि होते हैं। संसद ्ारा प्रत्येक वषभ ऄपने
सदस्यों में से एकि संिमणीय मत के माध्यम से अनुपानतक प्रनतनननधत्व प्रणािी के अधार पर सदस्यों का चुनाव ककया जाता
है। परम्परागत रूप से िोकसभा ऄध्यक्ष ्ारा नवपक्ष के ककसी सदस्य को आस सनमनत का ऄध्यक्ष ननयुि ककया जाता है।
 कायभ मंत्रणा सनमनत: यह सदन के कायभिम और समय सारितरणी को ननयंनत्रत करती है। यह सरकार ्ारा सदन के समक्ष िाये गये
नवधायी और ऄन्य कायों के ननष्पादन के निए समय अवंरितटत करती है। िोक सभा की कायभ मंत्रणा सनमनत में 15 सदस्य
(ऄध्यक्ष सनहत) होते हैं तथा िोकसभा ऄध्यक्ष आसका पदेन ऄध्यक्ष होता है। राज्य सभा की कायभ मंत्रणा सनमनत में कु ि 11
सदस्य (ऄध्यक्ष सनहत) होते हैं तथा आसका पदेन ऄध्यक्ष, सभापनत होता है।
 अचार सनमनत: आसका गठन राज्यसभा में 1997 में और िोक सभा में 2000 में ककया गया था। यह संसद सदस्यों के निए
अचार संनहता िागू करती है। यह दुव्यव
भ हार के मामिों की जांच करती है और ईनचत कायभवाही की संस्तुनत करती है। आस
प्रकार यह संसद में ऄनुशासन और नशिाचार को बनाए रखती है।
 ननयम सनमनत: यह सदन में प्रकिया और कायभ संचािन संबंधी मामिों पर नवचार करती है और सदन के ननयमों में अवश्यक
संशोधन या ईसमें कु छ जोड़ने संबंधी संस्तुनत करती है। िोकसभा की ननयम सनमनत में 15 सदस्य (ऄध्यक्ष सनहत) होते हैं तथा
िोकसभा ऄध्यक्ष आसका पदेन ऄध्यक्ष होता है। राज्य सभा की ननयम सनमनत में कु ि 16 सदस्य होते हैं और राज्यसभा का
सभापनत आसका पदेन ऄध्यक्ष होता है।
 सामान्य प्रयोजन सनमनत, सदन के कायों से सम्बनन्धत ऐसे मामिों पर नवचार करती है और परामशभ देती है जो कक ककसी ऄन्य
संसदीय सनमनत के ऄनधकार क्षेत्र में नहीं अते हैं। प्रत्येक सदन की सनमनत में ऄध्यक्ष के रूप में पीठासीन ऄनधकारी
(ऄध्यक्ष/सभापनत) आसके पदेन ऄध्यक्ष होते हैं। आसके सदस्यों में ईपाध्यक्ष (राज्यसभा के मामिे में ईप सभापनत), सभी स्थायी
सनमनतयों के ऄध्यक्ष, सदन में मान्यता प्राप्त दिों और समूहों के नेता और कु छ ऄन्य सदस्य (नजन्हें ऄध्यक्ष ्ारा नानमत ककया
गया हो) शानमि होते हैं।

Q 24.A
 संनवधान में प्रावधान ककया गया है कक यकद कोइ सदस्य दसवीं ऄनूसूची के प्रावधानों के ऄंतगभत दि-बदि के अधार पर दोषी
पाया जाता है तो ईसे संसद की सदस्यता से वंनचत कर कदया जाएगा। दि बदि कानून के ऄंतगभत एक सदस्य ननम्ननिनखत
नस्थनतयों में ऄयोग्य हो जाता है:
o यकद वह स्वेच्छा से ईस राजनीनतक दि की सदस्यता त्याग देता है, नजसका वह चुनाव से पहिे सदस्य था।
o यकद वह सदन में ऄपने राजनीनतक दि ्ारा कदए गये ननदेश के नवरुि मतदान करता है या मतदान में सनम्मनित नहीं
होता है।
o यकद कोइ ननवाभनचत ननदभिीय सदस्य ककसी भी राजनीनतक दि में सनम्मनित हो जाता है।
o यकद कोइ मनोनीत सदस्य छह महीने की समानप्त के बाद ककसी भी राजनीनतक दि में सनम्मनित हो जाता है।
 राजनीनतक दि से ननष्कासन की नस्थनत में, ककसी सदस्य को 10 वीं ऄनुसच
ू ी के तहत नवधानसभा या संसद की सदस्यता से
स्वतः ऄयोग्य घोनषत नहीं ककया जा सकता। वह सदस्य सदन के ऄध्यक्ष के ननदेशानुसार एक ऄसंबि सदस्य (unattached
member) के रूप में बना रह सकता है। आसनिए, कथन 3 सही नहीं है। आसनिए सही ईत्तर (a) है।

Q 25.A
 कथन 1 सही है: ऄनुच्छेद 74 के तहत राष्ट्रपनत के कायों के ननवभहन में सहायता और परामशभ देने हेतु प्रधानमंत्री की ऄध्यक्षता
में एक मंनत्रपरितरषद का प्रावधान ककया गया है। संनवधान के 42वें और 44वें संशोधन ऄनधननयमों ्ारा आस परामशभ को राष्ट्रपनत
के निए बाध्यकारी बना कदया गया है।
 कथन 2 सही नहीं है: निटेन के नवपरीत, भारत के संनवधान में मंनत्रयों के नवनधक ईत्तरदानयत्व की प्रणािी का प्रावधान नहीं
ककया गया है। ककसी सावभजननक कायभ के निए राष्ट्रपनत के अदेश को मंत्री ्ारा प्रनतहस्ताक्षरितरत ककया जाना अवश्यक नहीं है।
7 www.visionias.in ©Vision IAS
Q 26.D
 प्रत्येक अम चुनाव के पश्चात प्रथम सत्र और प्रत्येक वषभ के प्रथम सत्र को राष्ट्रपनत ्ारा सम्बोनधत ककया जाता है। आस सम्बोधन
में राष्ट्रपनत नपछिे वषभ की तथा अने वािे वषभ में सरकार की नीनतयों और कायभिमों की रूपरे खा प्रस्तुत करते हैं। सम्बोधन की
यह परम्परा ‘निटेन के सम्राट या सम्राज्ञी के भाषण’ के ऄनुरूप है। राष्ट्रपनत के सम्बोधन पर संसद के दोनों सदनों में ‘धन्यवाद
प्रस्ताव’ के माध्यम से चचाभ की जाती है। चचाभ के ऄंत में प्रस्ताव पर मतदान ककया जाता है। आस प्रस्ताव को िोकसभा ्ारा
पारितरत ककया जाना अवश्यक है, ऄन्यथा आसे सदन में सरकार की पराजय माना जाता है। राष्ट्रपनत का यह ईद्घाटन भाषण संसद
सदस्यों को सरकार और प्रशासन की त्रुरितटयों और ऄसफिताओं की जााँच एवं अिोचना के निए चचाभ और बहस का ऄवसर
ईपिब्ध कराता है।

Q 27.A
 संवैधाननक रूप से राष्ट्रपनत को कोइ नववेकाधीन शनि प्रदान नहीं की गयी है, हािााँकक यह कु छ परितरनस्थनतजन्य नववेकाधीन
शनियों का प्रयोग कर सकता है। ऄतः राष्ट्रपनत ननम्ननिनखत नस्थनतयों में ऄपने नववेक (ऄथाभत नबना मंनत्रयों के परामशभ के ) से
कायभ कर सकता है:
o िोकसभा में ककसी भी दि को स्पि बहुमत प्राप्त न होने की नस्थनत में ऄथवा पद पर रहते हुए प्रधानमंत्री की ऄचानक मृत्यु
होने और कोइ स्पि ईत्तरानधकारी नहीं होने की नस्थनत में, प्रधानमंत्री की ननयुनि।
o िोकसभा में नविास मत नसि न कर पाने की नस्थनत में मंनत्रपरितरषद की बखाभस्तगी। आसनिए, कथन 3 सही है।
o मंनत्रपरितरषद ्ारा बहुमत खो देने पर िोकसभा का नवघटन।
o यकद राष्ट्रपनत को यह ज्ञात होता है कक मंनत्रपरितरषद ्ारा कदए गए परामशभ में कु छ त्रुरितट है या कानूनी कमी है या यह देश के
सवोत्तम नहत में नहीं है, तो राष्ट्रपनत मंनत्रपरितरषद को आस पर पुनः नवचार करने को कह सकता है। आसनिए, कथन 1 सही
है।
o राष्ट्रपनत संसद ्ारा पारितरत नवधेयक (धन नवधेयक के ऄनतरितरि) पर सहमनत को रोक सकता है या देने से आंकार कर सकता
है। आसनिए, कथन 2 सही नहीं है।

Q 28.D
 सदन में चचाभ हेतु स्वीकृ नत योग्य होने के निए, ककसी कटौती प्रस्ताव को ननम्ननिनखत शतों को पूरा करना अवश्यक है:
o आसे के वि एक ही मांग से सम्बनन्धत होना चानहए।
o आसे स्पि रूप से व्यि ककया जाना चानहए और आसमें नववादात्मक या ऄपमानजनक कथन सनम्मनित नहीं होने चानहए।
o यह एक नवनशि मामिे तक ही सीनमत होना चानहए।
o आसे वतभमान कानूनों में संशोधन या ननरसन के सुझाव देने वािा नहीं होना चानहए।
o आसमें ककसी ऐसे मामिे का संदभभ नहीं होना चानहये जो प्राथनमक रूप से के न्द्र सरकार से सम्बनन्धत नहीं है।
o यह भारत की समेककत नननध पर अरोनपत व्यय से सम्बनन्धत नहीं होना चानहए।
o यह ककसी ऐसे मामिे से सम्बनन्धत नहीं होना चानहए जो न्यायािय में नवचाराधीन हो।
o आसमें नवशेषानधकार पर प्रनगत नहीं ईठाया जाना चानहए।
o यह पुनः ककसी ऐसे मुद्दे को न ईठा रहा हो नजस पर ईसी सत्र में ननणभय निया जा चुका है।
o यह ककसी ऄल्प महत्व के नवषय से सम्बनधत नहीं होना चानहए।

Q 29.D
 गणपूर्मत, सदन में ईपनस्थत सदस्यों की वह न्यूनतम संख्या होती है जो सदन की कायभवाही प्रारं भ करने के निए अवश्यक है।
प्रत्येक सदन के निए यह सदस्यों की कु ि संख्या का दसवां भाग होती है नजसमें पीठासीन ऄनधकारी भी सनम्मनित होते हैं।
आसका ऄथभ है कक सदन की कायभवाही प्रारं भ करने हेतु िोकसभा में कम से कम 55 सदस्य और राज्यसभा में 25 सदस्य ईपनस्थत
होने ऄननवायभ हैं। यकद सदन की बैठक के दौरान गणपूर्मत नहीं है तो पीठासीन ऄनधकारी का यह कतभव्य है कक गणपूर्मत होने तक
वह या तो सदन को स्थनगत कर दे या बैठक की कायभवाही को रोक दे।

8 www.visionias.in ©Vision IAS


Q 30.A
 कथन 1 सही है। राज्य सभा के ईप-सभापनत का चयन सदन के सदस्यों में से ककया जाता है। राज्य सभा का सदस्य होने के निए
ककसी व्यनि की न्यूनतम अयु 30 वषभ होना ऄननवायभ है।
 कथन 2 सही नहीं है। ईप-सभापनत का पद रितरि होने की नस्थनत में रितरनि को भरने के निए ककसी ऄन्य सदस्य का चुनाव राज्य
सभा करती है।
 कथन 3 सही नहीं है। ईप सभापनत, सभापनत के ऄधीन नहीं है। हािांकक, वह प्रत्यक्ष रूप से राज्य सभा के प्रनत ईत्तरदायी होता
है।

Q 31.C
 संनवधान और संसद ्ारा संसद सदस्य चुने जाने के निए नवनभन्न योग्यताएं ननधाभरितरत की गयी हैं।
 संनवधान में ननम्ननिनखत योग्यताएं ननधाभरितरत हैं:
o वह भारत का नागरितरक होना चानहए।
o ननवाभचन अयोग ्ारा आस नननमत्त प्रानधकृ त व्यनि के समक्ष शपथ या पुनि करनी होगी और ईसपर हस्ताक्षर करने होंगे।
आस शपथ या प्रनतज्ञान में शपथ िेता है कक वह (a) भारत के संनवधान के प्रनत स्च ी श्रृिा और ननष्ठा रखेगा (b) भारत की
सम्प्रभुता और ऄखंडता को बनाए रखेगा। आसनिए कथन 1 सही है।
o राज्य सभा के मामिे में ईसकी अयु 30 वषभ से कम नहीं होनी चानहए और िोक सभा के मामिे न्यूनतम अयु 25 वषभ
होनी चानहए।
o ईसके पास संसद ्ारा ननधाभरितरत ऄन्य योग्यताएं होनी चानहए।
 संसद ने जन प्रनतनननध ऄनधननयम (1951) में ननम्ननिनखत ऄनतरितरि योग्यताएं ननधाभरितरत की हैं:
o ईसे ककसी भी ससंदीय क्षेत्र का पंजीकृ त मतदाता होना चानहए। यह राज्यसभा और िोकसभा, दोनों के मामिों में समान
है। राज्यसभा के मामिे में, प्रत्याशी नजस राज्य से चुनाव िड़ रहा होता है, वहां का मतदाता होने की अवश्यकता को
2003 में समाप्त कर कदया गया था। 2006 में सवो्च न्यायािय ने आस परितरवतभन की वैधता की पुनि कर दी थी। आसनिए,
कथन 2 सही नहीं है।
o यकद वह ककसी राज्य या संघ राज्यक्षेत्र की अरनक्षत सीट से चुनाव िड़ रहा है तो ईसे ककसी राज्य या संघ राज्यक्षेत्र में
ऄनुसनू चत जानत या ऄनुसनू चत जनजानत का सदस्य होना चानहए। हािााँकक ककसी ऄनुसूनचत जानत या ऄनुसूनचत जनजानत
का सदस्य ऄनारनक्षत सीट से भी चुनाव िड़ सकता है। आसनिए, कथन 3 सही है।

Q 32.B
 के वि युग्म 2 और 3 सही सुमने ित हैं।
 1921 में फ्रेडरितरक ्हाआट और सनचदानन्द नसन्हा को भारत के गवनभर जनरि ्ारा के न्द्रीय नवधानसभा का प्रथम ऄध्यक्ष और
प्रथम ईपाध्यक्ष (िमशः) ननयुि ककया गया था।
 1925 में नवट्ठिभाइ जे. पटेि के न्द्रीय नवधान सभा के प्रथम भारतीय और प्रथम ननवाभनचत ऄध्यक्ष बने। 1935 तक कें द्रीय
नवधानसभा के ऄध्यक्ष और ईपाध्यक्ष को िमशः प्रेनसडेंट और नडप्टी प्रेनसडेंट कहा जाता था। 1935 के भारत सरकार
ऄनधननयम ने आन नामों को परितरवर्मतत कर िमशः स्पीकर और नडप्टी स्पीकर कर कदया। हािााँकक 1935 के ऄनधननयम के संघीय
भाग को िागू नहीं ककए जाने के कारण पुराने नाम 1947 तक प्रचनित रहे।
 जी.वी. माविंकर को िोकसभा के प्रथम ऄध्यक्ष होने का गौरव प्राप्त हुअ था जबकक एम.ए. ऄयंगर प्रथम ईपाध्यक्ष थे। जी.वी.
माविंकर संनवधान सभा (नवधायी) और ऄंतरितरम संसद में भी आस पद पर असीन रहे थे। ईन्होंने 1946 से 1965 के एक दशक
तक िगातार िोकसभा के ऄध्यक्ष का कायभभार सम्भािा।
 ऄध्यक्ष और ईपाध्यक्ष (प्रेनसडेंट और नडप्टी प्रेनसडेंट) के पद 1921 में भारत सरकार ऄनधननयम, 1919 (मोंटेग्यू चेम्सफोडभ
सुधार) के प्रावधानों के ऄंतगभत स्थानपत ककए गए थे। 1921 से पूवभ, कें द्रीय नवधायी परितरषदों की बैठकों की ऄध्यक्षता भारत के
गवनभर जनरि ्ारा की जाती थी।

9 www.visionias.in ©Vision IAS


Q 33.A
 जब राज्यपाि ककसी नवधेयक को राष्ट्रपनत के नवचार के निए अरनक्षत करता है तो नवधेयक के ऄनधननयमन में ईसकी ऄन्य कोइ
भूनमका नहीं रह जाती। नवधानमंडि के सदन या सदनों को पुनर्मवचार के निए राष्ट्रपनत ्ारा िौटाए गए नवधेयक को यकद
नवधानमंडि ्ारा पुनःपारितरत कर कदया जाता है, तो नवधेयक को सहमनत के निए पुनः राष्ट्रपनत के समक्ष प्रस्तुत करना
अवश्यक है। हािााँकक राष्ट्रपनत नवधेयक पर सहमनत देने के निए बाध्य नहीं है। राष्ट्रपनत ्ारा नवधेयक को सहमनत प्रदान करने
की नस्थनत में यह एक ऄनधननयम बन जाता है। आसका ऄथभ है कक राज्यपाि की सहमनत की ऄब कोइ अवश्यकता नहीं रहती है।
आसनिए, कथन 1 सही है और कथन 2 सही नहीं है।

Q 34.C
 कथन 1 सही है: संनवधान के ऄंतगभत, िोक सेवकों को ऄपने अनधकारितरक कायों के नवनधक ईत्तरदानयत्व से व्यनिगत ईन्मुनि
प्रदान की जाती है। आसका ऄथभ है कक कोइ िोक सेवक ऄपने पद से सम्बि अनधकारितरक कायभ के निए व्यनिगत रूप से
ईत्तरदायी नहीं होगा बनल्क ईसके निए सरकार (के न्द्रीय या राज्य) ईत्तरदायी होगी। परन्तु यकद कायभ संनवधान में ननर्ददि शतों
का पािन ककए नबना ककया जाता है तो ऐसा कायभ करने वािा िोक सेवक व्यनिगत रूप से ईत्तरदायी होगा।
 आसके ऄनतरितरि िोक सेवकों को सरकार के सम्प्रभु कायों के सम्बन्ध में ईनके ऄपकृ त्यों के निए क़ानूनी ईत्तरदानयत्व के नवरुि
ईन्मुनि प्राप्त है। ऄन्य मामिों में, ऄपकृ त्यों या ऄवैध कृ त्यों के निए नसनवि सेवकों का ईत्तरदानयत्व भी ककसी साधारण
नागरितरक के समान ही होता है।
 अनधकरितरक क्षमता में ककये गये ईनके ककसी भी कायभ के निए ईनके नवरुि दीवानी कायभवाही दो माह के ऄनरामम नोरितटस के
पश्चात ही प्रारम्भ की जा सकती है। परन्तु यकद कोइ कायभ, अनधकारितरक कायों के कायभक्षेत्र के बाहर ककया गया हो तो ऐसी
ककसी भी नोरितटस की अवश्यकता नहीं है।
 जहां भी अवश्यक हो, राष्ट्रपनत या राज्यपाि की पूवभ सहमनत के पश्चात् अनधकारितरक क्षमता में ककये गए कायों के निए
अपरानधक कायभवाही की जा सकती है।
 कथन 2 सही है: न्यानयक ऄनधकारितरयों को ऄपने अनधकारितरक कायों के निए ईन्मुनि प्राप्त है आसनिए ईन पर ऄनभयोग नहीं
चिाया जा सकता। न्यानयक ऄनधकारी सरं क्षण ऄनधननयम (1850) में प्रावधान ककया गया है कक, “ककसी भी न्यायाधीश,
मनजस्रेट, शांनत न्यायाधीश, किेक्टर या न्यानयक रूप में कायभ करने वािे ऄन्य ककसी व्यनि पर ईनके कतभव्य ननवभहन के दौरान
ककये गए ककसी भी कायभ के निए न्यायािय में ऄनभयोग नहीं चिाया जा सकता।”

Q 35.A
 भारतीय संनवधान के ऄनुछेद 1 के ऄंतगभत, जम्मू-कश्मीर राज्य (J&K) भारतीय संघ का एक ऄनवनच्छन्न राज्य है और आसका

क्षेत्र भारत के राज्यक्षेत्र का एक ऄंग है। दूसरी ओर, संनवधान के भाग XXI का ऄनुच्छेद 370 आसे एक नवशेष दजाभ प्रदान करता
है। तदनुसार, भारतीय संनवधान के कु छ प्रावधान (न कक सभी) आस पर िागू होते हैं। आसनिए, कथन 2 सही नहीं है।
 भारतीय संघ का यह एकमात्र ऐसा राज्य है नजसका ऄपना पृथक संनवधान है– ‘जम्मू और कश्मीर का संनवधान’। आसनिए,
कथन 1 सही है।

Q 36.D
 ‘नवपक्ष के नेता’ को ऄनधकारितरक रूप से पहिी बार मान्यता 1969 में प्रदान की गइ थी। हािााँकक िोकसभा और राज्यसभा में
नवपक्ष के नेता को सांनवनधक मान्यता 1977 में प्रदान की गयी। आसनिए, कथन 1 और 2 सही नहीं हैं।
 संसद के प्रत्येक सदन में एक ‘नवपक्ष का नेता’ होता है। ऐसे सबसे बड़े नवपक्षी दि के नेता को नवपक्ष के नेता के रूप में मान्यता
दी जाती है नजसके पास सदन की कु ि सदस्य संख्या के कम से कम दसवें भाग के बराबर सीटें हों। आसनिए कथन 3 सही है।

10 www.visionias.in ©Vision IAS


Q 37.B
 संनवधान ने ई्च न्यायिय के न्यायधीश का कायभकाि ननधाभरितरत नहीं ककया है। हािााँकक आस सम्बन्ध में आसमें ननम्ननिनखत
प्रावधान ककये गये हैं:
o वह 62 वषभ की अयु तक ही ऄपने पद पर बना रह सकता है। ईसकी अयु से संबंनधत ककसी भी प्रनगत पर ननणभय भारत के
मुख्य न्यायाधीश से परामशभ के पश्चात् राष्ट्रपनत ्ारा निया जाता है और राष्ट्रपनत का ननणभय ऄंनतम होता है।
o वह राष्ट्रपनत को निनखत संबोधन के माध्यम से त्यागपत्र दे सकता है | आसनिए, के वि कथन 2 ही सही है।
o राष्ट्रपनत ्ारा संसद की संस्तुनत के अधार पर ईसे ईसके पद से हटाया जा सकता है। यद्यनप ई्च न्यायािय का न्यायाधीश
राष्ट्रपनत के प्रसादपयंत कायभ नहीं करता है।
o जब ईसे ई्च तम न्यायािय का न्यायाधीश ननयुि ककया जाता है या ईसे ककसी ऄन्य ई्च न्यायािय को ऄंतरितरत ककया
जाता है तो ईसे ऄपना पद त्याग करना पड़ता है।
Q 38.D
 राष्ट्रपनत भारत के मुख्य न्यायाधीश से परामशभ करने के पश्चात् ककसी न्यायाधीश का एक ई्च न्यायािय से दूसरे ई्च न्यायािय
को ऄन्तरण कर सकता है। जब ईसका ऄन्तरण ककया जाता है तो वह ऄपने वेतन के ऄनतरितरि संसद ्ारा ननधाभरितरत
प्रनतकरात्मक भत्ता प्राप्त करने का भी हकदार होता है।
 1977 में ई्च तम न्यायािय ने ननणभय कदया था कक ई्च न्यायािय के न्यायाधीशों का ऄन्तरण के वि ऄपवादस्वरूप और के वि
जननहत में ही ककया जा सकता है, न कक दंड के रूप में। पुनः 1994 में, ई्च तम न्यायािय ने कहा कक न्यायाधीशों के ऄन्तरण में
मनमानी रोकने के निए न्यानयक समीक्षा अवश्यक है। ककन्तु आस मामिे को चुनौती के वि वही न्यायाधीश दे सकता है नजसका
ऄन्तरण ककया गया हो।
 तृतीय न्यायाधीश वाद (1998) में ई्च तम न्यायािय ने राय दी कक ई्च न्यायािय के न्यायाधीशों के ऄन्तरण के मामिे में
भारत के मुख्य न्यायाधीश को ई्च तम न्यायािय के चार वरितरष्ठतम न्यायाधीशों के कॉिेनजयम के ऄनतरितरि दो ई्च न्यायाियों
के मुख्य न्यायाधीशों (एक ई्च न्यायािय वह जहां से ऄन्तरण ककया जा रहा है और दूसरा ई्च न्यायािय वह जहां ऄन्तरण
ककया जा रहा है) से परामशभ िेना चानहए। आस प्रकार के वि भारत के मुख्य न्यायाधीश की राय से ही ‘परामशभ’ प्रकिया पूणभ नहीं
होती है।

Q 39.A
 भारत का राष्ट्रपनत प्रधानमंत्री के परामशभ के ऄनुसार मंनत्रपरितरषद के सदस्यों के मध्य नवभागों के अवंटन का ननदेश देता है।
मंनत्रयों को प्रधानमंत्री के परामशभ पर राष्ट्रपनत ्ारा ननयुि ककया जाता है।
 संसदीय मामिों की सनमनत का कायभ संसद में सरकारी कायभ-संचािन की प्रगनत पर नज़र रखना है। सनमनत को मंनत्रयों को पद
एवं नवभागों को अवंरितटत करने का कोइ प्रानधकार प्राप्त नहीं है।
 कै नबनेट की ननयुनि सनमनत कें द्रीय सनचवािय, सावभजननक ईपिमों, बैंकों और नवत्तीय संस्थानों में सभी ई्च स्तरीय ननयुनियों
का ननणभय करती है। सनमनत को मंनत्रयों को पद एवं नवभाग अवंरितटत करने का कोइ प्रानधकार प्राप्त नहीं है।

Q 40.D
 ई्च तम न्यायािय के एक न्यायाधीश को राष्ट्रपनत के अदेश पर ईसके पद से हटाया जा सकता है।
 राष्ट्रपनत ई्च तम न्यायािय के एक न्यायाधीश को पद से हटाने का अदेश तभी जारी कर सकता है जब संसद ईसको हटाए जाने
सम्बन्धी समावेदन ऄपने एक ही सत्र के दौरान पारितरत कर राष्ट्रपनत के पास भेजे।
 आस समावेदन को संसद के प्रत्येक सदन के नवशेष बहुमत ्ारा समर्मथत होना चानहए (ऄथाभत, ईस सदन की कु ि सदस्यता का
बहुमत और ईस सदन में ईपनस्थत और मत देने वािे सदस्यों के कम-से-कम दो-नतहाइ बहुमत ्ारा)।
 पद से हटाए जाने के दो अधार हैं – सानबत कदाचार ऄथवा ऄसमथभता।

Q 41.C
 इ-गवनेंस, सरकारी एजेंनसयों ्ारा सूचना प्रौद्योनगकी (जैसे वाआड एरितरया नेटवकभ , आं टरनेट और मोबाआि कं प्यूटटग) की
ईपयोनगता को संदर्मभत करता है नजसमें नागरितरकों, व्यवसायों और सरकार के नवनभन्न ऄंगों के मध्य संबंधों को रूपांतरितरत करने
की क्षमता है। आसके माध्यम से नागरितरकों को सरकारी सेवाओं की बेहतर प्रदायगी, व्यापार और ईद्योग के साथ ऄंत:किया में

11 www.visionias.in ©Vision IAS


सुधार तथा सूचना तक पहुंच के माध्यम से नागरितरक सशनिकरण होता है या दूसरे शब्दों में, सरकारी प्रबंधन ऄनधक दक्ष हो
जाता है। आसके परितरणामी िाभ, ननम्न भ्रिाचार, पारदर्मशता में वृनि, ऄनधक सुनवधा, राजस्व वृनि और िागत में कमी हैं।
 इ-गवनेंस के मुख्य िक्ष्य ननम्ननिनखत हैं:
o नागरितरकों को बेहतर सेवा प्रदायगी
o पारदर्मशता और जवाबदेही का अरं भ
o सूचना के जरितरए िोगों का सशनिकरण
o सरकारों के भीतर बेहतर दक्षता
o व्यावसाय और ईद्योग के साथ बेहतर आं टरफे स

Q 42.A
 राज्यसभा में राज्यों के प्रनतनननधयों को राज्य नवधानसभाओं के ननवाभनचत सदस्यों ्ारा ननवाभनचत ककया जाता है। चुनाव एकि
संिमणीय मत के माध्यम से अनुपानतक प्रनतनननधत्व प्रणािी के ऄनुसार अयोनजत ककया जाता है। आसनिए, कथन 1 सही है।
 राज्य सभा में संघ राज्यक्षेत्रों के प्रनतनननधयों को ननवाभचक मंडि (के वि प्रनतनननधयों के ननवाभचन के निए गरितठत) के सदस्यों
्ारा ऄप्रत्यक्ष रूप से ननवाभनचत ककया जाता है। यह चुनाव अनुपानतक प्रनतनननधत्व पिनत के ऄनुसार एकि संिमणीय मत
पिनत के माध्यम से अयोनजत ककया जाता है। आसनिए, कथन 2 सही नहीं है।
 सात संघ राज्यक्षेत्रों में से के वि दो (कदल्िी और पुडुचरे ी) का राज्यसभा में प्रनतनननधत्व है।

Q 43.C
 स्वायत्त ननकाय: ऐसे ननकाय जो सरकारी कायों से संबनं धत गनतनवनधयों के ननष्पादन हेतु सरकार ्ारा स्थानपत ककए जाते हैं।
यद्यनप ऐसे ननकायों को मेमोरें डम ऑफ एसोनसएशन अकद के ऄनुसार ऄपने कायों का ननष्पादन करने हेतु स्वायत्तता प्रदान
की जाती है, िेककन आन पर सरकार का ननयंत्रण होता है क्योंकक आन्हें भारत सरकार ्ारा नवत्त पोनषत ककया जाता है। ऄतः
कथन 1 सही है।
 संिग्न कायाभिय सामान्यतः ऄपने नवभाग ्ारा ननधाभरितरत नीनतयों के कायाभन्वयन हेतु अवश्यक कायभकारी ननदेश प्रदान करने के
निए ईत्तरदायी होते हैं। वे तकनीकी सूचनाओं के भंडार (संरामह) के रूप में कायभ करते हैं तथा व्यवहारितरक तकनीकी पहिुओं पर
नवभाग को परामशभ प्रदान करते हैं। ऄतः, कथन 2 सही है।

Q 44. A
 संनवधान, कें द्र और राज्यों में नननहत नवधायी शनियों की क्षेत्रीय सीमाओं को ननम्ननिनखत तरीकों से परितरभानषत करता है:
o संसद भारतीय भू-भाग के संपूणभ क्षेत्र या ईसके ककसी नवशेष क्षेत्र के निए कानून का ननमाभण कर सकती है। भारतीय भू -
भाग में राज्य और कें द्रशानसत प्रदेशों तथा समय के साथ भारतीय राज्यक्षेत्र में शानमि ककए जाने वािे कोइ ऄन्य क्षेत्र भी
सनम्मनित हैं।
o राज्य नवधानमंडि राज्य के संपण ू भ या ककसी नवशेष भाग के निए कानून बना सकती है। राज्य नवधानमंडि ्ारा बनाए गए
कानून राज्य के बाहर िागू नहीं होते हैं, ककन्तु ऄपवादस्वरूप, ये ईस दशा में राज्य के बाहर िागू हो सकते हैं जब ईि
राज्य और ईस क़ानून की नवषयवस्तु के मध्य पयाभप्त सम्बन्ध (sufficient nexus) मौजूद हो। आस प्रकार, कथन 2 सही
नहीं है।
o के वि संसद को ही ‘राज्य क्षेत्रातीत कानून’ ननमाभण की शनि प्राप्त है। आस प्रकार, संसदीय कानून नवि के ककसी भी भाग में
नस्थत भारतीय नागरितरकों और ईनकी संपनत्त पर िागू होते हैं। आस प्रकार, कथन 1 सही है।

Q 45.C
 सरकार की संसदीय व्यवस्था की कायभ प्रणािी का मूिभूत नसिांत सामूनहक ईत्तरदानयत्व का नसिांत है।
 ऄनुच्छेद 75 के ऄनुसार, मंनत्रपरितरषद िोकसभा के प्रनत सामूनहक रूप से ईत्तरदायी होती है। आसका ऄथभ है कक सभी मंनत्रयों का
ईनके समस्त कायों के निए िोक सभा के प्रनत संयुि ईत्तरदानयत्व होता है। आस प्रकार, कथन 1 सही है।
12 www.visionias.in ©Vision IAS
 मंनत्रमंडि के ननणभय सभी कें द्रीय मंनत्रयों (और ऄन्य मंनत्रयों) के निए बाध्यकारी होते हैं, यहां तक कक तब भी जब मंनत्रमंडि की
बैठक में ईनके नवचार आसके नवरुि हों। सभी मंनत्रयों का कत्तभव्य है कक वे मंनत्रमंडि के ननणभयों का ऄनुसरण करें और संसद के
भीतर व बाहर ईनका समथभन करें । यकद कोइ भी मंत्री, मंनत्रमंडि के ननणभय से ऄसहमत है और ईसके समथभन हेतु तैयार नहीं है
तो ईसे त्यागपत्र देना होता है। आस प्रकार, कथन 2 सही है।
Q 46.B
 संघ सूची में युि एवं शांनत, नौसेना, थि-सेना व वायु सेना से संबनं धत मामिे; नवदेशी क्षेत्रानधकार; नागरितरकता, देशीयकरण
और नवदेशी व्यनियों से सम्बंनधत मामिे अकद नवषय सनम्मनित हैं।
 राज्य सूची में कृ नष, पुनिस, कारागार, स्थानीय शासन, सावभजननक स्वास्थ्य, भूनम, मकदरा जैसे नवषय सनम्मनित हैं।
 समवती सूची में नशक्षा, कृ नष भूनम, वन अकद के ऄनतरितरि संपनत्त का हस्तांतरण, रेड यूननयन जैसे नवषय सनम्मनित हैं।
 ऄवनशि शनियों में ऐसे नवषय सनम्मनित हैं जो ककसी भी सूची में ईनल्िनखत नहीं हैं, जैसे साआबर कानून। ऐसे मामिों पर
नवधान बनाने का ऄनधकार के वि संसद को है।

Q 47.A
 पांचवीं ऄनुसच
ू ी: आसमें राज्यों के ऄनुसूनचत क्षेत्रों व ऄनुसूनचत जनजानतयों के प्रशासन व ननयंत्रण सम्बन्ध में प्रावधान हैं
(प्रनतकू ि परितरनस्थनतयों के कारण आन क्षेत्रों और जनजानतयों को नवशेष संरक्षण की अवश्यकता है)।
 छठी ऄनुसच
ू ी: आसमें ऄसम, मेघािय, नत्रपुरा, नमजोरम में जनजातीय क्षेत्रों के प्रशासन के संबंध में प्रावधान है।

Q 48.B
 संनवधान, कठोरता को कम करने और गनतरोध की नस्थनत से बचने के निए कायभकारी कृ त्यों के ऄंतर-सरकारी प्रत्यायोजन का
प्रावधान करता है।
 तदनुसार, राष्ट्रपनत राज्य सरकार की सहमनत से कें द्र सरकार के ककसी भी कायभकारी कृ त्य को ईस राज्य को सौंप सकता है।
आसनिए, कथन 1 सही नहीं है।
 आसके नवपरीत, ककसी राज्य का राज्यपाि कें द्र की सहमनत से ईस राज्य के ककसी भी कायभकारी कृ त्य को कें द्र सरकार को सौंप
सकता है। आसनिए, कथन 2 सही है।
 प्रशासननक प्रकायों का यह पारस्परितरक प्रत्यायोजन सशतभ या शतभरनहत हो सकता है।
 संनवधान ककसी राज्य को ईसकी सहमनत के नबना कें द्र के कायभकारी कृ त्यों को सौंपने का प्रावधान भी करता है। िेककन, आस
मामिे में प्रत्यायोजन संसद ्ारा ककया जाता है, न कक राष्ट्रपनत ्ारा। आस प्रकार, संघ सूची के नवषय पर संसद ्ारा बनाए
गए कानून ककसी राज्य को शनियां प्रदान कर सकते हैं और कतभव्यों को ऄनधरोनपत कर सकते हैं, या कें द्र सरकार को ककसी
राज्य को शनियां प्रदान करने और कतभव्यों को िागू करने के निए ऄनधकृ त कर सकते हैं (संबंनधत राज्य की सहमनत पर ध्यान
कदए नबना)। यह ध्यान देने योग्य है कक आसी कायभ को राज्य नवधानयका ्ारा नहीं ककया जा सकता है।

Q 49.D
 भारतीय संनवधान के ऄनुच्छेद 75 में कहा गया है, कक प्रधानमंत्री को राष्ट्रपनत ्ारा ननयुि ककया जाएगा।
 कथन 1 सही नहीं है: 1980 में, कदल्िी ई्च न्यायािय ने यह ननणभय कदया कक संनवधान आस बात का कोइ प्रावधान नहीं करता
है कक ककसी व्यनि को प्रधानमंत्री के रूप में ननयुि होने से पहिे िोकसभा में ऄपना बहुमत सानबत करना होगा। राष्ट्रपनत पहिे
ईसे प्रधानमंत्री ननयुि कर सकता है और कफर ईसे एक ईनचत ऄवनध के भीतर िोकसभा में ऄपना बहुमत सानबत करने के निए
कह सकता है।
 कथन 2 सही नहीं है: 1997 में सवो्च न्यायािय ने ननणभय कदया कक एक व्यनि जो संसद के ककसी भी सदन का सदस्य नहीं है,
ईसे छह महीने की ऄवनध के निए प्रधानमंत्री ननयुि ककया जा सकता है। आस ऄवनध के भीतर ईसे संसद के ककसी एक सदन का
सदस्य बनना होगा ऄन्यथा वह आसके बाद प्रधानमंत्री के पद पर बना नहीं रह सकता।

13 www.visionias.in ©Vision IAS


Q 50.B
 ननवाभचन अयोग, चुनावों के ईद्देश्य से राजनीनतक दिों को पंजीकृ त करता है और ईनके चुनाव प्रदशभन के अधार पर ईन्हें
राष्ट्रीय या राज्य दिों के रूप में मान्यता प्रदान करता है। ऄन्य दिों को मात्र पंजीकृ त गैर-मान्यता प्राप्त दिों के रूप में घोनषत
ककया जाता है। आसनिए, कथन 1 सही नहीं है।
 ननवाभचन अयोग ्ारा दिों को प्रदत्त मान्यता कु छ नवशेषानधकारों के प्रनत ईनके ऄनधकार को ननधाभरितरत करती है जैसे- दिों को
ईनके चुनाव नचह्नों का अवंटन, राज्य के स्वानमत्व वािे टेिीनवजन और रे नडयो स्टेशनों पर राजनीनतक प्रसारण के निए समय
का प्रावधान और मतदाता सूनचयों तक पहुंच अकद। आसनिए, कथन 2 सही है।

Q 51.C
 73वां संवैधाननक संशोधन, प्रत्येक स्तर पर पंचायत के निए पांच वषभ के कायभकाि का प्रावधान करता है। हािांकक, आसे ऄपने
कायभकाि के पूणभ होने से पहिे भंग ककया जा सकता है। आसके ऄनतरितरि, पंचायत का गठन करने के निए नये चुनावों को (a)
आसकी पांच वषभ की ऄवनध समाप्त होने से पहिे; या (b) नवघटन के मामिे में, आसके नवघटन से 6 महीने की ऄवनध समाप्त होने
से पहिे पूरा ककया जाएगा।
 िेककन, जहां शेष ऄवनध (नजसके निए नवघरितटत पंचायत जारी रहती) छह महीने से भी कम होती है, वहां आस ऄवनध के निए
नइ पंचायत का गठन करने के निए ककसी भी चुनाव का अयोजन ऄननवायभ नहीं होगा। आसके ऄनतरितरि, कायभकाि के समाप्त
होने से पहिे ककसी पंचायत के नवघटन के कारण गरितठत होने वािी पंचायत, के वि ईस शेष कायभकाि के निए जारी रहेगी
नजसके निए नवघरितटत होने वािी पंचायत जारी रहती, यकद वह भंग न हुइ होती। दूसरे शब्दों में, समय पूवभ नवघटन के कारण
पुनगभरितठत पंचायत पांच वषभ के पूणभ कायभकाि की ऄवनध तक नहीं, ऄनपतु के वि शेष ऄवनध के निए कायभ करती है।

Q 52.D
 इ-गवनेंस, शासन में नवनभन्न नहतधारकों के मध्य ऄंत:किया को सुगम बनाता है। आन ऄंतःकियाओं को ननम्ननिनखत रूप से
वर्मणत ककया जा सकता है:
 G2G (सरकार से सरकार) - आस मामिे में, सूचना एवं संचार प्रौद्योनगकी का ईपयोग के वि सरकारी संस्थाओं की कायभप्रणािी
में संिग्न सरकारी प्रकियाओं को पुनसंरनचत करने के निए ही नहीं ककया जाता है ऄनपतु नवनभन्न संस्थाओं के भीतर और ईनके
मध्य सूचना व सेवाओं के प्रवाह को बढ़ाने के निए भी ककया जाता है। आस प्रकार की ऄंतःकिया के वि सरकार के क्षेत्र में ही
घरितटत होती हैं और ये ऄंतःकिया, क्षैनतज ऄथाभत् नवनभन्न सरकारी एजेंनसयों के बीच तथा आसके साथ-साथ एक संगठन के भीतर
नवनभन्न कायाभत्मक क्षेत्रों के बीच; या िम्बवत ऄथाभत राष्ट्रीय, प्रांतीय व स्थानीय सरकारी एजेंनसयों के बीच तथा आसके साथ-
साथ एक संगठन में नवनभन्न स्तरों के बीच, दोनों प्रकार से हो सकती हैं। आसका प्राथनमक ईद्देश्य दक्षता, ननष्पादन और परितरणाम
(अईटपुट) में वृनि करना है।
 G2C (सरकार से नागरितरक) - आस मामिे में, सरकार और नागरितरकों के बीच एक आं टरफे स का ननमाभण ककया जाता है जो
नागरितरकों को नवनभन्न प्रकार की सावभजननक सेवाओं के कु शि नवतरण से िाभ प्राप्त करने में सक्षम बनाता है। यह एक तरफ
सावभजननक सेवाओं की ईपिब्धता और ऄनभगम्यता का नवस्तार करता है और दूसरी तरफ सेवाओं की गुणवत्ता में सुधार करता
है। यह नागरितरकों को आस बात का नवकल्प देता है कक वे सरकार के साथ कब (जैसे- कदन के चौबीस घंटे, सप्ताह के सातों कदन),
कहााँ (जैस-े सेवा कें द्र, ऄप्रयुि ककयोस्क या ककसी के घर/कायभस्थि से) और कै से (जैसे- आं टरनेट, फै क्स, टेिीफोन, इ-मेि, अमने-
सामने अकद के माध्यम से) संपकभ स्थानपत कर सकते हैं। आसका प्राथनमक ईद्देश्य सरकार को नागरितरक-ऄनुकूि बनाना है।
 G2B (सरकार से व्यवसाय) - यहां, इ-गवनेंस ईपकरणों का ईपयोग व्यापार समुदाय – वस्तुओं व सेवाओं के प्रदाताओं - की
सहायता के निए ककया जाता है, ताकक वे ननबाभध रूप से सरकार के साथ संपकभ कर सकें । आसका ईद्देश्य सरकार के साथ
संव्यवहार करते समय िािफीताशाही में कमी, समय की बचत, परितरचािन िागतों को कम करना और ऄनधक पारदशी
व्यावसानयक वातावरण का ननमाभण करना है। G2B पहिें िेन-देन से संबंनधत हो सकती हैं जैसे कक िाआसेंलसग, परनमट, खरीद
और राजस्व संरामह में होती हैं। वे प्रचारक और सुनवधाप्रदाता भी हो सकती हैं जैसे कक व्यापार, पयभटन और ननवेश में होती हैं। ये

14 www.visionias.in ©Vision IAS


ईपाय व्यवसायों को एक ऄनुकूि वातावरण प्रदान करने में सहायता करते हैं ताकक ईन्हें ऄनधक कु शितापूवक
भ प्रदशभन करने हेतु
सक्षम बनाया जा सके ।
 G2E (सरकार से कमभचारी) - सरकार ऄब तक की सबसे बड़ी ननयोिा है और ककसी ऄन्य संगठन की तरह, आसे ऄपने
कमभचारितरयों से ननयनमत रूप से संपकभ स्थानपत करना होता है। संगठन और कमभचारी के मध्य यह ऄंतःकिया दो-तरफा प्रकिया
होती है। अइसीटी ईपकरणों का ईपयोग एक तरफ ऄंतःकिया को तीव्र व कु शि बनाने में सहायक है तो वहीं दूसरी तरफ
कमभचारितरयों के संतनु ि स्तर में भी वृनि करता है।
Q 53.A
 ऄनुच्छेद 371 के तहत, राष्ट्रपनत को यह प्रावधान करने हेतु ऄनधकृ त ककया गया है कक महाराष्ट्र और गुजरात के राज्यपाि
ननम्ननिनखत क्षेत्रों में पृथक नवकास बोडों को स्थानपत करने के निए नवशेष तौर पर पर ईत्तरदायी होंगे:
o नवदभभ, मराठवाड़ा और शेष महाराष्ट्र,
o सौराष्ट्र, कच्छ और शेष गुजरात।

Q 54.B
 संसद के ककसी भी सदन के सदस्य, संसद ्ारा ननधाभरितरत वेतन और भत्ते प्राप्त करने के ऄनधकारी होते हैं िेककन संनवधान में
पेंशन का कोइ प्रावधान नहीं है। आसनिए, कथन 2 सही है। (हािांकक, संसद ने संसद सदस्य वेतन, भत्ता और पेंशन ऄनधननयम
के तहत सदस्यों को पेंशन प्रदान की है।)

Q 55.C
 सेवोत्तम वस्तुत: दो लहदी शब्दों 'सेवा + ईत्तम' का संयोजन है, नजसका ऄथभ है ईत्तम सेवा ऄथाभत् सेवाओं में ईत्कृ िता। आसमें मूि
रूप से तीन घटक हैं:
o नसटीजन चाटभर (नागरितरक ऄनधकार पत्र)
o िोक नशकायत ननवारण तंत्र
o सेवा प्रदायगी क्षमता।
 न्तीय प्रशासननक सुधार अयोग ने ऄपनी 12वीं रितरपोटभ "नागरितरक कें कद्रत प्रशासन" में यह संस्तुनत की है कक संघ और राज्य
सरकारों को सावभजननक क्षेत्र से संबि सभी संगठनों के निए सेवोत्तम को ऄननवायभ बनाना चानहए। आसका ईद्देश्य सावभजननक
सेवा प्रदायगी में ईत्कृ िता िाने के निए मूल्यांकन सुधार फ्रेमवकभ प्रदान करना है। यह मॉडि अंतरितरक प्रकियाओं की गुणवत्ता
का अकिन करने और सेवा प्रदायगी की गुणवत्ता पर ईनके प्रभाव का अकिन करने हेतु एक मूल्यांकन तंत्र के रूप में कायभ
करता है।
 सेवोत्तम फ्रेमवकभ , वषभ 2005 में भारत सरकार के प्रशासननक सुधार व िोक नशकायत नवभाग ्ारा बनाया गया था। भारतीय
मानक 15700:2005 को संति
ु करने वािा संगठन "सेवोत्तम" प्रमाणन प्राप्त करने का ऄनधकारी होगा, "सेवोत्तम" सेवा
प्रदायगी में ईत्कृ िता के निए एक भारतीय नाम है।

Q 56.A
 जब िोकसभा नवघरितटत हो जाती है, तो आसके या आसकी सनमनतयों के समक्ष िंनबत नवधेयक, प्रस्ताव, संकल्प, नोरितटस,
यानचकाओं अकद सनहत सभी कायभ व्यपगत हो जाते हैं। ईन्हें पुनः पारितरत कराने के निए नवगरितठत िोकसभा में पुरःस्थानपत
ककया जाना चानहए। यद्यनप, कु छ िंनबत नवधेयक तथा सभी िंनबत अिासनों नजनकी सरकारी अिासनों संबंधी सनमनत ्ारा
जांच की जानी हो, िोकसभा के नवघटन पर व्यपगत नहीं होते। नवधेयकों के व्यपगत होने के संबंध में नस्थनत ननम्नानुसार हैः
o िोकसभा में िंनबत नवधेयक व्यपगत हो जाता है (चाहे ईसका प्रस्ताव िोकसभा में रखा गया हो या कफर ईसे, राज्यसभा
्ारा िोकसभा में प्रेनषत ककया गया हो)।
o िोकसभा ्ारा पारितरत परं तु राज्यसभा में िंनबत नवधेयक व्यपगत हो जाता है।
o कोइ नवधेयक यकद ऄसहमनत के कारण दोनों ही सदनों में पारितरत न ककया गया हो तथा यकद राष्ट्रपनत ने िोकसभा के
नवघटन से पहिे संयुि बैठकों के अयोजन को ऄनधसूनचत ककया हो, तो वह व्यपगत नहीं होता है।
o राज्यसभा में िंनबत परं तु िोकसभा ्ारा पारितरत नहीं ककया गया नवधेयक व्यपगत नहीं होता है।
o दोनों सदनों ्ारा पारितरत परं तु राष्ट्रपनत की स्वीकृ नत के निए िंनबत नवधेयक व्यपगत नहीं होता है।
o दोनों सदनों ्ारा पारितरत परं तु राष्ट्रपनत ्ारा पुनर्मवचार के निए िौटाया गया नवधेयक व्यपगत नहीं होता है।
15 www.visionias.in ©Vision IAS
Q 57.D
 जून 1983 में न्यायमूर्मत अर. एस. सरकारितरया अयोग की ननयुि की गइ थी। अयोग ने सहकारी संघवाद पर बि देते हुए स्पि
ककया कक एक नस्थर संस्थागत ऄवधारणा की तुिना में संघवाद सहकारी कायभवाही के निए ज्यादा कियात्मक व्यवस्था है। साथ
ही, आसके सशि कें द्र का मतिब यह नहीं बताया कक शनियों का कें द्रीकरण हो, क्योंकक कें द्र में शनियों के ऄनधक संकेन्द्रण से
ननणभय िेने का दबाव रहता है जबकक राज्य ननणभयनवहीन हो जाते हैं।
 एम. एम. पुंछी अयोग की स्थापना यूपीए सरकार के ्ारा 2007 में की गयी थी। आसने 2010 में ऄपनी ऄनुशस
ं ाएं प्रस्तुत कीं।
कु छ महत्वपूणभ ऄनुशंसाएं ननम्नानुसार हैं:
o समवती सूची के नवषयों पर कानून ननमाभण के निए ऄंतराभज्यीय परितरषद के माध्यम से संघ व राज्यों के मध्य एक परामशभ
प्रकिया होनी चानहए।
o राज्य के नवधेयकों के संबंध में, राष्ट्रपनत के पॉके ट वीटो को समाप्त कर एक ईनचत ऄवनध नननश्चत की जानी चानहए (6
महीने), नजसमें राष्ट्रपनत ऄपने ननणभय को संप्रेनषत कर सकें ।
o संघ की संनध करने की शनियों को नवननयनमत ककया जाना चानहए तथा राज्यों को ईन संनधयों में, जहां राज्यों के नहत
सनम्मनित हों, ऄनधक भागीदारी नमिनी चानहए ।
o राज्यपाि को मुख्यमंत्री की ननयुनि के संबंध में स्पि कदशा-ननदेश कदया जाना चानहए ताकक वह आस संदभभ में ऄपनी
नववेकानधकार का दुरुपयोग न कर सके , आत्याकद।
 1969 में तनमिनाडु में द्रमुक सरकार ्ारा राजमन्नार सनमनत की स्थापना की गइ थी। ऄंतराभज्यीय परितरषद के तत्काि गठन की
संस्तुनत करने के ऄनतरितरि आस सनमनत ने ननम्ननिनखत संस्तुनतयां कीः
o कें द्र सरकार को ऄंतराभज्यीय परितरषद से परामशभ ककये नबना ऐसा कोइ ननणभय नहीं िेना चानहए जो एक या एक से ऄनधक
राज्यों के नहतों को प्रभानवत कर सकता है।
o राज्यों के नहतों को प्रभानवत करने वािे प्रत्येक नवधेयक को संसद में प्रस्तुत ककए जाने से पूवभ ऄंतराभज्यीय परितरषद को
संदर्मभत ककया जाना चानहए।
o ऄनुच्छेद 356 का प्रयोग राज्य में कानून व व्यवस्था के पूणभतः नवफि हो जाने की ऄसामान्य नस्थनत में ही ककया जाना
चानहए, अकद।

Q 58.D
 कदल्िी, मुब
ं इ, कोिकाता, हैदराबाद, बैंगिोर जैसे बड़े शहरों के प्रशासन के निए नगर ननगम गरितठत ककए जाते हैं। राज्यों में
आसकी स्थापना संबनं धत राज्य नवधानमंडि के ऄनधननयम ्ारा तथा कें द्रशानसत प्रदेशों में आसकी स्थापना संसद के ऄनधननयम
्ारा की जाती है। एक राज्य में सभी नगर ननगमों के निए एक समान ऄनधननयम हो सकता है या प्रत्येक नगर ननगम के निए
एक पृथक ऄनधननयम। ऄतः, कथन 1 सही नहीं है तथा कथन 2 सही है।
 एक नगर ननगम के पास तीन प्रानधकरण होते हैं - परितरषद, स्थायी सनमनतयां एवं अयुि। परितरषद का नेतत्ृ व महापौर के ऄधीन
होता है। ईन्हें एक ईप महापौर ्ारा सहायता नमिती है। ऄनधकांश राज्यों में वे एक वषभ की नवीकरणीय ऄवनध के निए चुने
जाते हैं। यह मूि रूप से एक सांकेनतक पद होता है तथा महापौर, ननगम का एक औपचारितरक प्रमुख है। ईनका मुख्य कायभ
परितरषद की बैठकों की ऄध्यक्षता करना है।
 नगर अयुि (Municipal Commissioner) परितरषद व आसकी स्थायी सनमनतयों ्ारा ककए गए ननणभयों के कायाभन्वयन के
निए ईत्तरदायी है। आस प्रकार, वह ननगम का मुख्य कायभकारी प्रानधकारी होता है। ईसकी ननयुनि राज्य सरकार ्ारा की जाती
है तथा वह प्रायः भारतीय प्रशासननक सेवा (IAS) का सदस्य होता है। ऄतः, कथन 3 सही नहीं है।

Q 59.B
 नजिा न्यायाधीश के रूप में ननयुि होने वािे व्यनि की ननम्ननिनखत योग्यताएं होनी चानहए:
o ईसे पहिे से ही कें द्र या राज्य सरकार की सेवा में नहीं होना चानहए।
o ईसे कम से कम सात वषभ तक एक ऄनधविा या प्िीडर होना चानहए ।
16 www.visionias.in ©Vision IAS
o ननयुनि के निए ई्च न्यायािय ्ारा ईसकी संस्तुनत की जानी चानहए।
 एक राज्य में नजिा न्यायाधीशों की ननयुनि, पदस्थापना और प्रोन्ननत ई्च न्यायािय के परामशभ से राज्य के राज्यपाि ्ारा की
जाती है।
Q 60.D
 एक व्यनि एक ही समय में संसद के दोनों सदनों का सदस्य नहीं हो सकता है। ऄतः, जन प्रनतनननधत्व ऄनधननयम (1951) के
ऄंतगभत ननम्ननिनखत प्रावधान ककए गए हैं:
o यकद ककसी सदन का एक वतभमान सदस्य दूसरे सदन के निए भी ननवाभनचत हो जाता है, तो पहिे सदन में ईसकी सीट रितरि
हो जाती है।
o यकद कोइ व्यनि संसद के दोनों सदनों के निए ननवाभनचत हो जाता है, तो ईसे 10 कदनों के भीतर यह सूनचत करना होता है
कक वह ककस सदन की सेवा करना चाहता है। आस प्रकार की सूचना न नमिने पर राज्यसभा में ईसकी सीट स्वतः ही रितरि
मान िी जाती है।
o यकद कोइ व्यनि सदन में दो सीटों के निए ननवाभनचत होता है, तो ईसे ऄपने नवकल्प के रूप में एक सीट का चयन करना
होता है। ऄन्यथा, दोनों सीटें रितरि हो जाती हैं।
o आसी प्रकार, एक व्यनि एक ही समय में संसद तथा राज्य नवधानमंडि, दोनों का सदस्य नहीं हो सकता है। यकद कोइ
व्यनि संसद तथा राज्य नवधानमंडि दोनों में ननवाभनचत होता है तथा यकद वह 14 कदनों के भीतर राज्य नवधानमंडि में
ऄपनी सीट से आस्तीफा नहीं देता है, तो संसद में ईसकी सीट रितरि हो जाती है। ऄतः, सही ईत्तर (d) है।

Q 61.A
 ननवाभचन अयोग मीनडया को नवननयनमत नहीं करता है। हािांकक, कानून के प्रावधानों या न्यायािय के ननदेशों (जो मीनडया या
मीनडया के कामकाज के कु छ पहिुओं से संबंनधत हो सकता है) को िागू करने की नजम्मेदारी अयोग की है।
o जन प्रनतनननधत्व ऄनधननयम, 1951 की धारा 126A ननर्ददि ऄवनध (ऄथाभत, प्रथम चरण में चुनाव अरं भ होने के निए तय
समय तथा सभी राज्यों व कें द्र-शानसत प्रदेशों में ऄंनतम चरण के निए मतदान समाप्त होने के अधे घंटे बाद तक) के दौरान
एनग्जट पोि के संचािन तथा ईनके परितरणामों का प्रसार करने पर रोक िगाती है। ऄतः, कथन 2 सही नहीं है।
o जन प्रनतनननधत्व ऄनधननयम, 1951 की धारा 126, मतदान के समापन के निए ननधाभरितरत समय से 48 घंटे पूवभ की ऄवनध
के दौरान नसनेमाघरों, टेिीनवजन या ऄन्य समान साधनों के माध्यम से ककसी भी प्रकार के चुनावी मामिे को प्रदर्मशत
करने पर रोक िगाती है। ऄतः, कथन 1 सही है।
o जन प्रनतनननधत्व ऄनधननयम, 1951 की धारा 127A: चुनावी पैंफिेट, पोस्टरों आत्याकद का मुद्रण एवं प्रकाशन आसके
प्रावधानों ्ारा शानसत है, नजसमें लप्रटर एवं प्रकाशक का नाम व पता प्रदर्मशत करना ऄननवायभ है।

Q 62.A
 ऄंतराभज्यीय नदी जि नववाद ऄनधननयम, कें द्र सरकार को (संसद को नहीं) ककसी ऄंतराभज्यीय नदी या नदी घाटी के जि को
िेकर दो ऄथवा दो से ऄनधक राज्यों के मध्य नवद्यमान नववाद के न्यायननणभयन हेतु एक तदथभ न्यायानधकरण गरितठत करने का
ऄनधकार प्रदान करता है। न्यायानधकरण का ननणभय ऄंनतम होगा तथा नववाद से संबंनधत सभी पक्षों के निए बाध्यकारी होगा।
न्यायानधकरण को सौंपे गये ककसी भी जि नववाद के संबंध में न तो ई्च तम न्यायािय और न ही ककसी ऄन्य न्यायािय का
क्षेत्रानधकार होगा। ऄब तक कें द्र सरकार ने अठ ऄंतराभज्यीय जि नववाद न्यायानधकरण गरितठत ककए हैं। आसनिए, के वि कथन 1
ही सही है।

Q 63.D
 करों की शुि प्रानप्तयां जो भारत की संनचत नननध का भाग नहीं है, ईसमें ननम्न कर तथा शुल्क सनम्मनित हैं:
o नवननमय पत्रों, चेक़, प्रॉनमसरी नोट, बीमा पॉनिसी, शेयरों का हस्तांतरण तथा ऄन्य पर िगाया जाने वािा स्टाम्प शुल्क।
17 www.visionias.in ©Vision IAS
o ऄंतराभज्यीय वानणज्य या व्यापार के दौरान वस्तुओं के िय-नविय पर अरोनपत कर (समाचार पत्रों को छोड़ कर)।
o ऄंतराभज्यीय वानणज्य तथा व्यापार के दौरान वस्तुओं के परे षण पर अरोनपत कर।
o कृ नष अय, कृ नष भूनम के ऄनुिमण तथा भू-संपनत्त पर अरोनपत कर।
Q 64.A
 चुनाव की ‘फस्टभ पास्ट द पोस्ट’ प्रणािी -
 देश को छोटी भौगोनिक आकाआयों में बांटा गया है। आन आकाआयों को ननवाभचन क्षेत्र कहा जाता है। प्रत्येक ननवाभचन क्षेत्र से एक
प्रनतनननध को ननवाभनचत ककया जाता है।
 जीतने वािे प्रत्याशी को कु ि मतों का बहुमत (ऄथाभत, 50 प्रनतशत + 1 मत) हानसि करने की अवश्यकता नहीं होती। चुनावी
दौड़ में जो प्रत्याशी ऄन्य प्रत्यानशयों से अगे ननकि जाता है वही नवजयी होता है।
 आस प्रणािी को बहुिवादी प्रणािी भी कहा जाता है।
 आस प्रणािी को भारत तथा यूनाआटेड ककगडम में ऄपनाया गया है।
 आसनिए, कथन 1 सही है, कथन 2 सही नहीं है।

Q 65.A
 राज्य सभा एक ननरं तर चिने वािा सदन है। ऄथाभत्, यह एक स्थायी संस्था है तथा आसे भंग नहीं ककया जा सकता। आसनिए,
कथन 1 सही है।
 संनवधान ने राज्यसभा सदस्यों की पदावनध नननश्चत नहीं कर ईसे संसद पर छोड़ कदया था। तदनुसार, संसद ्ारा जन
प्रनतनननधत्व ऄनधननयम (1951) के तहत राज्य सभा के सदस्यों की पदावनध 6 वषभ ननधाभरितरत की गइ है। आसनिए, कथन 2 सही
नहीं है।

Q 66.A
 73वें संशोधन, 1992 ्ारा भारतीय संनवधान में एक नवीन भाग 9 ऄंतःस्थानपत ककया गया। ‘पंचायत’ के रूप में नानमत आस
भाग में ऄनुच्छेद 243 से 243-O तक के प्रावधान समानवि हैं। आसके ऄनतरितरि, आस ऄनधननयम ्ारा संनवधान में एक नयी
ऄनुसूची (ग्यारहवीं ऄनुसच
ू ी) को भी जोड़ा गया है। आस ऄनुसच
ू ी में पंचायतों के 29 प्रकायाभत्मक नवषय समानहत हैं नजनका
संबंध ऄनुच्छेद 243-G से है।
 ऄनुच्छेद 243 के ऄनुसार रामाम, मध्यवती तथा नजिा स्तर के सभी सदस्य जनता िोगों ्ारा प्रत्यक्ष रूप से ननवाभनचत ककए
जाएंग।े आसके ऄनतरितरि, मध्यवती तथा नजिा स्तर की पंचायतों के ऄध्यक्ष – ननवाभनचत सदस्यों ्ारा ईन्हीं में से – ऄप्रत्यक्ष रूप
से ननवाभनचत होंगे। यद्यनप, रामाम स्तर पर पंचायत का मुनखया राज्य नवधानमंडि ्ारा ननधाभरितरत रीनत से ननवाभनचत ककया
जाएगा।
 आसनिए, के वि कथन 1 सही है।

Q 67.C
 ई्च तम न्यायािय की स्वतंत्र तथा ननष्पक्ष कायभपिनत सुनननश्चत तथा संरनक्षत करने के निए संनवधान में ननम्ननिनखत प्रावधान
ककए गए हैं:
o कायभपानिका से पृथिरण: संनवधान राज्य को यह ननदेश देता है कक वह िोक सेवाओं के नवषय में न्यायपानिका की
कायभपानिका से पृथकता सुनननश्चत करे । ऄथाभत्, कायभपानिका के ऄनधकारितरयों को न्यानयक शनियां प्राप्त नहीं होंगी।
परितरणामतः, आस प्रावधान के कायाभन्वयन के कारण, न्यानयक प्रशासन में कायभपानिका के ऄनधकारितरयों की भूनमका समाप्त
हो गयी।
o संनचत नननध पर भारितरत व्यय: न्यायाधीशों तथा ऄन्य कमभचारितरयों के वेतन, भत्ते एवं पेंशन तथा ई्च तम न्यायािय के
प्रशासननक व्यय भारत की संनचत नननध पर भारितरत ककये गए हैं। ऄतः आस व्यय की मदें संसद में मतदान योग्य नहीं हैं।
(तथानप ईन पर चचाभ की जा सकती है)।
18 www.visionias.in ©Vision IAS
o आसके न्यानयक क्षेत्रानधकार को सीनमत नहीं ककया जा सकता: संसद को ई्च तम न्यायािय के न्यानयक क्षेत्रानधकार तथा
शनियों पर सीमाएं अरोनपत करने का ऄनधकार प्राप्त नहीं है। संनवधान ने ई्च तम न्यायािय को नवनवध प्रकार के न्यानयक
क्षेत्रानधकार प्रदान ककए हैं। हािांकक, संसद आसके क्षेत्रानधकार में वृनि कर सकती है।
Q 68.C
 74वें संशोधन ऄनधननयम (ऄनुच्छेद 243 ZD) के ऄनुसार प्रत्येक राज्य, नजिा स्तर पर पंचायतों तथा नगरपानिकाओं ्ारा
तैयार की गयी योजनाओं को समेककत करने तथा और सम्पूणभ नजिे के निए एक नवकास योजना का प्रारूप तैयार करने के निए,
एक नजिा योजना सनमनत (DPC) का गठन करे गा।
 आस ऄनधननयम के ऄनुसार ककसी नजिा योजना सनमनत के कु ि सदस्य संख्या के कम से कम चार बटा पांच सदस्य, नजिा स्तर
पर पंचायत के और नजिे में नगरपानिकाओं के ननवाभनचत सदस्यों ्ारा, ऄपने में से, नजिे में रामामीण क्षेत्रों की और नगरीय क्षेत्रों
की जनसंख्या के ऄनुपात के ऄनुसार ननवाभनचत ककए जाएंग।े

Q 69.C
 कथन 1 सही है चूाँकक आस सनमनत के सदस्यों का ननवाभचन प्रनत वषभ, 1 वषभ की ऄवनध के निए ककया जाता है। मूितः, आसमें 15
सदस्य (िोकसभा से 10 तथा राज्य सभा से 5) सदस्य थे। हािांकक 1974 में, आसके सदस्यों की संख्या बढ़ाकर 22 (िोकसभा से
15 तथा राज्यसभा से 7) कर दी गइ। सदस्यों का ननवाभचन, एकि संिमणीय मत के माध्यम से अनुपानतक प्रनतनननधत्व
प्रणािी ्ारा ककया जाता है।
 कथन 2 सही है। सनमनत के ऄध्यक्ष की ननयुनि, िोकसभा ऄध्यक्ष (स्पीकर) ्ारा के वि िोकसभा के सदस्यों में से की जाती है।
ककसी मंत्री को आस सनमनत के सदस्य के रूप में ननवाभनचत नहीं जा सकता।

Q 70.A
 िोकसभा ऄध्यक्ष को पदावनध की सुरक्षा प्रदान की जाती है। ईसे िोकसभा के तत्कािीन समस्त सदस्यों के बहुमत से पारितरत
संकल्प ्ारा ऄपने पद से हटाया जा सकता है। पद से हटाने से संबध
ं ी प्रस्ताव पर के वि तभी नवचार भी ककया जा सकता है जब
आसे कम से कम 50 सदस्यों का समथभन प्राप्त हो। आसनिए, कथन 1 सही नहीं है।
 स्वतंत्र या मौनिक प्रस्ताव (substantive motion) के ऄनतरितरि ईसके कायभ व अचरण की िोकसभा में चचाभ एवं अिोचना
नहीं की जा सकती है। प्रकियाओं को नवननयनमत करने या काम-काज के संचािन या व्यवस्था बनाए रखने संबंधी ईसकी
शनियां ककसी न्यायािय के ऄनधकार क्षेत्र के ऄधीन नहीं हैं। आसनिए, कथन 2 तथा 3 सही हैं।

Q 71.C
 कें द्रीय स्थानीय शासन परितरषद की स्थापना 1954 में की गयी थी। आसका गठन राष्ट्रपनत के अदेश से संनवधान के ऄनुच्छेद 263
के तहत ककया गया था। मूितः, आसे कें द्रीय स्थानीय स्व-शासन परितरषद के नाम से जाना जाता था। हािांकक, 1980 के दशक में
‘स्व-शासन’ शब्द को ऄनावश्यक मानते हुए आसे ‘शासन’ शब्द से प्रनतस्थानपत कर कदया गया। 1958 तक आसका संबध
ं नगरीय
और रामामीण स्थानीय शासन दोनों से था िेककन 1958 के पश्चात् आसका संबध
ं के वि नगरीय स्थानीय शासन तक ही सीनमत हो
गया।
 यह परितरषद एक सिाहकारी ननकाय है। आसमें भारत सरकार के अवास और शहरी मामिों का के न्द्रीय मंत्री तथा राज्यों के
स्थानीय-शासन के मंत्री सनम्मनित होते हैं। अवास और शहरी मामिों का के न्द्रीय मंत्री आस परितरषद् के ऄध्यक्ष के रूप में कायभ
करता है।

Q 72.C
 ग्यारहवी ऄनुसच
ू ी: आसमें पंचायतों के ऄंतगभत रखे गए ननम्ननिनखत 29 प्रकायाभत्मक नवषय हैं:
o कृ नष, नजसके ऄंतगभत कृ नष नवस्तार शानमि हैं
o भूनम नवकास, भूनम सुधार का कायाभन्वयन, चकबंदी और भूनम संरक्षण
o िघु लसचाइ, जि प्रबन्धन और जि नवभाजक क्षेत्र का नवकास
19 www.visionias.in ©Vision IAS
o पशु पािन, डेयरी ईद्योग और कु क्कु ट पािन
o मत्स्य ईद्योग
o सामानजक वाननकी और फामभ वाननकी
o िघु वन ईपज
o िघु ईद्योग, नजसके ऄंतगभत खाद्य प्रसंस्करण ईद्योग भी शानमि हैं
o खादी, रामामोद्योग और कु टीर ईद्योग
o रामामीण अवासन
o पेयजि
o ईंधन और चारा
o सड़कें , पुनिया, पुि, फे री, जिमागभ और ऄन्य संचार साधन
o रामामीण नवद्युतीकरण, नजसके ऄंतगभत नवद्युत् का नवतरण शानमि है
o गैर-पारम्परितरक उजाभ स्रोत
o गरीबी ईन्मूिन कायभिम
o नशक्षा नजसके ऄंतगभत प्राथनमक और माध्यनमक नवद्यािय भी शानमि हैं
o तकनीकी प्रनशक्षण और व्यावसानयक नशक्षा
o प्रौढ़ और ऄनौपचारितरक नशक्षा
o पुस्तकािय
o सांस्कृ नतक कियाकिाप
o बाजार और मेिे
o स्वास्थ्य और स्वच्छता, नजसके ऄंतगभत ऄस्पताि, प्राथनमक स्वास्थ्य के न्द्र और औषधािय भी शानमि हैं
o परितरवार कल्याण
o मनहिा और बाि नवकास
o समाज कल्याण, नजसके ऄंतगभत कदव्यांगों और माननसक रूप से मंद व्यनियों का कल्याण भी शानमि है।
o दुबभि वगों का और नवनशितया, ऄनूसूनचत जानतयों और ऄनूसूनचत जनजानतयों का कल्याण शानमि है।
o सावभजननक नवतरण प्रणािी
o सामुदानयक अनस्तयों का ऄनुरक्षण
o मनिन बस्ती सुधार और ईन्नयन संनवधान की बारहवीं ऄनुसच
ू ी के ऄंतगभत शानमि है।
o आसनिए, ईत्तर (c) है।

Q 73.B
 रामाम न्यायािय, ककसी नजिे में मध्यवती स्तर पर प्रत्येक पंचायत के निए या मध्यवती स्तर पर संबंनधत ननकटवती पंचायतों के
समूह के निए या ककसी भी राज्य में जहां मध्यवती स्तर पर कोइ पंचायत नहीं है वहां ननकटवती पंचायतों के एक समूह के निए
स्थानपत ककया जा सकता है। आसनिए, कथन 1 सही नहीं है।
 रामाम न्यायािय, मोबाआि न्यायािय होंगे और आन्हें अपरानधक और दीवानी दोनों न्यायाियों की शनियां प्राप्त होंगी। आसनिए,
कथन 2 सही है।

Q 74.C
 संसदीय बैठकों का पहिा घंटा ‘प्रनगतकाि’ के निए होता है। आस दौरान, सदस्य प्रनगत पूछते हैं और सामान्यत: मंत्री ईत्तर देते हैं।
शून्यकाि प्रनगतकाि के तुरंत बाद अरं भ होता है और दैननक कायभिम (ऄथाभत सदन का ननयनमत कायभ) समाप्त होने तक जारी
रहता है। दूसरे शब्दों में, प्रनगतकाि और कायभिम तय करने के बीच का समय शून्यकाि के रूप में जाना जाता है।

20 www.visionias.in ©Vision IAS


 प्रनगतकाि के नवपरीत, प्रकिया नवषयक ननयमों में शून्यकाि का ईल्िेख नहीं है। आसनिए, यह संसद सदस्यों के निए ईपिब्ध एक
ऄनौपचारितरक साधन है नजसमें सदस्य नबना ककसी पूवभ सूचना के मामिे को ईठा सकते हैं। आसनिए, कथन 1 सही है और कथन
3 सही नहीं है।
 संसदीय प्रकियाओं के क्षेत्र में यह नवाचार भारतीय देन है और 1962 से ऄनस्तत्व में है। आसनिए, कथन 2 सही नहीं है।
Q 75.C
 परितरसीमन ऄनधननयम, 2002 को 2001 की जनगणना के अधार पर परितरसीमन को प्रभावी बनाने के ईद्देश्य से एक
परितरसीमन अयोग की स्थापना के निए ऄनधननयनमत ककया गया था ताकक ननवाभचन क्षेत्रों के अकार में नवद्यमान नवरूपण
में सुधार ककया जा सके । परितरसीमन अयोग ने 1971 की जनगणना के अधार पर ननधाभरितरत सीटों की कु ि संख्या को
प्रभानवत ककये नबना 2001 की जनगणना के अधार पर ऄनूसूनचत जानतयों और ऄनूसूनचत जनजानतयों के निए सीटों की
संख्या को पुनः ननधाभरितरत कर कदया है। सीटों के अबंटन के संबंध में संवैधाननक और कानूनी प्रावधान आस प्रकार हैं:
o 1971 की जनगणना के अधार पर िोकसभा में नवनभन्न राज्यों को अवंरितटत वतभमान सीटों की कु ि संख्या वषभ 2026 के
पश्चात् पहिी जनगणना तक ऄपरितरवर्मतत रहेगी।
o 1971 की जनगणना के अधार पर ननधाभरितरत की गयी सभी नवधान सभाओं में वतभमान सीटों की कु ि संख्या वषभ 2026 के
पश्चात् पहिी जनगणना तक ऄपरितरवर्मतत रहेगी।
o िोकसभा और राज्य नवधानसभाओं में ऄनूसनू चत जानतयों (SCs) और ऄनूसनू चत जनजानतयों (STs) के निए अरनक्षत
सीटों की संख्या को 2001 की जनगणना के अधार पर पुन:ननधाभरितरत ककया गया है।

Q 76.A
 कथन 1 सही है: संनवधान में राज्यों में नवधान परितरषदों का ईत्सादन या सृजन का प्रावधान ककया गया है। तदनुसार, संसद एक
नवधान परितरषद को समाप्त (जहााँ यह पहिे से ऄनस्तत्व में है) या सृजन (जहााँ आसका ऄनस्तत्व नहीं है) कर सकती है, यकद
संबंनधत राज्य की नवधान सभा ्ारा आस अशय का प्रस्ताव पारितरत ककया जाता है। आस प्रकार के प्रस्ताव को राज्य की नवधान
सभा ्ारा नवशेष बहुमत से पारितरत ककया जाना चानहए, ऄथाभत् नवधान सभा के कु ि सदस्यों का बहुमत तथा ईपनस्थत और मत

देने वािे सदस्यों के कम-से-कम दो-नतहाइ बहुमत ्ारा पारितरत होना चानहए। संसद के आस कायभ को ऄनुच्छेद 368 के प्रयोजनों
हेतु संनवधान में संशोधन के रूप में नहीं समझा जाता है और आसे एक साधारण ऄनधननयम (ऄथाभत्, साधारण बहुमत से) के
समान ही पारितरत ककया जाता है।
 कथन 2 सही नहीं है: 1962 में ऄनुच्छेद 239A को कु छ कें द्र शानसत प्रदेशों में नवधानमंडिों या मंनत्रपरितरषदों या दोनों के सृजन
के निए संसद को ऄनधकार प्रदान करने हेतु समानवि ककया गया था।
 कथन 3 सही नहीं है: यकद नवधानसभा में अंग्ि-भारतीय समुदाय को पयाभप्त प्रनतनननधत्व प्राप्त नहीं है, तो राज्यपाि अंग्ि-
भारतीय समुदाय के एक सदस्य को मनोनीत कर सकता है।

Q 77.A
 ‘संसदीय संप्रभुता’ का नसिांत निरितटश संसद से संबंनधत है। संप्रभुता का ऄथभ है राज्य में सवो्च शनि का नननहत होना। रामेट
निटेन में सवो्च शनि संसद में नननहत है। दूसरी ओर भारतीय संसद को आसके समान एक संप्रभु ननकाय नहीं माना जा सकता
है, क्योंकक आसके प्रानधकार और ऄनधकार क्षेत्र पर ‘कानूनी’ प्रनतबन्ध नवद्यमान हैं।
 भारतीय ससंदीय संप्रभुता को कइ कारक सीनमत करते हैं:
o निनखत संनवधान: संनवधान, हमारे देश की सीमा के भीतर मौनिक कानून है। आसने कें द्र सरकार के सभी तीनों ऄंगों के
प्रानधकार और ऄनधकार क्षेत्र तथा ईनके मध्य परस्पर संबंधों की प्रकृ नत को परितरभानषत ककया है। आसनिए, संसद को
संनवधान ्ारा ननधाभरितरत सीमाओं के भीतर ही कायभ करना होता है। संसद के नवधायी प्रानधकार और संवैधाननक प्रानधकार
के मध्य कानूनी नवभेद भी है। आसके ऄनतरितरि, संनवधान में कु छ संशोधनों को प्रभावी बनाने के निए कम से कम अधे
राज्यों का ऄनुसमथभन भी अवश्यक होता है। आसनिए, कथन 1 सही है।

21 www.visionias.in ©Vision IAS


o मूि ऄनधकार: संनवधान के भाग 3 के ऄंतगभत न्यायोनचत मूि ऄनधकारों को शानमि करने से भी संसद के प्रानधकार सीनमत
हुए हैं। ऄनुच्छेद 13 राज्यों को ऐसी नवनध ननर्ममत करने से रोकता है जो प्रदत्त मूि ऄनधकारों को छीनती है या न्यून करती
है। आसनिए, संसदीय नवनध जो मूि ऄनधकारों का ईल्िंघन करती है, शून्य होगी। आसनिए, कथन 2 सही है।
o सरकार की संघीय व्यवस्था: भारत में कें द्र और राज्यों के मध्य शनियों के संवैधाननक नवभाजन सनहत संघीय सरकार
नवद्यमान है। दोनों को ईनके निए ननधाभरितरत क्षेत्रों के भीतर कायभ करना होता है। आसनिए, संसद का नवनध ननमाभण संबंधी
प्रानधकार संघ सूची और समवती सूची में ईनल्िनखत नवषयों तक ही सीनमत हो जाता है और राज्य सूची में ईनल्िनखत
नवषयों तक नवस्तारितरत नहीं होता है (नवशेष परितरनस्थनतयों और कु छ समयावनध के निए ननर्ममत नवनध के ऄनतरितरि)।
o न्यानयक समीक्षा संबधं ी व्यवस्था: न्यानयक समीक्षा की शनि सनहत एक स्वतंत्र न्यायपानिका को ऄपनाना भी हमारी
संसद की सवो्च ता को सीनमत करता है। ई्च तम न्यायािय और ई्च न्यायािय दोनों ही संसद ्ारा ऄनधननयनमत ककसी
नवनध को शून्य एवं ऄसंगत (ऄसंवैधाननक) घोनषत कर सकते हैं, यकद वह नवनध संनवधान के ककसी प्रावधान का ईल्िंघन
करती हों या ईसके मूि ढांचे में संशोधन करती हों।
 राष्ट्रपनत स्वयं संसद का भाग है; आसनिए ईसकी वीटो शनियााँ संसदीय संप्रभुता को सीनमत नहीं करती है। आसनिए, कथन 3
सही नहीं है। आसनिए, नवकल्प (a) सही है।

Q 78.A
 सांनवनधक ऄनुदान: ऄनुच्छेद 275 संसद को ईन राज्यों को ऄनुदान प्रदान करने की शनि प्रदान करता है, नजन्हें नवत्तीय
सहायता की अवश्यकता होती है, न कक प्रत्येक राज्य को। आसके ऄनतरितरि, नवनभन्न राज्यों के निए ऄिग-ऄिग रानश ननधाभरितरत
की जा सकती है। आस रानश को प्रत्येक वषभ भारत की संनचत नननध पर भारितरत ककया जाता है। आसनिए, कथन 1 सही है।
 नववेकाधीन ऄनुदान: ऄनुच्छेद 282 कें द्र और राज्य दोनों को ककसी भी िोक प्रयोजन के निए कोइ भी ऄनुदान प्रदान करने की
शनि प्रदान करता है, भिे ही यह ईनके संबंनधत नवधायी सामथ्यभ में न हो। आसनिए, कथन 2 सही नहीं है।

Q 79.B
 जनवरी 1957 में, भारत सरकार ने सामुदानयक नवकास कायभिम (1952) और राष्ट्रीय नवस्तार सेवा (1953) ्ारा ककए गए
कायों की जााँच करने एवं ईनके बेहतर ढंग से कायभ करने के निए ईपायों का सुझाव देने हेतु एक सनमनत का गठन ककया था। आस
सनमनत के ऄध्यक्ष बिवंत राय मेहता थे। सनमनत ने नवम्बर, 1957 में ऄपनी रितरपोटभ प्रस्तुत की और ‘िोकतांनत्रक नवकें द्रीकरण’
की योजना की संस्तुनत की, नजसे ऄंतत: पंचायती राज के नाम से जाना गया। आसने नत्र-स्तरीय पंचायती राज व्यवस्था (रामाम
स्तर पर रामाम पंचायत, ब्िाक स्तर पर पंचायत सनमनत और नजिा स्तर पर नजिा परितरषद) की स्थापना की संस्तुनत की थी।
 पंचायती राज की स्थापना करने वािा राजस्थान पहिा राज्य था। आस योजना का ईद्घाटन प्रधानमन्त्री ्ारा 2 ऄिू बर, 1959
को राजस्थान के नागौर नजिे में ककया गया था। राजस्थान का ऄनुसरण अंध्रप्रदेश ने ककया और आस व्यवस्था को 1959 में िागू
ककया। आसके पश्चात ऄनधकांश राज्यों ने आस व्यवस्था को ऄपना निया।
 73वां संनवधान संशोधन ऄनधननयम, 1992 जो 24 ऄप्रैि, 1993 से प्रभावी हुअ और आसने पंचायती राज संस्थानों को
संवध
ै ाननक दजाभ प्रदान ककया। यह आन्हें संनवधान के न्यायोनचत भाग के दायरे में िे अया। दूसरे शब्दों में, ऄनधननयम के
प्रावधानों के ऄनुसार पंचायती राज व्यवस्था को ऄपनाना राज्य सरकारों का संवैधाननक दानयत्व है। परितरणामस्वरूप, ऄब
पंचायतों का गठन और ननयनमत ऄंतराि पर चुनाव अयोनजत करना राज्य सरकार की आच्छा पर ननभभर नहीं करता है।
 आसनिए, के वि कथन 2 ही सही है।

Q 80.A
 क्षेत्रीय परितरषदें सांनवनधक ननकाय हैं (न कक संवैधाननक)। आनका गठन संसद के ऄनधननयम ऄथाभत् राज्य पुनगभठन ऄनधननयम,
1956 ्ारा ककया गया है। आस ऄनधननयम ने देश को पांच क्षेत्रों (ईत्तरी, मध्य, पूवी, पनश्चमी और दनक्षणी) में नवभानजत ककया
और प्रत्येक क्षेत्र के निए एक क्षेत्रीय परितरषद का गठन ककया। आसनिए, कथन 1 सही है।
 कें द्र सरकार का गृह मंत्री पांचों परितरषदों का ऄध्यक्ष होता है। प्रत्येक मुख्यमंत्री िमानुसार एक वषभ की ऄवनध के निए परितरषद के
ईपाध्यक्ष के रूप में कायभ करता है। आसनिए, कथन 2 सही नहीं है।
22 www.visionias.in ©Vision IAS
Q 81.D
 संनवधान ्ारा कें द्र और राज्यों के मध्य कराधान शनियों को ननम्ननिनखत तरीकों से नवभानजत ककया गया है:
o संसद को संघ सूची में ईनल्िनखत सभी नवषयों पर कर िगाने की ऄनन्य शनि प्राप्त है। संसद और राज्य नवधानमंडि
समवती सूची में ईनल्िनखत नवषयों पर कर िगा सकते हैं। आसनिए, कथन 1 सही नहीं है।
o राज्य नवधानमंडि को राज्य सूची में ईनल्िनखत सभी नवषयों पर कर िगाने की ऄनन्य शनि प्राप्त है।
o कर िगाने की ऄवनशि शनि (ऄथाभत तीनों सूनचयों में से ककसी में भी ईनल्िनखत न ककए गए नवषय पर कराधान की
शनि) संसद में नननहत है। आस प्रावधान के ऄंतगभत संसद ने ईपहार कर, सम्पनत्त कर और व्यय कर िगाए हैं। आसनिए कथन
2 सही नहीं है।

Q 82.D
 ऄनुच्छेद 343 – संघ की राजभाषा - (1) संघ की राजभाषा लहदी और निनप देवनागरी होगी। संघ के शासकीय प्रयोजनों के
निए प्रयोग होने वािे ऄंकों का रूप भारतीय ऄंकों का ऄंतराभष्ट्रीय रूप होगा।
 खंड (1) में ककसी बात के होते हुए भी, आस संनवधान के प्रारं भ से पन्द्रह वषभ की ऄवनध तक संघ के ईन सभी शासकीय प्रयोजनों
के निए ऄंरामजे ी भाषा का प्रयोग ककया जाता रहेगा नजनके निए ईसका ऐसे प्रारं भ से ठीक पहिे प्रयोग ककया जा रहा था: परं तु
राष्ट्रपनत ईि ऄवनध के दौरान अदेश ्ारा संघ के शासकीय प्रयोजनों में से ककसी के निए ऄंरामज े ी भाषा के ऄनतरितरि लहदी भाषा
का और भारतीय ऄंकों के ऄंतराभष्ट्रीय रूप के ऄनतरितरि देवनागरी रूप का प्रयोग प्रानधकृ त कर सके गा।
 आस ऄनुच्छेद में ककसी बात के होते हुए भी, संसद ईि पन्द्रह वषभ की ऄवनध के पश्चात नवनध ्ारा- (a) ऄंरामज
े ी भाषा का, या
(b) ऄंकों के देवनागरी रूप का, ईस नवनध में नवननर्ददि प्रयोजनों के निए प्रयोग का प्रावधान कर सके गी।
 ऄनुच्छेद 346- एक राज्य और दूसरे राज्य के बीच या ककसी राज्य और संघ के बीच पत्राकद की राजभाषा – संघ में शासकीय
प्रयोजनों के निए प्रयोग ककए जाने के निए तत्समय प्रानधकृ त भाषा, एक राज्य से दूसरे राज्य के बीच तथा ककसी राज्य और संघ
के बीच पत्राकद की भाषा होगी: परं तु यकद दो या ऄनधक राज्य यह करार करते हैं कक ईन राज्यों के बीच पत्राकद की राजभाषा
लहदी भाषा होगी तो ऐसे पत्राकद के निए ईस भाषा का प्रयोग ककया जा सके गा।

Q 83.C
 ऄनभिेख न्यायािय के रूप में, ई्च तम न्यायािय के पास दो शनियााँ हैं:
o ई्च तम न्यायािय की कायभवाही एवं ईसके ननणभय सावभकानिक ऄनभिेख व साक्ष्य के रूप में रखे जाएंग।े आन ऄनभिेखों को
साक्ष्य के रूप में माना जाता है और ककसी ऄन्य न्यायािय में चि रहे मामिे के दौरान ईन पर प्रनगत नहीं ईठाया जा सकता
है। ईन्हें नवनधक दृिान्त और सन्दभभ के रूप में स्वीकार ककया जाएगा।
o ई्च तम न्यायािय को न्यायािय की ऄवमानना के निए दण्ड देने की शनि प्राप्त है, आसमें छह माह के निए सामान्य
कारावास या 2000 रुपए तक ऄथभदड
ं ऄथवा दोनों शानमि हैं। 1991 में ई्च तम न्यायािय ने ननणभय कदया है कक दंड देने
की यह शनि न के वि ई्च तम न्यायािय में नननहत है बनल्क ई्च न्यायाियों, ऄधीनस्थ न्यायाियों और पूरे देश में कायभरत
न्यायानधकरणों को भी ऄवमानना के निए दण्ड देने की शनि है।

Q 84.D
 संनवधान के ऄनुसार भारत की संसद के तीन ऄंग हैं जैसे राष्ट्रपनत, कौंनसि ऑफ स्टेट्स, और हाईस ऑफ द पीपि। 1954 में
कौंनसि ऑफ स्टेट्स और हाईस ऑफ द पीपि के लहदी नामों के रूप में ‘राज्य सभा’ और ‘िोक सभा’ को ऄपनाया गया था।
आसनिए, कथन 1 सही नहीं है।
 कौंनसि ऑफ स्टेट्स या राज्य सभा का सभापनत ऄथाभत् ईपराष्ट्रपनत, स्पीकर या िोकसभा ऄध्यक्ष (जो सदन का सदस्य होता
है) के नवपरीत सदन का सदस्य नहीं होता है। आसनिए, कथन 2 सही नहीं है।

Q 85.D
 सावभजननक ईपयोनगता सेवाओं से संबंनधत वादों का ननपटान करने हेतु स्थायी िोक ऄदाितों की स्थापना के निए नवनधक सेवा
प्रानधकरण ऄनधननयम, 1987 का 2002 में संशोधन ककया गया था। आसनिए, कथन 1 सही है।

23 www.visionias.in ©Vision IAS


 आस ऄनधननयम के तहत गरितठत िोक ऄदाितों के संगठन की वतभमान योजना में एक प्रमुख कमी यह है कक िोक ऄदाितों की
व्यवस्था मुख्य रूप से पक्षों के मध्य समझौते या ननपटान पर अधारितरत है। यकद पक्ष ककसी भी समझौते या समाधान पर सहमत
नही हो पाते हैं तो मामिा या तो न्यायािय में िौटाया जाता है या पक्षों को न्यायािय में समाधान प्राप्त करने का परामशभ
कदया जाता है। आसनिए, कथन 2 सही है।
 िोक ऄदाितों के ऄंतगभत सावभजननक ईपयोनगताओं से संबंनधत छोटे मामिों (नजन्हें ननयनमत ऄदाितों में नहीं जाना चानहए)
का मुकदमेबाजी के प्राथनमक चरणों में समाधान ककया जायेगा; जैसे महानगर टेिीफ़ोन ननगम निनमटेड, कदल्िी नवद्यािय बोडभ
अकद से सम्बंनधत मामिे। आस प्रकार, आसके परितरणामस्वरूप ननयनमत न्यायाियों के कायभभार में काफी हद तक कमी अएगी।
आसनिए, कथन 3 सही है।

Q 86.D
 भारत का राष्ट्रपनत, नवनधवत योग्य व्यनि को ई्च न्यायािय के कायभकारी न्यायधीश के रूप में ननयुि कर सकता है, जब ईस
ई्च न्यायािय का एक न्यायाधीश (मुख्य न्यायाधीश के ऄनतरितरि):
o ऄनुपनस्थनत या ककसी ऄन्य कारण से ऄपने पद के कतभव्यों को पूरा करने में ऄसमथभ हो या
o ऄस्थायी रूप से संबनं धत ई्च न्यायािय के मुख्य न्यायधीश के रूप में ननयुि ककया गया हो।
 एक कायभकारी न्यायाधीश पद पर बना रहता है, जब तक स्थायी न्यायधीश ऄपना पदभार न संभाि िे।

Q 87.D
 सूचना के ऄनधकार ऄनधननयम की धारा 6(2) में नवशेष रूप से कहा गया है कक “सूचना के निए ऄनुरोध करने वािे अवेदक से
सूचना का ऄनुरोध करने के निए ककसी कारण की या ककसी ऄन्य व्यनिगत नववरण को, नसवाय ईसके जो ईससे संपकभ करने के
निए ऄवश्यक हो, देने की ऄपेक्षा नही की जाएगी।” यह नहतकारी प्रावधान यह सुनननश्चत करने के निए महत्त्वपूणभ है कक
ऄनुरोध को ककसी व्यनि-ननष्ठ अकिन एवं संशय के अधार पर ऄस्वीकार न ककया जा सके । आसनिए, कथन 1 सही नहीं है।
 गैर-सरकारी ननकाय को ऄनधननयम के ऄंतगभत सावभजननक प्रानधकरण के रूप में वगीकृ त करने के निए ईसका सरकार ्ारा
पयाभप्त रूप से नवत्तपोनषत होना अवश्यक है। हािााँकक “पयाभप्त रूप से नवत्तपोनषत” की कोइ परितरभाषा नहीं दी गइ है। आसनिए
कथन 2 सही नहीं है।

Q 88.C
 िोकसभा और राज्य नवधान सभाओं के सदस्यों को िोगों ्ारा प्रत्यक्ष रूप से ननवाभनचत ककया जाता है। चुनाव के ईद्देश्य के
निए पूरे देश (राज्य, राज्य नवधानसभा के मामिे में) को िगभग समान जनसंख्या के क्षेत्रीय ननवाभचन क्षेत्रों में नवभानजत ककया
गया है। एक प्रनतनननध सावभभौनमक वयस्क मतानधकार के माध्यम से एक ननवाभचन क्षेत्र से ननवाभनचत होता है जहााँ प्रत्येक व्यनि
के मत का मूल्य बराबर होता है। वतभमान में 543 ननवाभचन क्षेत्र हैं। 1971 की जनगणना से आस संख्या में परितरवतभन नहीं हुअ है।

Q 89.D
 बहु-दिीय प्रणािी: देश का महा्ीपीय अकार, भारतीय समाज का नवनवध चरितरत्र, सावभभौनमक वयस्क मतानधकार, नवनशि
राजनीनतक प्रकिया और ऄन्य कारकों ने बड़ी संख्या में राजनीनतक दिों के ईदय का मागभ प्रशस्त ककया है। भारत में ऄनत
नवनवध बहु-दिीय राजनीनतक प्रणािी नवद्यमान है। 13 ऄप्रैि, 2018 तक भारत में तीन प्रकार के राजनीनतक दि नवद्यमान थे
ऄथाभत् सात राष्ट्रीय दि, 50 से ऄनधक राज्य मान्यता प्राप्त दि और 2044 पंजीकृ त गैर-मान्यता प्राप्त दि।
 प्रनतनष्ठत नेताओं के नेतत्ृ व में दिों की स्थापना: प्रायः दिों को प्रनतनष्ठत नेताओं के नेतृत्व में संगरितठत ककया जाता है जो दि और
ईसकी नवचारधारा से ऄनधक महत्वपूणभ हो जाते हैं। दिों को ईनके मैननफे स्टो (घोषणा पत्र) की बजाय ईनके नेताओं के कारण
जाना जाता है। ईदाहरण के निए AIADMK।
 क्षेत्रीय दिों का ईदय: AIADMK, DMK, SAD आत्याकद जैसे कइ क्षेत्रीय दि का ईदय हुअ है और ये चुनावी राजनीनत में
महत्वपूणभ भूनमका ननभाते हैं।
 ऄतः, सभी कथन सही हैं।

24 www.visionias.in ©Vision IAS


Q 90.A
 संसद में धन नवधेयक पारितरत करने के निए संनवधान एक नवशेष प्रकिया प्रस्तुत करता है। एक धन नवधेयक राष्ट्रपनत की ऄनुशस
ं ा
पर के वि िोकसभा में ही प्रस्तुत ककया जा सकता है। ऐसे प्रत्येक नवधेयक को सरकारी नवधेयक माना जाता है और आसे के वि
एक मंत्री ्ारा ही पेश ककया जा सकता है। यकद ऐसा कोइ प्रनगत ईठता है कक नवधेयक एक धन नवधेयक है ऄथवा नहीं, तो आस
पर िोकसभा के ऄध्यक्ष का ननणभय ऄंनतम होता है।
 आस संबध
ं में ईसके ननणभय पर ककसी भी न्यायािय में ऄथवा संसद में या कफर यहााँ तक कक राष्ट्रपनत ्ारा भी चुनौती नहीं दी
जा सकती है। जब कोइ धन नवधेयक राज्यसभा में ऄनुमोदन हेतु प्रस्तुत ककया जाता है और स्वीकृ नत हेतु राष्ट्रपनत के समक्ष
प्रस्तुत ककया जाता है, तो िोकसभा ऄध्यक्ष ्ारा आसे धन नवधेयक के रूप में ऄनुमत ककया जाता है।
 संनवधान का ऄनुच्छेद 110 धन नवधेयकों की परितरभाषा से संबंनधत है। यह व्याख्या करता है कक कोइ नवधेयक धन नवधेयक
समझा जाएगा, यकद ईसमें के वि ननम्ननिनखत सभी या ककन्हीं नवषयों से संबंनधत ईपबंध हैं, ऄथाभत-्
o ककसी कर का ऄनधरोपण, ईत्सादन, परितरहार, परितरवतभन या नवननयमन;
o संघीय सरकार ्ारा धन ईधार िेने का नवननयमन;
o भारत की संनचत नननध या अकनस्मकता नननध की ऄनभरक्षा, ऐसी ककसी नवनध में धन जमा करना या ईसमें से धन
ननकािना;
o भारत की संनचत नननध में से धन का नवननयोग
o ककसी व्यय को भारत की संनचत नननध पर भारितरत व्यय घोनषत करना या आस प्रकार के ककसी व्यय की रानश में वृनि;
o भारत की संनचत नननध या भारत की िोक िेखा नननध से धन प्राप्त करना ऄथवा ऐसे धन की ऄनभरक्षा या ईसका ननगभमन
ऄथवा संघ या राज्य के िेखाओं की संपरीक्षा; या
o ईपयुभि ककसी भी मामिे से संबंनधत कोइ अकनस्मक प्रकरण।

Q 91.B
 कें द्र की संपनत्त, राज्य ्ारा या राज्य के भीतर ककसी भी प्रानधकरण जैसे कक नगरपानिका, नजिा परितरषद्, पंचायतें तथा ऄन्य,
्ारा अरोनपत सभी करों से मुि होती है। िेककन, संसद को यह प्रनतबंध हटाने का ऄनधकार है। कें द्र सरकार ्ारा गरितठत ननगम
या कं पननयां राज्य कराधान या स्थानीय कराधान से मुि नहीं हैं। आसका कारण यह है कक एक ननगम या कं पनी पृथक कानूनी
आकाइ होती है। ऄतः, कथन 1 सही नहीं है।
 राज्य की संपनत्त और अय के न्द्रीय कराधान से मुि होती है। ऐसी अय स्वतंत्र कायों या वानणनज्यक कायों से व्युत्पन्न की जा
सकती है। िेककन यकद संसद प्रावधान करती है तो कें द्र सरकार राज्य के वानणनज्यक संचािनों पर कर िगा सकती है। हािांकक,
यकद संसद ककसी नवशेष व्यापार या व्यवसाय को सरकार के सामान्य कायों के निए अनुषांनगक घोनषत कर सकती है और तब
वह कर योग्य नहीं रहेगा। ऄतः, कथन 2 सही है।

Q 92.C
 राज्यपाि, राज्य का प्रमुख होता है जबकक मुख्यमंत्री सरकार का। राज्यपाि राज्य का कायभकारी प्रमुख होता है। िेककन,
राष्ट्रपनत की भांनत, वह भी नाममात्र का कायभकारी प्रमुख (नाममात्र का या संवैधाननक प्रमुख) होता है।
 कथन 1 सही है। राज्यपाि का पद भारत सरकार ऄनधननयम,1935 से रामहण ककया गया है जबकक कें द्र ्ारा राज्यपाि को
ननयुि करने की पिनत कनाडा के संनवधान से िी गयी है।
 कथन 2 सही है। प्रायः, प्रत्येक राज्य में एक राज्यपाि होता है, िेककन 7वें संनवधान संशोधन ऄनधननयम,1956 के ऄनुसार दो
या दो से ऄनधक राज्यों के राज्यपाि के रूप में एक ही व्यनि को ननयुि ककया जा सकता है।

Q 93. B
 कथन 1 सही नहीं है। मुख्यमंत्री का कायभकाि नननश्चत नहीं है और वह राज्यपाि के प्रसादपयंत पद धारण करता है। हािांकक,
आसका ऄथभ यह नहीं है कक राज्यपाि ईसे ककसी भी समय पदच्युत कर सकता है। जब तक ईसे नवधानसभा में बहुमत प्राप्त है,

25 www.visionias.in ©Vision IAS


तब तक ईसे राज्यपाि ्ारा पदच्युत नहीं ककया जा सकता है। यकद वह नवधानसभा में बहुमत खो देता है, तो ईसे या तो
आस्तीफा देना होगा या कफर राज्यपाि भी ईसे पदच्युत कर सकता है।
 कथन 2 सही है। वह ककसी भी समय राज्यपाि से नवधान सभा को भंग करने की ऄनुशंसा कर सकता है।

Q 94C
 न्यानयक सकियता ने राजनैनतक व्यवस्था पर नवनवध प्रभाव डािा है।
िाभ-
o आसने न के वि व्यनियों बनल्क समूहों को न्यायाियों तक पहुाँच प्रदान कर न्यानयक प्रणािी को िोकतांनत्रक बनाया है।
o आसने कायभपानिका पर ईत्तरदानयत्व का दबाव बनाया है।
o आसने चुनाव प्रणािी को ऄनधक स्वतंत्र और ननष्पक्ष बनाने का प्रयास ककया है।
हानन -
o आसने ऄत्यनधक संख्या में PIL (जन नहत यानचका) दायर ककये जाने के कारण न्यायाियों के कायभभार में वृनि की है।
o आसने एक ओर कायभपानिका एवं नवधानयका के मध्य तथा दूसरी ओर कायभपानिका एवं न्यायपानिका के बीच के ऄंतर को
कम ककया है। न्यायािय, कायभपानिका से संबंनधत मुद्दों को हि करने में संिग्न है। कु छ िोगों का मानना है कक न्यानयक

सकियता ने सरकार के तीन ऄंगों में संति


ु न को ऄत्यनधक कमजोर बना कदया है। िोकतानन्त्रक सरकार, सरकार के प्रत्येक
ऄंग ्ारा दूसरे ऄंग की शनियों एवं ऄनधकारितरता के सम्मान के नसिांत पर अधारितरत होती है। न्यानयक सकियता ईपयुि

िोकतांनत्रत नसिांत पर दवाब ईत्पन्न कर सकती है। ऄतः, नवकल्प (c) सही नहीं है।

Q 95.C
 संसद का प्राथनमक कायभ देश में शासन के निए कानून का ननमाभण करना है। आसे संघ सूची में वर्मणत सभी नवषयों एवं ऄवनशि
नवषयों (ऄथाभत् ऐसे नवषय जो तीन में से ककसी भी सूची में वर्मणत नहीं ककए गए हैं) पर कानून ननमाभण का ऄनन्य ऄनधकार प्राप्त
है। समवती सूची के संबंध में संसद को ऄनधभावी शनियााँ (overriding powers) प्राप्त हैं, ऄथाभत् दोनों के मध्य मतभेद की
नस्थनत में संसदीय कानून राज्य नवधानमंडि के कानून पर ऄनधभावी होता है।
 संनवधान भी संसद को ननम्ननिनखत पांच ऄसामान्य परितरनस्थनतयों में राज्य सूची में वर्मणत नवषयों पर कानून बनाने का
ऄनधकार प्रदान करता है:
o जब राज्यसभा में आस अशय से प्रस्ताव पारितरत करती है।
o जब राष्ट्रीय अपात की ईदघोषणा िागू हो।
o जब दो या दो से ऄनधक राज्य संसद से संयि
ु ऄनुरोध करते हैं।
o जब ऄंतरराष्ट्रीय ऄनुबंधों, संनधयों और कन्वेंशनों को प्रभावी करने के निए यह अवश्यक हों।
o जब राज्य में राष्ट्रपनत शासन िागू हो।

Q 96.D
 ऄनुच्छेद 263 राज्यों के मध्य तथा कें द्र और राज्यों के मध्य समन्वय स्थानपत करने हेतु ऄंतराभज्यीय परितरषद की स्थापना का
प्रावधान करता है। आस प्रकार यकद राष्ट्रपनत को यह प्रतीत होता है कक आसकी स्थापना से जननहत संभव होगा तो वह ककसी भी
समय आस प्रकार की परितरषद की स्थापना कर सकता है। वह आस प्रकार की परितरषद ्ारा ककए जाने वािे कत्तभव्यों की प्रकृ नत को
ईसके संगठन और प्रकिया को ननधाभरितरत कर सकता है। ऄतः, कथन 1 और 2 सही नहीं हैं।
 यह परितरषद ऄंतराभज्यीय नववादों की जांच करने और परामशभ देने का कायभ करती है और ऄनुच्छेद 131 के ऄंतगभत सरकारों के
मध्य नवनधक नववाद के ननणभयन हेतु ई्च तम न्यायािय के क्षेत्रानधकार की ऄनुपूरक होती है। यह परितरषद ककसी भी प्रकार के
नवनधक या गैर-नवनधक नववाद का ननपटारा कर सकती है, परन्तु न्यायािय ्ारा कदए जाने वािे बाध्यकारी ननणभय के नवपरीत
आसका कायभ के वि परामशभदायी होता है। आसनिए, कथन 3 सही नहीं है।

26 www.visionias.in ©Vision IAS


Q 97.B
 कथन 1 सही नहीं है: भारत की संनचत नननध ऐसी नननध है नजसमें सभी प्रानप्तयों को जमा (िे नडट) ककया जाता है और सभी
भुगतानों का व्यय (डेनबट) ककया जाता है। दूसरे शब्दों में, (a) भारत सरकार ्ारा प्राप्त सभी राजस्व; (b) रेजरी नबि, ईधार
्ारा या ऄथोपाय ऄनराममों के माध्यम से सरकार ्ारा जुटाए गए सभी प्रकार के ऊण; और (c) ऊणों की पुनःऄदायगी में
सरकार ्ारा प्राप्त धनरानश भारत की संनचत नननध का ननमाभण करती है।
 कथन 2 सही है: भारत सरकार ्ारा या ईसकी ओर से प्राप्त ऄन्य सभी सावभजननक धनरानश (भारत की संनचत नननध में िे नडट
की जाने वािी धनरानश के ऄनतरितरि) को भारत के िोक िेखे में िे नडट ककया जाएगा। आसमें भनवष्य नननध जमारानशयााँ ,
न्यानयक जमा, बचत बैंक जमा, नवभागीय जमा, नवप्रेषण आत्याकद सनम्मनित होते हैं। आस खाते का संचािन कायभकारी प्रकिया
्ारा ककया जाता है, ऄथाभत,् आस खाते से ककए जाने वािे भुगतान संसदीय नवननयोजन के नबना ककए जा सकते हैं। आस प्रकार के
भुगतान ऄनधकतर बैंककग िेन-देन की प्रकृ नत के होते हैं।

Q 98.C
 राष्ट्रपनत एवं ईपराष्ट्रपनत के ननवाभचन के संबध
ं में ईत्पन्न नववाद पर ई्च तम न्यायािय ्ारा ननणभय ककया जाता है। आस संबध
ं में
आसे मूि, ऄनन्य एवं ऄंनतम प्रानधकार प्राप्त है।
 ई्च न्यायािय के मूि न्यायानधकार क्षेत्र के ऄंतगभत, आसे नववादों की प्राथनमक सुनवाइ करनी पड़ती है, न कक ऄपीि के माध्यम
से। संसद एवं राज्य नवधानमंडिों के सदस्यों के ननवाभचन से संबनं धत नववाद ई्च न्यायािय की मूि ऄनधकारितरता के ऄंतगभत अते
हैं।
 आसनिए, नवकल्प (c) सही है।

Q 99.B
 नवधान परितरषद की संवैधाननक नस्थनत (नवधानसभा की तुिना में) का ऄध्ययन दो दृनिकोणों से ककया जा सकता है: A. ऐसे
मामिे नजनमें नवधान परितरषद की शनियााँ नवधानसभा के समान होती हैं। B. ऐसे मामिे नजनमें नवधान परितरषद की शनियााँ
नवधानसभा के समान नहीं होती हैं।
 ननम्ननिनखत मामिों में परितरषद की शनियााँ और नस्थनत व्यापक रूप से नवधानमंडि के समान होती हैं :
o साधारण नवधेयकों को पुर:स्थानपत और पारितरत करना। यद्यनप, दोनों सदनों के मध्य ऄसहमनत की नस्थनत में, नवधानसभा
का मत प्रभावी होता है।
o राज्यपाि ्ारा जारी ऄध्यादेशों को स्वीकृ नत।
o मुख्यमंत्री सनहत मंनत्रयों का चयन। संनवधान के ऄंतगभत, मुख्यमंत्री सनहत ऄन्य सभी मंनत्रयों को राज्य नवधानमंडि के
ककसी एक सदन का सदस्य होना चानहए। तथानप ऄपनी सदस्यता के बावजूद वे के वि नवधानसभा के प्रनत ईत्तरदायी होते
हैं।
o संवैधाननक ननकायों जैसे राज्य नवत्त अयोग, राज्य िोक सेवा अयोग और भारत के ननयंत्रक एवं महािेखा परीक्षक की
रितरपोटों पर नवचार करना।
o राज्य िोक सेवा अयोग के क्षेत्रानधकार का नवस्तार करना।
 ननम्ननिनखत मामिों में परितरषद की शनियां और नस्थनत नवधानसभा के समान नहीं होती हैं:
o धन नवधेयक के वि नवधानसभा में पुर:स्थानपत ककया जा सकता है, परितरषद में नहीं। परितरषद धन नवधेयक को संशोनधत या
ननरस्त नहीं कर सकती। आसे नवधेयक को ऄपनी ऄनुशस
ं ाओं के साथ या ऄनुशंसाओं के नबना 14 कदन के भीतर नवधानसभा
को वापस कर देना चानहए। नवधानसभा, परितरषद की सभी ऄनुशस
ं ाओं या ककसी ऄनुशस
ं ा को या तो स्वीकार या ऄस्वीकार
कर सकती है। दोनों ही मामिों में, धन नवधेयक को दोनों सदनों ्ारा पारितरत माना जाता है। कोइ नवधेयक धन नवधेयक है
या नहीं आसके ननधाभरण की ऄंनतम शनि नवधानसभा ऄध्यक्ष में नननहत होती है। साधारण नवधेयक को पारितरत करने की भी

27 www.visionias.in ©Vision IAS


ऄंनतम शनि नवधानसभा में नननहत होती है। ऄनधक से ऄनधक, परितरषद नवधेयक को चार महीने की ऄवनध के निए रोक या
नविंनबत कर सकती है— पहिी बार में तीन महीने और दूसरी बार में एक महीने के निए। ऄन्य शब्दों में, नवधान परितरषद
राज्यसभा की भांनत संशोधक ननकाय तक भी नहीं होती है; यह के वि एक नविम्बकारी सदन या सिाहकार ननकाय मात्र
होती है। परितरषद बजट पर के वि चचाभ कर सकती है िेककन यह ऄनुदानों की मांग पर मतदान नहीं कर सकती (जो
नवधानसभा का ऄनन्य नवशेषानधकार होता है).
o संनवधान संशोधन नवधेयक का ऄनुसमथभन करने में परितरषद का कोइ प्रभावी मत नहीं होता है। आस संबंध में भी
नवधानसभा ही ऄनधभावी होती है। ऄंतत:, परितरषद का ऄनस्तत्व ही नवधानसभा की आच्छा पर ननभभर रहता है। नवधानसभा
की नसफारितरश के पश्चात् संसद नवधान परितरषद को समाप्त कर सकती है।
Q 100.C
 संसद के एक सत्र में कइ बैठकें होती हैं। एक कदन की प्रत्येक बैठक में दो सभाएं होती हैं, प्रात:कािीन सभा और कदन के भोजन के
बाद की सभा। संसद की बैठक को स्थगन या ऄनननश्चत काि के निए स्थगन या सत्रावसान या नवराम (िोकसभा के मामिे में)
्ारा समाप्त ककया जा सकता है।
 सत्र का कामकाज समाप्त होने के बाद, पीठासीन ऄनधकारी (िोकसभा ऄध्यक्ष या सभापनत) सदन का ऄनननश्चत काि के निए
स्थगन घोनषत करता है। ऄगिे कु छ कदनों में, राष्ट्रपनत सत्र के सत्रावसान की ऄनधसूचना जारी करता है। हािांकक, राष्ट्रपनत सत्र
के दौरान भी सदन का सत्रावसान कर सकता है। स्थगन के नवपरीत, सत्रावसान न के वि बैठक को समाप्त करता है ऄनपतु सदन
के सत्र को भी समाप्त कर देता है। आसनिए, नवकल्प c सही है।
 ऄनननश्चत काि के निए स्थगन का ऄथभ संसद की बैठक को ऄनननश्चत समयावनध हेतु स्थनगत कर देना होता है। ऄतः, जब सदन
को पुन:बैठक के कदन की घोषणा के नबना स्थनगत ककया जाता है, तो आसे ऄनननश्चत काि के निए स्थगन कहा जाता है। दूसरी
ओर, स्थगन बैठक में होने वािे कायभ को ननर्ददि समय के निए ननिंनबत कर देता है, जो कु छ घंटे, कदनों या सप्ताह का समय हो
सकता है। स्थगन एवं साथ ही ऄनननश्चत काि के निए स्थगन की शनि सदन के पीठासीन ऄनधकारी में नननहत होती है। वह
नजस नतनथ और समय तक के निए सदन की बैठक स्थनगत की गइ है ईससे पूवभ भी, और बैठक ऄनननश्चत काि के निए स्थनगत
होने के बाद ककसी भी समय सदन की बैठक का अह्वान कर सकता है।
 स्थायी सदन होने के कारण, राज्य सभा का नवघटन नहीं होता है। के वि िोकसभा का ही नवघटन होता है। सत्रावसन के
नवपरीत, नवघटन से सदन का ऄनस्तत्व ही समाप्त हो जाता है और अम चुनाव के अयोजन पश्चात् नए सदन का गठन ककया
जाता है।

Copyright © by Vision IAS


All rights are reserved. No part of this document may be reproduced, stored in a retrieval system or transmitted
in any form or by any means, electronic, mechanical, photocopying, recording or otherwise, without prior
permission of Vision IAS.
28 www.visionias.in ©Vision IAS

You might also like